Download as pdf or txt
Download as pdf or txt
You are on page 1of 150

Chuyn

S HC

Din n Ton hc
Chuyn

S HC
Ch bn

Trn Quc Nht Hn [perfectstrong]


Trn Trung Kin [Ispectorgadget]
Phm Quang Ton [Phm Quang Ton]
L Hu in Khu [Nesbit]
inh Ngc Thch [T*genie*]


c 2012 Din n Ton hc
Li gii thiu

Bn c thn mn,

S hc l mt phn mn quan trng trong ton hc gn b vi


chng ta xuyn sut qu trnh hc Ton t bc tiu hc n trung hc
ph thng. Chng ta c tip xc vi S hc bt u bng nhng
khi nim n gin nh tnh chia ht, c chung ln nht, bi chung
nh nht... gip lm quen d dng hn vi s k diu ca nhng con
s cho n nhng vn i hi nhiu t duy hn nh ng d, s
nguyn t, cc phng trnh Diophantine m ni ting nht l nh l
ln Fermat..., u u t tm vi m n v m, t cu b lp mt bi
b 4 chia ht cho 2 n Gio s thin ti Andrew Wiles (ngi gii
quyt bi ton Fermat), chng ta u c th thy c hi th ca S
hc trong .

S hc quan trng nh vy nhng l thay s chuyn vit v n li


khng nhiu nu em so vi kho tng s cc bi vit v bt ng
thc trn cc din n mng. Xut pht t s thiu ht cng nh
k nim trn mt nm Din n Ton hc khai trng trang
ch mi (16/01/2012 - 16/01/2013), nhm bin tp chng ti cng vi
nhiu thnh vin tch cc ca din n chung tay bin son mt
chuyn gi n bn c.

Chuyn l tp hp cc bi vit ring l ca cc tc gi Nguyn Mnh


Trng Dng (duongld) , Nguyn Trn Huy (yeutoan11), Nguyn
Trung Hiu (nguyentrunghieua), Phm Quang Ton (Phm Quang
Ton), Trn Nguyn Thit Qun (L Lawliet), Trn Trung Kin (Is-
pectorgadget), Nguyn nh Tng (tungc3sp)... cng s gp sc

i
ii

gin tip ca nhiu thnh vin tch cc trn Din n Ton hc nh


Nguyen Lam Thinh, nguyenta98, Karl Heinrich Marx, The
Gunner, perfectstrong...

Kin thc cp trong chuyn tuy khng mi nhng c th gip


cc bn phn no hiu su hn mt s khi nim c bn trong S hc
cng nh trao i cng cc bn nhiu dng bi tp hay v kh t cp
d n cc bi ton trong cc k thi Hc sinh gii quc gia, quc t.

Chuyn gm 7 chng. Chng 1 cp n cc khi nim v c


v Bi. S nguyn t v mt s bi ton v n c gii thiu trong
chng 2. Chng 3 ni su hn v Cc bi ton chia ht. Phng
trnh nghim nguyn, Phng trnh ng d c phc ha trong
cc chng 4 v 5. H thng d v nh l Thng d Trung Hoa
s c gi n chng ta qua chng 6 trc khi kt thc chuyn
bng Mt s bi ton s hc hay trn VMF chng 7.

Do thi gian chun b gp rt ni dung chuyn cha c u t


tht s t m cng nh c th cn nhiu sai st trong cc bi vit,
chng ti mong bn c thng cm. Mi s ng h, ng gp, ph
bnh ca c gi s l ngun ng vin tinh thn to ln cho ban bin
tp cng nh cho cc tc gi nhng phin bn cp nht sau ca
chuyn c tt hn, ng gp nhiu hn na cho kho tng hc
thut ca cng ng ton mng. Chng ti hi vng qua chuyn ny
s gip cc bn tm thm c cm hng trong s hc v thm yu v
p ca nhng con s. Mi trao i gy xin gi v a ch email :
contact@diendantoanhoc.net.

Trn trng,
Nhm bin tp Chuyn S hc.

Din n Ton hc Chuyn S hc


Mc lc

i Li gii thiu

Chng 1
1 c v Bi

1.1 c s, c s chung, c s chung ln nht 1


1.2 Bi s, bi s chung, bi s chung nh nht 4
1.3 Bi tp ngh 6

Chng 2
9 S Nguyn T

2.1 Mt s kin thc c bn v s nguyn t 9


2.2 Mt s bi ton c bn v s nguyn t 13
2.3 Bi tp 19
2.4 Ph lc: Bn nn bit 24

Chng 3
29 Bi ton chia ht
3.1 L thuyt c bn 29
3.2 Phng php gii cc bi ton chia ht 31

Chng 4
57 Phng trnh nghim nguyn

iii
iv Mc lc

4.1 Xt tnh chia ht 57


4.2 S dng bt ng thc 74
4.3 Nguyn tc cc hn, li v hn 86

Chng 5
89 Phng trnh ng d

5.1 Phng trnh ng d tuyn tnh 89


5.2 Phng trnh ng d bc cao 90
5.3 H phng trnh ng d bc nht mt n 90
5.4 Bc ca phng trnh ng d 95
5.5 Bi tp 95
5.6 ng dng nh l Euler gii phng trnh
ng d 96
5.7 Bi tp 101

Chng 6
103 H thng d v nh l Thng d Trung Hoa

6.1 Mt s k hiu s dng trong bi vit 103


6.2 H thng d 104
6.3 nh l thng d Trung Hoa 117
6.4 Bi tp ngh & gi p s 125

Chng 7
129 Mt s bi ton s hc hay trn VMF
.
7.1 m3 + 17..3n 129
7.2 c(ac + 1)2 = (5c + 2)(2c + b) 136

141 Ti liu tham kho

Din n Ton hc Chuyn S hc


Chng

1
c v Bi
1.1 c s, c s chung, c s chung
ln nht 1
1.2 Bi s, bi s chung, bi s chung
nh nht 4
1.3 Bi tp ngh 6

Nguyn Mnh Trng Dng (duongld)


Nguyn Trn Huy (yeutoan11)

c v bi l 2 khi nim quan trng trong chng trnh s hc THCS.


Chuyn ny s gii thiu nhng khi nim v tnh cht c bn v
c, c s chung, c chung ln nht, bi, bi s chung, bi chung
nh nht. Mt s bi tp ngh v cc vn ny cng s c
cp n cui bi vit.

1.1 c s, c s chung, c s chung ln nht


Trong phn ny, chng ti s trnh by mt s khi nim v c s,
c s chung v c s chung ln nht km theo mt vi tnh cht ca
chng. Mt s bi tp v d cho bn c tham kho cng s c a
ra.

1.1.1 nh ngha
nh ngha 1.1 S t nhin d 6= 0 c gi l mt c s ca s t
nhin a khi v ch khi a chia ht cho d. Ta ni d chia ht a, k hiu d|a.
Tp hp cc c ca a l: U (a) = {d N : d|a}. 4

1
2 1.1. c s, c s chung, c s chung ln nht

Tnh cht 1.1 Nu U (a) = {1; a} th a l s nguyn t. 


nh ngha 1.2 Nu U (a) v U (b) c nhng phn t chung th nhng
phn t gi l c s chung ca a v b. Ta k hiu:
U SC(a; b) = {d N : (d|a) (d|b)}
= {d N : (d U (a)) (d U (b))}.

Tnh cht 1.2 Nu U SC(a; b) = {1} th a v b nguyn t cng nhau.


nh ngha 1.3 S d N c gi l c s chung ln nht ca a v b
(a; b Z) khi d l phn t ln nht trong tp U SC(a; b). K hiu c
chung ln nht ca a v b l U CLN (a; b), (a; b) hay gcd(a; b). 4

1.1.2 Tnh cht


Sau y l mt s tnh cht ca c chung ln nht:
Nu (a1 ; a2 ; . . . .; an ) = 1 th ta ni cc s a1 ; a2 ; . . . ; an nguyn
t cng nhau.
Nu (am ; ak ) = 1, m 6= k, {m; k} {1; 2; . . . ; n} th ta ni cc
a1 ; a2 ; . . . ; an i mt nguyn t cng nhau.
 
a b (a; b)
c U SC(a; b) th ; = .
c c c
 
a b
d = (a; b) ; = 1.
d d
(ca; cb) = c(a; b).
(a; b) = 1 v b|ac th b|c.
(a; b) = 1 v (a; c) = 1 th (a; bc) = 1.
(a; b; c) = ((a; b); c).
Cho a > b > 0
Nu a = b.q th (a; b) = b.
Nu a = bq + r(r 6= 0) th (a; b) = (b; r).

Din n Ton hc Chuyn S hc


1.1. c s, c s chung, c s chung ln nht 3

1.1.3 Cch tm c chung ln nht bng thut ton Euclide


tm (a; b) khi a khng chia ht cho b ta dng thut ton Euclide
sau:

 a = b.q + r1 th (a; b) = (b; r1 ).

 b = r1 .q1 + r2 th (b; r1 ) = (r1 ; r2 ).

 rn2 = rn1 .qn1 + rn th (rn2 ; rn1 ) = (rn1 ; rn ).

 rn1 = rn .qn th (rn1 ; rn ) = rn .

 (a; b) = rn .

 (a; b) l s d cui cng khc 0 trong thut ton Euclide.

1.1.4 Bi tp v d
V d 1.1. Tm (2k 1; 9k + 4), k N . 4
Li gii. Ta t d = (2k 1; 9k + 4). Theo tnh cht v c s chung
ta c d|2k 1 v d|9k + 4. Tip tc p dng tnh cht v chia ht ta li
c d|9(2k 1) v d|2(9k + 4). Suy ra d|2(9k + 4) 9(2k 1) hay d|17.
Vy (2k 1; 9k + 4) = 1. 

V d 1.2. Tm (123456789; 987654321). 4


Li gii. t b = 123456789; a = 987654321. Ta nhn thy a v b u
chia ht cho 9.
Ta li c :
a + b = 1111111110
1010 10
= . (1.1)
9
9a + 9b = 1010 10
Mt khc :

10b + a = 9999999999
(1.2)
= 1010 1.

Chuyn S hc Din n Ton hc


4 1.2. Bi s, bi s chung, bi s chung nh nht

Tr (1.2) v (1.1) v theo v ta c b8a = 9. Do nu t d = (a; b)


.
th 9..d.
M a v b u chia ht cho 9, suy ra d = 9. 

Da vo thut ton Euclide, ta c li gii khc cho V d 1.2 nh sau :


Li gii.  987654321 = 123456789.8+9 th (987654321; 123456789) =
(123456789; 9).

 123456789 = 9.1371421.

 (123456789; 987654321) = 9. 

1
V d 1.3. Chng minh rng dy s An = n(n + 1), n N cha
2
nhng dy s v hn nhng s i mt nguyn t cng nhau. 4
Li gii. Gi s trong dy ang xt c k s i mt nguyn t cng
nhau l t1 = 1; t2 = 3; . . . ; tk = m(m N ). t a = t1 t2 . . . tk . Xt s
hng t2a+1 trong dy An :

1
t2a+1 = (2a + 1)(2a + 2)
2
= (a + 1)(2a + 1)
tk

Mt khc ta c (a + 1; a) = 1 v (2a + 1; a) = 1 nn (t2a+1 ; a) = 1.


Do t2a+1 nguyn t cng nhau vi tt c k s {t1 ; t2 ; . . . tk }. Suy ra
dy s An cha v hn nhng s i mt nguyn t cng nhau. 

1.2 Bi s, bi s chung, bi s chung nh nht


Tng t nh cu trc trnh by phn trc, trong phn ny
chng ti cng s a ra nhng nh ngha, tnh cht c bn ca bi
s, bi s chung, bi s chung nh nht v mt s bi tp v d minh
ha.

Din n Ton hc Chuyn S hc


1.2. Bi s, bi s chung, bi s chung nh nht 5

1.2.1 nh ngha
nh ngha 1.4 S t nhin m c gi l mt bi s ca a 6= 0 khi
v ch khi m chia ht cho a hay a l mt c s ca m. 4

Nhn xt. Tp hp cc bi s ca a 6= 0 l: B(a) = {0; a; 2a; . . . ; ka}, k


Z.

nh ngha 1.5 S t nhin m c gi l mt bi s ca a 6= 0 khi


v ch khi m chia ht cho a hay a l mt c s ca m 4

nh ngha 1.6 Nu 2 tp B(a) v B(b) c phn t chung th cc phn


t chung gi l bi s chung ca a v b. Ta k hiu bi s chung
ca a v b: BSC(a; b).

nh ngha 1.7 S m 6= 0 c gi l bi chung nh nht ca a v


b khi m l phn t dng nh nht trong tp BSC(a; b). K hiu :
BCN N (a; b), [a; b] hay lcm(a; b). 4

1.2.2 Tnh cht


Mt s tnh cht ca bi chung ln nht:
 
M M
Nu [a; b] = M th ; = 1.
a b

[a; b; c] = [[a; b]; c].

[a; b].(a; b) = a.b.

1.2.3 Bi tp v d
V d 1.4. Tm [n; n + 1; n + 2]. 4
Li gii. t A = [n; n + 1] v B = [A; n + 2]. p dng tnh cht
[a; b; c] = [[a; b]; c], ta c: B = [n; n + 1; n + 2].
D thy (n; n + 1) = 1, suy ra [n; n + 1] = n(n + 1).

Chuyn S hc Din n Ton hc


6 1.3. Bi tp ngh

a.b
Li p dng tnh cht [a; b] = th th
(a; b)

n(n + 1)(n + 2)
[n; n + 1; n + 2] =
(n(n + 1); n + 2)
.
Gi d = (n(n + 1); n + 2). Do (n + 1; n + 2) = 1 nn

d = (n; n + 2)
= (n; 2).

Xt hai trng hp:


n(n + 1)(n + 2)
Nu n chn th d = 2, suy ra [n; n + 1; n + 2] = .
2
Nu n l th d = 1, suy ra [n; n + 1; n + 2] = n(n + 1)(n + 2) . 

V d 1.5. Chng minh rng [1; 2; . . . 2n] = [n + 1; n + 2; . . . ; 2n]. 4


Li gii. Ta thy c trong k s nguyn lin tip c mt v ch mt s
chia ht cho k. Do bt trong cc s {1; 2; . . . ; 2n} u l c ca mt
s no trong cc s {n + 1; n + 2; . . . ; 2n}. Do [1; 2; . . . n; 2n] =
[n + 1; n + 2; . . . ; 2n]. 

1.3 Bi tp ngh
Thay cho li kt, chng ti xin gi n bn c mt s bi tp ngh
luyn tp nhm gip cc bn quen hn vi cc khi nim v cc
tnh cht trnh by trong chuyn .

Bi 1. a. Cho A = 5a + 3b; B = 13a + 8b(a; b N ) chng minh


(A; B) = (a; b).
b. Tng qut A = ma+nb; B = pa+qb tha mn |mq np| =
1 vi a, b, m, n, p, q N . Chng minh (A; B) = (a; b).

Bi 2. Tm (6k + 5; 8k + 3)(k N).

Din n Ton hc Chuyn S hc


1.3. Bi tp ngh 7

Bi 3. T cc ch s 1; 2; 3; 4; 5; 6 thnh lp tt c s c su ch s
(mi s ch vit mt ln). Tm U CLN ca tt c cc s .
n(n + 1)
Bi 4. Cho A = 2n + 1; B = (n N ). Tm (A; B).
2
Bi 5. a. Chng minh rng trong 5 s t nguyn lin tip bao gi
cng chn c mt s nguyn t cng nhau vi cc s
cn li.
b. Chng minh rng trong 16 s nguyn lin tip bao gi cng
chn c mt s nguyn t cng nhau vi cc s cn li.

Bi 6. Cho 1 m n(m; n N).


n n
a. Chng minh rng (22 1; 22 + 1) = 1.
b. Tm (2m 1; 2n 1).

Bi 7. Cho m, n N vi (m, n) = 1. Tm (m2 + n2 ; m + n).

Bi 8. Cho A = 2n +3; B = 2n+1 +3n+1 (n N ); C = 2n+2 +3n+2 (n


N ). Tm (A; B) v (A; C).

Bi 9. Cho su s nguyn dng a; b; a0 ; b0 ; d; d0 sao cho (a; b) = d; (a0 ; b0 ) =


d0 . Chng minh rng (aa0 ; bb0 ; ab0 ; a0 b) = dd0 .
1
Bi 10. Chng minh rng dy s Bn = n(n + 1)(n + 2)(n N ) cha
6
v hn nhng s nguyn t cng nhau.

Bi 11. Chng minh rng dy s 2n 3 vi mi n N v n 2 cha


dy s v hn nhng s nguyn t cng nhau.

Bi 12. Chng minh dy Mersen Mn = 2n 1(n N ) cha dy s v


hn nhng s nguyn t cng nhau.
n
Bi 13. Chng minh rng dy Fermat Fn = 22 + 1(n N) l dy s
nguyn t cng nhau.
n
Bi 14. Cho n N; n > 1 v 2n 2 chia ht cho n. Tm (22 ; 2n 1).

Chuyn S hc Din n Ton hc


8 1.3. Bi tp ngh

21n + 1
Bi 15. Chng minh rng vi mi n N, phn s ti gin.
14n + 3
Bi 16. Cho ba s t nhin a; b; c i mt nguyn t cng nhau. Chng
minh rng (ab + bc + ca; abc) = 1.

Bi 17. Cho a; b N . Chng minh rng tn ti v s n N sao cho


(a + n; b + n) = 1.

Bi 18. Gi s m; n N(m n) tha mn (199k1; m) = (19931; n).


Chng minh rng tn ti t(t N) sao cho m = 1993t .n.
 m 
a 1
Bi 19. Chng minh rng nu a; m N; a > 1 th ;a 1 =
a1
(m; a 1).

Bi 20. Tm s nguyn dng n nh nht cc phn s sau ti gin:


1
a. ,
n1996
+ 1995n + 2
2
b. 1996 ,
n + 1995n + 3
1994
c. 1996 ,
n + 1995n + 1995
1995
d. 1996 .
n + 1995n + 1996
Bi 21. Cho 20 s t nhin khc 0 l a1 ; a2 ; . . . an c tng bng S
v U CLN bng d. Chng minh rng U CLN ca S a1 ; S
a2 ; . . . ; S an bng tch ca d vi mt c no ca n 1.

Din n Ton hc Chuyn S hc


Chng

2
S Nguyn T
2.1 Mt s kin thc c bn v s
nguyn t 9
2.2 Mt s bi ton c bn v s nguyn
t 13
2.3 Bi tp 19
2.4 Ph lc: Bn nn bit 24

Nguyn Trung Hiu (nguyentrunghieua)


Phm Quang Ton (Phm Quang Ton)

2.1 Mt s kin thc c bn v s nguyn t


2.1.1 nh ngha, nh l c bn
nh ngha 2.1 S nguyn t l nhng s t nhin ln hn 1, ch c 2
c s l 1 v chnh n. 4

nh ngha 2.2 Hp s l s t nhin ln hn 1 v c nhiu hn 2


c. 4

Nhn xt. Cc s 0 v 1 khng phi l s nguyn t cng khng phi


l hp s. Bt k s t nhin ln hn 1 no cng c t nht mt c
s nguyn t.

nh l 2.1 Dy s nguyn t l dy s v hn. 

9
10 2.1. Mt s kin thc c bn v s nguyn t

Chng minh. Gi s ch c hu hn s nguyn t l p1 ; p2 ; p3 ; ...; pn ;


trong pn l s ln nht trong cc nguyn t.
Xt s N = p1 p2 ...pn + 1 th N chia cho mi s nguyn t pi (i = 1, n)
u d 1 (*)
Mt khc N l mt hp s (v n ln hn s nguyn t ln nht l pn )
do N phi c mt c nguyn t no , tc l N chia ht cho mt
trong cc s pi (**).
Ta thy (**) mu thun (*). Vy khng th c hu hn s nguyn t.

nh l 2.2 Mi s t nhin ln hn 1 u phn tch c ra tha


s nguyn t mt cch duy nht (khng k th t cc tha s). 

Chng minh. * Mi s t nhin ln hn 1 u phn tch c ra tha


s nguyn t:
Tht vy: gi s iu khng nh trn l ng vi mi s m tho mn:
1 < m < n ta chng minh iu ng n n.
Nu n l nguyn t, ta c iu phi chng minh.
Nu n l hp s, theo nh ngha hp s, ta c: n = a.b (vi a, b < n)
Theo gi thit quy np: a v b l tch cc tha s nh hn n nn n l
tch cu cc tha s nguyn t.
* S phn tch l duy nht:
Gi s mi s m < n u phn tch c ra tha s nguyn t mt
cch duy nht, ta chng minh iu ng n n:
Nu n l s nguyn t th ta c iu phi chng minh. Nu n l hp
s: Gi s c 2 cch phn tch n ra tha s nguyn t khc nhau:

n = p.q.r....
n = p0 .q 0 .r0 ....

Trong p, q, r..... v p0 , q 0 , r0 .... l cc s nguyn t v khng c s


nguyn t no cng c mt trong c hai phn tch (v nu c s
tho mn iu kin nh trn, ta c th chia n cho s lc thng
s nh hn n, thng ny c hai cch phn tch ra tha s nguyn t
khc nhau, tri vi gi thit ca quy np).
Khng mt tnh tng qut, ta c th gi thit p v p0 ln lt l cc s
nguyn t nh nht trong phn tch th nht v th hai.
V n l hp s nn n > p2 v n > p02 . Do p 6= p n > p.p0

Din n Ton hc Chuyn S hc


2.1. Mt s kin thc c bn v s nguyn t 11

Xt m = n pp0 < n c phn tch ra tha s nguyn t mt cch


duy nht ta thy:

p|n p|n pp0 hay p|m


Khi phn tch ra tha s nguyn t ta c: m = n pp0 = p0 p.P.Q... vi
P, Q P ( P l tp cc s nguyn t).
pp0 |n pp0 |p.q.r... p|q.r... p l c nguyn t ca q.r...
M p khng trng vi mt tha s no trong q, r... (iu ny tri vi
ga thit quy np l mi s nh hn n u phn tch c ra tha s
nguyn t mt cch duy nht).
Vy, iu gi s khng ng. nh l c chng minh. 

2.1.2 Cch nhn bit mt s nguyn t


Cch 1
Chia s ln lt cho cc nguyn t t nh n ln: 2; 3; 5; 7...

Nu c mt php chia ht th s khng nguyn t.

Nu thc hin php chia cho n lc thng s nh hn s chia m cc


php chia vn c s d th s l nguyn t.

Cch 2
Mt s c hai c s ln hn 1 th s khng phi l s nguyn t.

Cho hc sinh lp 6 hc cch nhn bit 1 s nguyn t bng phng


php th nht (nu trn), l da vo nh l c bn:

c
s nguyn t nh nht ca mt hp s A l mt s khng vt
qu A.

Vi quy tc trn trong mt khon thi gian ngn, vi cc du hiu chia


ht th ta nhanh chng tr li c mt s c hai ch s no l

Chuyn S hc Din n Ton hc


12 2.1. Mt s kin thc c bn v s nguyn t

nguyn t hay khng.

H qu
2.1 Nu c s A > 1 khng c mt c s nguyn t no t
2 n A th A l mt nguyn t. 

2.1.3 S cc c s v tng cc c s ca 1 s
Gi s: A = px1 1 .px2 2 ......pn xn ; trong : pi P; xi N; i = 1, n

Tnh cht 2.1 S cc c s ca A tnh bng cng thc:

T (A) = (x1 + 1)(x2 + 1).....(xn + 1)

V d 2.1. 30 = 2.3.5 th T (A) = (1 + 1)(1 + 1)(1 + 1) = 8. Kim tra:


(30) = {1; 2; 3; 5; 6; 10; 15; 30} nn (30) c 8 phn t. 4

Tnh cht 2.2 Tng cc c mt s ca A tnh bng cng thc:


n
Y pxi +1 1
i
(A) =
pi 1
i=1

2.1.4 Hai s nguyn t cng nhau


nh ngha 2.3 Hai s t nhin c gi l nguyn t cng nhau khi
v ch khi chng c c chung ln nht (CLN) bng 1. 4

Tnh cht 2.3 Hai s t nhin lin tip lun nguyn t cng nhau. 

Tnh cht 2.4 Hai s nguyn t khc nhau lun nguyn t cng nhau.

Tnh cht 2.5 Cc s a, b, c nguyn t cng nhau khi v ch khi (a, b, c)


= 1. 

nh ngha 2.4 Nhiu s t nhin c gi l nguyn t snh i khi


chng i mt nguyn t cng nhau. 4

Din n Ton hc Chuyn S hc


2.2. Mt s bi ton c bn v s nguyn t 13

2.1.5 Mt s nh l c bit
nh l 2.3 (Dirichlet) Tn ti v s s nguyn t p c dng:
p = ax + b (x, a, b N, a, b l 2 s nguyn t cng nhau). 

Vic chng minh nh l ny kh phc tp, tr mt s trng hp c


bit, chng hn c v s s nguyn t dng: 2x 1; 3x 1; 4x + 3; 6x +
5; . . .

nh l 2.4 (Tchebycheff-Betrand) Trong khong t s t nhin


n n s t nhin 2n c t nht mt s nguyn t (n > 2). 

nh l 2.5 (Vinogradow) Mi s l ln hn 33 l tng ca 3 s


nguyn t. 

2.2 Mt s bi ton c bn v s nguyn t


2.2.1 C bao nhiu s nguyn t dng ax + b
V d 2.2. Chng minh rng: c v s s nguyn t c dng 3x 1.4
Li gii. Mi s t nhin khng nh hn 2 c 1 trong 3 dng: 3x; 3x+1
hoc 3x 1

Nhng s c dng 3x (vi x > 1) l hp s

Xt 2 s c dng 3x + 1: l s 3m + 1 v s 3n + 1.
Xt tch (3m + 1)(3n + 1) = 9mn + 3m + 3n + 1. Tch ny c
dng: 3x + 1

Ly mt s nguyn t p bt c dng 3x 1, ta lp tch ca p


vi tt c cc s nguyn t nh hn p ri tr i 1 ta c: M =
2.3.5.7....p 1 = 3(2.5.7....p) 1 th M c dng 3x 1.
C 2 kh nng xy ra:

1. Kh nng 1: M l s nguyn t, l s nguyn t c dng


3x 1 > p, bi ton c chng minh.

Chuyn S hc Din n Ton hc


14 2.2. Mt s bi ton c bn v s nguyn t

2. Kh nng 2: M l hp s: Ta chia M cho 2, 3, 5, ...., p u tn


ti mt s d khc 0 nn cc c nguyn t ca M u ln
hn p, trong cc c ny khng c s no c dng 3x + 1 (
chng minh trn). Do t nht mt trong cc c nguyn
t ca M phi c dng 3x (hp s) hoc 3x + 1

V nu tt c c dng 3x + 1 th M phi c dng 3x + 1 ( chng


minh trn). Do , t nht mt trong cc c nguyn t ca M
phi c dng 3x 1, c ny lun ln hn p.

Vy: C v s s nguyn t dng 3x 1. 

V d 2.3. Chng minh rng: C v s s nguyn t c dng 4x + 3.4


Li gii. Nhn xt. Cc s nguyn t l khng th c dng 4x hoc
4x + 2. Vy chng ch c th tn ti di 1 trong 2 dng 4x + 1 hoc
4x + 3.
Ta s chng minh c v s s nguyn t c dng 4x + 3.

Xt tch 2 s c dng 4x + 1 l: 4m + 1 v 4n + 1.
Ta c: (4m+1)(4n+1) = 16mn+4m+4n+1 = 4(4mn+m+n)+1.
Vy tch ca 2 s c dng 4x + 1 l mt s cng c dng 4x + 1.

Ly mt s nguyn t p bt k c dng 4x + 3, ta lp tch ca 4p


vi tt c cc s nguyn t nh hn p ri tr i 1 khi ta c:
N = 4(2.3.5.7.....p) 1. C 2 kh nng xy ra

1. N l s nguyn t N = 4(2.3.5.7....p) 1 c dng 4x 1.


Nhng s nguyn t c dng 4x 1 cng chnh l nhng s
c dng 4x + 3 v bi ton c chng minh.
2. N l hp s. Chia N cho 2, 3, 5, ...., p u c cc s d
khc 0. Suy ra cc c nguyn t ca N u ln hn p.

Cc c ny khng th c dng 4x hoc 4x + 2 (v l hp s).


Cng khng th ton cc c c dng 4x + 1 v nh th N phi
c dng 4x + 1. Nh vy trong cc c nguyn t ca N c t
nht 1 c c dng 4x 1 m c ny hin nhin ln hn p.

Din n Ton hc Chuyn S hc


2.2. Mt s bi ton c bn v s nguyn t 15

Vy: C v s s nguyn t c dng 4x 1 (hay c dng 4x + 3). 

Trn y l mt s bi ton chng minh n gin ca nh l Dirichlet:


C v s s nguyn t dng ax + b trong a, b, x N, (a, b) = 1.

2.2.2 Chng minh s nguyn t


V d 2.4. Chng minh rng: (p 1)! chia ht cho p nu p l hp s,
khng chia ht cho p nu p l s nguyn t. 4
Li gii. Xt trng hp p l hp s: Nu p l hp s th p l tch
ca cc tha s nguyn t nh hn p v s m cc lu tha ny
khng th ln hn s m ca chnh cc lu tha y cha trong
.
(p 1)!. Vy: (p 1)!..p (pcm).

Xt trng hp p l s nguyn t: V p P p nguyn t cng


nhau vi mi tha s ca (p 1)! (pcm). 

V d 2.5. Cho 2m 1 l s nguyn t. Chng minh rng m cng l


s nguyn t. 4
Li gii. Gi s m l hp s m = p.q (p, q N; p, q > 1)
Khi : 2m 1 = 2pq 1 = (2p )q 1 = (2p 1)((2p )q1 +(2p )q2 +.....+1)
v p > 1 2p 1 > 1 v (2p )q1 + (2p )q2 + ..... + 1 > 1
Dn n 2m 1 l hp s :tri vi gi thit 2m 1 l s nguyn t.
Vy m phi l s nguyn t (pcm) 

V d 2.6. Chng minh rng: mi c nguyn t ca 1994! 1 u ln


hn 1994. 4
Li gii. Gi p l c s nguyn t ca 1994! 1
. .
Gi s p 1994 1994.1993.....3.2.1..p 1994!..p.
. .
M 1994! 1..p 1..p (v l)
Vy: p > 1994 (pcm). 

V d 2.7. Chng minh rng: n >2 th gia n v n! c t nht 1 s


nguyn t (t suy ra c v s s nguyn t). 4

Chuyn S hc Din n Ton hc


16 2.2. Mt s bi ton c bn v s nguyn t

Li gii. V n > 2 nn k = n! 1 > 1, do k c t nht mt c s


nguyn t p. Tng t bi tp 3, ta chng minh c mi c nguyn
t p ca k u ln hn k.
Vy: p > n n < p < n! 1 < n! (pcm) 

2.2.3 Tm s nguyn t tha mn iu kin cho trc


V d 2.8. Tm tt c cc gi tr ca s nguyn t p : p + 10 v
p + 14 cng l s nguyn t. 4
Li gii. Nu p = 3 th p + 10 = 3 + 10 = 13 v p + 14 = 3 + 14 = 17
u l cc s nguyn t nn p = 3 l gi tr cn tm.
Nu p > 3 p c dng 3k + 1 hoc dng 3k 1
.
Nu p = 3k + 1 th p + 14 = 3k + 15 = 3(k + 5)..3
.
Nu p = 3k 1 th p + 10 = 3k + 9 = 3(k + 3)..3

Vy nu p > 3 th hoc p + 10 hoc p + 14 l hp s : khng tha mn


bi. Vy p = 3. 

V d 2.9. Tm k N trong 10 s t nhin lin tip:

k + 1; k + 2; k + 3; ....k + 10

c nhiu s nguyn t nht. 4


Li gii. Nu k = 0: t 1 n 10 c 4 s nguyn t: 2; 3; 5; 7.
Nu k = 1: t 2 n 11 c 5 s nguyn t: 2; 3; 5; 7; 11.
Nu k > 1: t 3 tr i khng c s chn no l s nguyn t. Trong 5
s l lin tip, t nht c 1 s l bi s ca 3 do , dy s c t hn 5
s nguyn t.
Vy vi k = 1, dy tng ng: k + 1; k + 2, .....k + 10 c cha nhiu s
nguyn t nht (5 s nguyn t). 

V d 2.10. Tm tt c cc s nguyn t p : 2p +p2 cng l s nguyn


t. 4
Li gii. Xt 3 trng hp:

Din n Ton hc Chuyn S hc


2.2. Mt s bi ton c bn v s nguyn t 17

p = 2 2p + p2 = 22 + 22 = 8 6 P

p = 3 2p + p2 = 23 + 32 = 17 P
.
p > 3 p 6 ..3. Ta c 2p + p2 = (p2 1) + (2p + 1).
. .
V p l 2p + 1..3 v p2 1 = (p + 1)(p 1)..3 2p + p2 6 P

Vy c duy nht 1 gi tr p = 3 tho mn. 

V d 2.11. Tm tt c cc s nguyn t p sao cho: p|2p + 1. 4


Li gii. V p P : p|2p + 1 p > 2 (2; p) = 1
Theo nh l Fermat, ta c: p|2p1 1. M

p|2p + 1 p|2(2p1 1) + 3 p|3 p = 3

Vy: p = 3. 

2.2.4 Nhn bit s nguyn t


V d 2.12. Nu p l s nguyn t v 1 trong 2 s 8p + 1 v 8p 1 l
s nguyn t th s cn li l s nguyn t hay hp s? 4
Li gii. Nu p = 2 8p + 1 = 17 P; 8p 1 = 15 6 P

Nu p = 3 8p 1 = 23 P; 8p 1 = 25 6 P

Nu p > 3, xt 3 s t nhin lin tip: 8p 1; 8p v 8p + 1. Trong


3 s ny t c 1 s chia ht cho 3. Nn mt trong hai s 8p + 1
v 8p 1 chia ht cho 3.
Kt lun: Nu p P v 1 trong 2 s 8p + 1 v 8p 1 l s nguyn t
th s cn li phi l hp s. 

V d 2.13. Nu p 5 v 2p + 1 l cc s nguyn t th 4p + 1 l
nguyn t hay hp s? 4
Li gii. Xt 3 s t nhin lin tip: 4p; 4p + 1; 4p + 2. Trong 3 s t
c mt s l bi ca 3.
M p 5; p P nn p c dng 3k + 1 hoc 3k + 2
.
Nu p = 3k + 1 th 2p + 1 = 6k + 3..3: (tri vi gi thit)

Chuyn S hc Din n Ton hc


18 2.2. Mt s bi ton c bn v s nguyn t

.
Nu p = 3k +2. Khi 4p+1 = 4(3k +2)+1 = 12k +9..3 4p+1
l hp s 

V d 2.14. Trong dy s t nhin c th tm c 1997 s lin tip


nhau m khng c s nguyn t no hay khng ? 4
..
Li gii. Chn dy s: (a ) : a = 1998! + i + 1 (i = 1, 1997) a .i +
i i i
1 i = 1, 1997
Nh vy: Dy s a1 ; a2 ; a3 ; .....a1997 gm c 1997 s t nhin lin tip
khng c s no l s nguyn t. 

V d 2.15 (Tng qut bi tp 2.14). Chng minh rng c th tm


c 1 dy s gm n s t nhin lin tip (n > 1) khng c s no
l s nguyn t ? 4
..
Li gii. Ta chn dy s sau: (a ) : a = (n + 1)! + i + 1 a .i + 1 i =
i i i
1, n.
Bn c hy t chng minh dy (ai ) trn s gm c n s t nhin
lin tip trong khng c s no l s nguyn t c. 

2.2.5 Cc dng khc


V d 2.16. Tm 3 s nguyn t sao cho tch ca chng gp 5 ln tng
ca chng. 4
Li gii. Gi 3 s nguyn t phi tm l a, b, c. Ta c: abc = 5(a + b +
.
c) abc..5
V a, b, c c vai tr bnh ng nn khng mt tnh tng qut, gi s:
.
a..5 a = 5
= 5(5 + b + c)
Khi : 5bc  5 + b + c = bc (c 1)(b 1) = 6
b1=1 b=2
chn
 c1=6  c=7

Do vy:
b1=2 b=3
loi
c1=3 c=4
Vy b s (a; b; c) cn tm l hon v ca (2; 5; 7). 

V d 2.17. Tm p, q P sao cho p2 = 8q + 1. 4

Din n Ton hc Chuyn S hc


2.3. Bi tp 19

Li gii. Ta c:

p2 = 8q + 1 8q = p2 1 = (p + 1)(p 1) (2.1)

Do p2 = 8q + 1 : l p2 : l p : l. t p = 2k + 1.
Thay vo (2.1) ta c:

8q = 2k(2k + 2) 2q = k(k + 1) (2.2)

Nu q = 2 4 = k(k + 1) khng tm c k N
Vy q > 2. V q P (2, q) = 1.
T (2.2) ta c:

a) k = 2 v q = k + 1 k = 2; q = 3. Thay kt qu trn vo (2.2)


ta c: p = 2.2 + 1 = 5

b) q = k v 2 = k + 1 q = 1 :loi.

Vy (q; p) = (5; 3). 

2.3 Bi tp
2.3.1 Bi tp c hng dn
Bi 1. Ta bit rng c 25 s nguyn t nh hn 100. Tng ca 25 s
nguyn t nh hn 100 l s chn hay s l?
HD :Trong 25 s nguyn t nh hn 100 c cha mt s nguyn
t chn duy nht l 2, cn 24 s nguyn t cn li l s l. Do
tng ca 25 s nguyn t l s chn.

Bi 2. Tng ca 3 s nguyn t bng 1012. Tm s nguyn t nh nht


trong ba s nguyn t .
HD: V tng ca 3 s nguyn t bng 1012, nn trong 3 s
nguyn t tn ti t nht mt s nguyn t chn. M s
nguyn t chn duy nht l 2 v l s nguyn t nh nht. Vy
s nguyn t nh nht trong 3 s nguyn t l 2.

Chuyn S hc Din n Ton hc


20 2.3. Bi tp

Bi 3. Tng ca 2 s nguyn t c th bng 2003 hay khng? V sao?


HD: V tng ca 2 s nguyn t bng 2003, nn trong 2 s
nguyn t tn ti 1 s nguyn t chn. M s nguyn t
chn duy nht l 2. Do s nguyn t cn li l 2001. Do
2001 chia ht cho 3 v 2001 > 3. Suy ra 2001 khng phi l s
nguyn t.

Bi 4. Tm s nguyn t p, sao cho p + 2; p + 4 cng l cc s nguyn


t.

Bi 5. Cho p v p + 4 l cc s nguyn t (p > 3). Chng minh rng


p + 8 l hp s.
HD: V p l s nguyn t v p > 3, nn s nguyn t p c 1
trong 2 dng:
.
Nu p = 3k + 2 th p + 4 = 3k + 6 = 3(k + 2) p + 4..3 v
p + 4 > 3. Do p + 4 l hp s: tri bi.
.
Nu p = 3k + 1 th p + 8 = 3k + 9 = 3(k + 3) p + 8..3 v
p + 8 > 3. Do p + 8 l hp s.

Bi 6. Chng minh rng mi s nguyn t ln hn 2 u c dng 4n+1


hoc 4n 1.

Bi 7. Tm s nguyn t, bit rng s bng tng ca hai s nguyn


t v bng hiu ca hai s nguyn t.
HD: Gi s a, b, c, d, e l cc s nguyn t v d > e. Theo
bi:
a = b + c = d e ()
T (*) a > 2 nn a l s nguyn t l b + c; d e l s l.
Do b, d l cc s nguyn t b, d l s l c, e l s chn.
c = e = 2 (do c, el s nguyn t) a = b + 2 = d 2
d = b + 4.
Vy ta cn tm s nguyn t b sao cho b + 2 v b + 4 cng l
cc s nguyn t.

Din n Ton hc Chuyn S hc


2.3. Bi tp 21

Bi 8. Tm tt c cc s nguyn t x, y sao cho: x2 6y 2 = 1.

Bi 9. Cho p v p + 2 l cc s nguyn t (p > 3). Chng minh rng


.
p + 1..6.

2.3.2 Bi tp khng c hng dn


Bi 1. Tm s nguyn t p sao cho cc s sau cng l s nguyn t:

a) p + 2 v p + 10.
b) p + 10 v p + 20.
c) p + 10 v p + 14.
d) p + 14 v p + 20.
e) p + 2 v p + 8.
f) p + 2 v p + 14.
g) p + 4 v p + 10.
h) p + 8 v p + 10.

Bi 2. Tm s nguyn t p sao cho cc s sau cng l s nguyn t:

a) p + 2, p + 8, p + 12, p + 14
b) p + 2, p + 6, p + 8, p + 14
c) p + 6, p + 8, p + 12, p + 14
d) p + 2, p + 6, p + 8, p + 12, p + 14
e) p + 6, p + 12, p + 18, p + 24
f) p + 18, p + 24, p + 26, p + 32
g) p + 4, p + 6, p + 10, p + 12, p + 16

Bi 3. Cho trc s nguyn t p > 3 tha

a) p + 4 P. Chng minh rng: p + 8 l hp s.


b) 2p + 1 P. Chng minh rng: 4p + 1 l hp s.
c) 10p + 1 P. Chng minh rng: 5p + 1 l hp s.

Chuyn S hc Din n Ton hc


22 2.3. Bi tp

d) p + 8 P. Chng minh rng: p + 4 l hp s.


e) 4p + 1 P. Chng minh rng: 2p + 1 l hp s.
f) 5p + 1 P. Chng minh rng: 10p + 1 l hp s.
g) 8p + 1 P. Chng minh rng: 8p 1 l hp s.
h) 8p 1 P. Chng minh rng: 8p + 1 l hp s.
i) 8p2 1 P. Chng minh rng: 8p2 + 1 l hp s.
j) 8p2 + 1 P. Chng minh rng: 8p2 1 l hp s.

Bi 4. Chng minh rng:


.
a) Nu p v q l hai s nguyn t ln hn 3 th p2 q 2 ..24.
b) Nu a, a + k, a + 2k(a, k N ) l cc s nguyn t ln hn
.
3 th k ..6.

Bi 5. a) Mt s nguyn t chia cho 42 c s d r l hp s. Tm s


d r.
b) Mt s nguyn t chia cho 30 c s d r. Tm s d r bit
rng r khng l s nguyn t.

Bi 6. Tm s nguyn t c ba ch s, bit rng nu vit s theo


th t ngc li th ta c mt s l lp phng ca mt s
t nhin.

Bi 7. Tm s t nhin c 4 ch s, ch s hng nghn bng ch s


hng n v, ch s hng trm bng ch s hng chc v s
vit c di dng tch ca 3 s nguyn t lin tip.

Bi 8. Tm 3 s nguyn t l cc s l lin tip.

Bi 9. Tm 3 s nguyn t lin tip p, q, r sao cho p2 + q 2 + r2 P.

Bi 10. Tm tt c cc b ba s nguyn t a, b, c sao cho abc < ab +


bc + ca.

Bi 11. Tm 3 s nguyn t p, q, r sao cho pq + q p = r.

Din n Ton hc Chuyn S hc


2.3. Bi tp 23

Bi 12. Tm cc s nguyn t x, y, z tho mn xy + 1 = z.

Bi 13. Tm s nguyn t abcd tha ab, ac l cc s nguyn t v b2 =


cd + b c.

Bi 14. Cho cc s p = bc + a, q = ab + c, r = ca + b(a, b, c N ) l


cc s nguyn t. Chng minh rng 3 s p, q, r c t nht hai s
bng nhau.

Bi 15. Tm tt c cc s nguyn t x, y sao cho:

a) x2 12y 2 = 1
b) 3x2 + 1 = 19y 2
c) 5x2 11y 2 = 1
d) 7x2 3y 2 = 1
e) 13x2 y 2 = 3
f) x2 = 8y + 1

Bi 16. Chng minh rng iu kin cn v p v 8p2 + 1 l cc s


nguyn t l p = 3.

Bi 17. Chng minh rng: Nu a2 b2 l mt s nguyn t th a2 b2 =


a + b.

Bi 18. Chng minh rng mi s nguyn t ln hn 3 u c dng 6n+1


hoc 6n 1.

Bi 19. Chng minh rng tng bnh phng ca 3 s nguyn t ln hn


3 khng th l mt s nguyn t.

Bi 20. Cho s t nhin n 2. Gi p1 , p2 , ..., pn l nhng s nguyn t


sao cho pn n + 1. t A = p1 .p2 ...pn . Chng minh rng trong
dy s cc s t nhin lin tip: A + 2, A + 3, ..., A + (n + 1),
khng cha mt s nguyn t no.

Bi 21. Chng minh rng: Nu p l s nguyn t th 2.3.4...(p 3)(p


.
2) 1..p.

Chuyn S hc Din n Ton hc


24 2.4. Ph lc: Bn nn bit

Bi 22. Chng minh rng: Nu p l s nguyn t th 2.3.4...(p 2)(p


.
1) + 1..p.

2.4 Ph lc: Bn nn bit


Mi s nguyn t c 93 ch s lp thnh cp s cng
Sau y l mt s nguyn t gm 93 ch s:

100996972469714247637786655587969840329509324689190041
803603417758904341703348882159067229719

K lc ny do 70 nh ton hc lp c nm 1998 tht kh m nh


bi c. H mt nhiu thng tnh ton mi tm c mi s nguyn
t to thnh mt cp s cng.

T mc tr chi trong 1 tp ch khoa hc, hai nh nghin cu trng


i hc Lyonl (Php) o su tng: Tm 6 s nguyn t sao cho
hiu 2 s lin tip lun lun nh nhau. iu l d i vi cc chuyn
gia nhng h mun i xa hn. Cng khng c vn g kh khn i
vi mt dy 7 s. H cn s h tr mt cht t c 8 s, mt s
h tr hn na t ti 9 s. Cui cng thng 3 nm 1998 c 70 nh
ton hc t khp trn th gii cng vi 200 my in ton hot ng
lin tc tm ra 10 s, mi s c 93 ch s, m hiu s ca 2 s lin
tip lun lun l 210. T s nguyn t trn ch cn thm vo 210 l
c s nguyn t th 2....

K lc c l dng : Theo c tnh ca cc nh khoa hc mun tm


c 1 dy 11 s nguyn t th phi mt hn 10 t nm.

Sinh ba rt t, phi chng sinh i li rt nhiu


Ta bit rng cc s nguyn t c th xa nhau tu iu ny th hin
bi tp:

Din n Ton hc Chuyn S hc


2.4. Ph lc: Bn nn bit 25

Bi ton 2.1. Cho trc s nguyn dng n tu . Chng minh rng


tn ti n s t nhin lin tip m mi s trong chng u l hp s.4

Vy nhng, cc s nguyn t cng c th rt gn nhau. Cp s (2, 3)


l cp s t nhin lin tip duy nht m c hai bn u l s nguyn
t. Cp s (p, q)c gi l cp s sinh i, nu c 2 u l s nguyn
t v q = p + 2. B 3 s (p, q, r) gi l b s nguyn t sinh ba nu
c 3 s p,q,r u l cc s nguyn t v q = p + 2; r = q + 2.

Bi ton 2.2. Tm tt c cc b s nguyn t sinh ba? 4

y l mt bi ton d, dng phng php chng minh duy nht ta


tm ra b (3, 5, 7) l b ba s nguyn t sinh ba duy nht, cc b 3 s
l ln hn 3 lun c 1 s l hp s v n chia ht cho 3.
T bi ton 2.2 th bi ton sau tr thnh mt gi thuyt ln ang ch
cu tr li.

D on 2.1 Tn ti v hn cp s sinh i. 

S hon ho (hon ton) ca nhng ngi Hy Lp c i


Ngi Hy Lp c i c quan nim thn b v cc s. H rt th v
pht hin ra cc s hon ho, ngha l cc s t nhin m tng cc c
s t nhin thc s ca n (cc c s nh hn s ) bng chnh n.

Chng hn:

6=1+2+3 28 = 1 + 2 + 4 + 7 + 14

Ngi Hy Lp c i bit tm tt c cc s hon ho chn ngha l


h lm c bi ton sau y:

Bi ton 2.3. Mt s t nhin chn n 6= 0 l s hon ho nu v ch


nu: n = 2m+1 (2m 1). Trong m l s t nhin khc 0 sao cho
2m 1 l s nguyn t. 4

T ta c gi thuyt

Chuyn S hc Din n Ton hc


26 2.4. Ph lc: Bn nn bit

D on 2.2 Khng tn ti s hon ho l. 

bi ton 2.3 trn, s nguyn t dng 2m 1 gi l s nguyn t


Merseme. Cc s nguyn t Merseme c vai tr rt quan trng. Cho
n nay ngi ta vn cha bit c hu hn hay v hn s nguyn t
Merseme.

D on 2.3 Tn ti v hn s nguyn t Merseme. 

Nm 1985 s nguyn t ln nht m ngi ta bit l s 2132049 1 gm


39751 ch s ghi trong h thp phn. Gn y 2 sinh vin M tm
ra mt s nguyn t ln hn na l s 2216091 1 gm 65050 ch s.

Ta bit rng vi hc sinh lp 6 th xem s A c t hn 20 ch s


c l s nguyn t khng bng cch th xem A c chia ht cho s no
nh hn A hay khng, th tm ht cc s nguyn t vi chic my
siu in ton cn hng th k !!!

David SlowinSky son mt phn mm, lm vic trn my siu in


ton Gray-2 , sau 19 gi ng tm ra s nguyn t 2756839 1. S ny
vit trong h thp phn s c 227832 ch s- vit ht s ny cn 110
trang vn bn bnh thng. Hoc nu vit hng ngang nhng s trn
phng ch .VnTime Size 14 th ta cn khong 570 m.

Li Kt
Thng qua ti ny, chng ta c th khng nh rng: Ton hc c
mt trong mi cng vic, mi lnh vc ca cuc sng quanh ta, n
khng th tch ri v lng qun c, nn chng ta phi hiu bit v
nm bt c n mt cch t gic v hiu qu.

Mc ch ca ti ny l trang b nhng kin thc c bn c o


su c nng cao v rn luyn t duy ton hc cho hc sinh, to ra nn
tng tin cy cc em c vn kin thc nht nh lm hnh trang cho

Din n Ton hc Chuyn S hc


2.4. Ph lc: Bn nn bit 27

nhng nm hc tip theo.

Vi iu kin c nhiu hn ch v thi gian, v nng lc trnh nn


trong khun kh ti ny phn chia dng ton, loi ton ch c tnh
tng i. ng thi cng mi ch a ra li gii ch cha c phng
php, thut lm r rng. Tuy c c gng nhiu nhng chnsg ti t
thy trong ti ny cn nhiu hn ch. Chng ti rt mong nhn
c nhng kin ng gp ca cc thy c gio cng bn c ton
hc tht s c ngha cao p nh cu ngn ng Php vit:

Ton hc l Vua ca cc khoa hc


S hc l N hong

Chuyn S hc Din n Ton hc


Chng

3
Bi ton chia ht
3.1 L thuyt c bn 29
3.2 Phng php gii cc bi ton chia
ht 31

Phm Quang Ton (Phm Quang Ton)

Chia ht l mt ti quan trng trong chng trnh S hc ca bc


THCS. i km theo l cc bi ton kh v hay. Bi vit ny xin
gii thiu vi bn c nhng phng php gii cc bi ton chia ht:
phng php xt s d, phng php quy np, phng php ng d,
v.v...

3.1 L thuyt c bn
3.1.1 nh ngha v chia ht
nh ngha 3.1 Cho hai s nguyn a v b trong b 6= 0, ta lun tm
c hai s nguyn q v r duy nht sao cho

a = bq + r

vi 0 r < b.
Trong , ta ni a l s b chia, b l s chia, q l thng, r l s d.4

Nh vy, khi a chia cho b th c th a ra cc s d r {0; 1; 2; ; |b|}.

c bit, vi r = 0 th a = bq, khi ta ni a chia ht cho b (hoc a l


bi ca b, hoc b l c ca a). Ta k hiu b | a. Cn khi a khng chia

29
30 3.1. L thuyt c bn

ht cho b, ta k hiu b - a.

Sau y l mt s tnh cht thng dng, chng minh c suy ra trc


tip t nh ngha.

3.1.2 Tnh cht


Sau y xin gii thiu mt s tnh cht v chia ht, vic chng minh
kh l d dng nn s dnh cho bn c. Ta c vi a, b, c, d l cc s
nguyn th:

Tnh cht 3.1 Nu a 6= 0 th a | a, 0 | a. 

Tnh cht 3.2 Nu b | a th b | ac. 

Tnh cht 3.3 Nu b | a v c | b th c | a. 

Tnh cht 3.4 Nu c | a v c | b th c | (ax by) vi x, y nguyn.

Tnh cht 3.5 Nu b | a v a | b th a = b hoc a = b.

Tnh cht 3.6 Nu c | a v d | b th cd | ab.

Tnh cht 3.7 Nu b | a, c | a th BCNN(b; c) | a.

Tnh cht 3.8 Nu c | ab v UCLN(b, c) = 1 th c | a.

Tnh cht 3.9 Nu p | ab, p l s nguyn t th p | a hoc p | b. 

T tnh cht trn ta suy ra h qu

H qu 3.1 Nu p | an vi p l s nguyn t, n nguyn dng th


pn | an . 

Din n Ton hc Chuyn S hc


3.2. Phng php gii cc bi ton chia ht 31

3.1.3 Mt s du hiu chia ht


Ta t N = an an1 . . . a1 a0
Du hiu chia ht cho 2; 5; 4; 25; 8; 125

2|N 2 | a0 a0 {0; 2; 4; 6; 8}
5|N 5 | a0 a0 {0; 5}
4; 25 | N 4; 25 | a1 a0
8; 125 | N 8; 125 | a2 a1 a0
Du hiu chia ht cho 3 v 9

3; 9 | N 3; 9 | (a0 + a1 + + an1 + an )

Mt s du hiu chia ht khc

11 | N 11 | [(a0 + a2 + ) (a1 + a3 + )]
101 | N 101 | [(a1 a0 + a5 a4 + ) (a3 a2 + a7 a6 + )]
7; 13 | N 7; 37 | [(a2 a1 a0 + a8 a7 a6 + ) (a5 a4 a3 + a11 a10 a9 + )]
37 | N 37 | (a2 a1 a0 + a5 a4 a3 + + an an1 an2 )
19 | N 19 | an + 2an1 + 22 an2 + + 2n a0

3.2 Phng php gii cc bi ton chia ht


3.2.1 p dng nh l Fermat nh v cc tnh cht ca chia
ht
nh l Fermat nh
nh l 3.1 (nh l Fermat nh) Vi mi s nguyn a v s
nguyn t p th ap p (mod p). 

Chng minh. 1. Nu p | a th p | (a5 a).

2. Nu p - a th 2a, 3a, 4a, , (p 1)a cng khng chia ht cho p.


Gi r1 , r2 , , rp1 ln lt l s d khi chia a, 2a, 3a, , (p1)a
cho p. th chng s thuc tp {1; 2; 3; ; p 1} v i mt khc
nhau (v chng hn nu r1 = r3 th p | (3a a) hay p | 2a,

Chuyn S hc Din n Ton hc


32 3.2. Phng php gii cc bi ton chia ht

ch c th l p = 2, m p = 2 th bi ton khng ng). Do


r1 r2 rp1 = 1 2 3 (p 1). Ta c

a r1 (mod p)
2a r2 (mod p)

(p 1)a rp1 (mod p)

Nhn v theo v ta suy ra

123 (p1)ap1 r1 r2 rp1 (mod p) ap1 1 (mod p)

V U CLN (a, p) = 1 nn ap a (mod p).


Nh vy vi mi s nguyn a v s nguyn t p th ap a (mod p). 

Nhn xt. Ta c th chng minh nh l bng quy np. Ngoi ra, nh


l cn c pht biu di dng sau:

nh l 3.2 Vi mi s nguyn a, p l s nguyn t, U CLN (a, p) =


1 th ap1 1 (mod p). 

Phng php s dng tnh cht chia ht v p dng nh l


Fermat nh
C s: S dng cc tnh cht chia ht v nh l Fermat nh gii
ton.

V d 3.1. Cho a v b l hai s t nhin. Chng minh rng 5a2 +15ab


b2 chia ht cho 49 khi v ch khi 3a + b chia ht cho 7. 4
2 2 2 2
Li gii. ) Gi s 49 | 5a + 15ab b 7 | 5a + 15ab b 7 |
(14a2 + 21ab) (5a2 + 15ab b2 ) 7 | (9a2 + 6ab + b2 ) 7 |
(3a + b)2 7 | 3a + b.

) Gi s 7 | 3a + b. t 3a + b = 7c (c Z. Khi b = 7c 3a. Nh
vy

5a2 + 15ab b2 = 5a2 + 15a(7c 3a) (7c 3a)2


= 49(c2 + 3ac a2 )

Din n Ton hc Chuyn S hc


3.2. Phng php gii cc bi ton chia ht 33

chia ht cho 49.


Vy 5a2 + 15ab b2 chia ht cho 49 khi v ch khi 3a + b chia ht cho
7. 

V d 3.2. Cho 11 | (16a + 17b)(17a + 16b) vi a, b l hai s nguyn.


Chng minh rng 121 | (16a + 17b)(17a + 16b). 4
Li gii. Ta c theo u bi, v 11 nguyn t nn t nht mt trong
hai s 16a + 17b v 17a + 16b chia ht cho 11. Ta li c (16a + 17b) +
(17a + 16b) = 33(a + b) chia ht cho 11. Do nu mt trong hai s
16a + 17b v 17a + 16b chia ht cho 11 th s cn li cng chia ht cho
11. Cho nn 121 | (16a + 17b)(17a + 16b). 

V d 3.3. Chng minh rng A = 130 + 230 + + 1130 khng chia ht


cho 11. 4
Li gii. Vi mi a = 1, 2, , 10 th (a, 10) = 1. Do theo nh l
Fermat b th a10 1 (mod 11) a30 1 (mod 11) vi mi a =
1, 2, , 10 v 1130 0 (mod 11). Nh vy

A 1 + 1 + + 1 +0 (mod 11)
| {z }
10 s 1
10 (mod 11) 11 - A

V d 3.4. Cho p v q l hai s nguyn t phn bit. Chng minh rng


pq1 + q p1 1 chia ht cho pq. 4
Li gii. V q nguyn t nn theo nh l Fermat nh th

pq1 1 (mod q)

Do
pq1 + q p1 1 (mod q)
V q v p c vai tr bnh ng nn ta cng d dng suy ra

q p1 + pq1 1 (mod p).

Cui cng v U CLN (q, p) = 1 nn pq1 + q p1 1 (mod pq) hay


pq1 + q p1 1 chia ht cho pq. 

Chuyn S hc Din n Ton hc


34 3.2. Phng php gii cc bi ton chia ht

Bi tp ngh
Bi 1. Chng minh rng 11a+2b chia ht cho 19 khi v ch khi 18a+5b
chia ht cho 19 vi a, b l cc s nguyn.

Bi 2. Chng minh rng 2a + 7 chia ht cho 7 khi v ch khi 3a2 +


10ab 8b2 .

Bi 3. Cho p l s nguyn t ln hn 5. Chng minh rng nu n l s


t nhin c p 1 ch s v cc ch s u bng 1 th n chia
ht cho p.

Bi 4. Gi s n N, n 2. Xt cc s t nhin an = 11 1 c vit
bi n ch s 1. Chng minh rng nu an l mt s nguyn t
th n l c ca an 1.

Bi 5. Gi s a v b l cc s nguyn dng sao cho 2a 1, 2b 1 v


a + b u l s nguyn t. Chng minh rng ab + ba v aa + bb
u khng chia ht cho a + b.

Bi 6. Chng minh rng vi mi s nguyn t p th tn ti s nguyn


n sao cho 2n + 3n + 6n 1 chia ht cho p.

3.2.2 Xt s d
C s: chng minh A(n) chia ht cho p, ta xt cc s n dng
n = kp + r vi r {0; 1; 2; ; p 1}.
Chng hn, vi p = 5 th s nguyn n c th vit li thnh 5k; 5k +
1; 5k + 2; 5k + 3; 5k + 4. Ta th mi dng ny vo cc v tr ca n ri
l lun ra p s. Sau y l mt s v d

V d 3.5. Tm k N tn ti n N sao cho

4 | n2 k

vi k {0; 1; 2; 3}. 4
2
Li gii. Gi s tn ti k N tn ti n N tha mn 4 | n k.
Ta xt cc Trng hp: (m N )

Din n Ton hc Chuyn S hc


3.2. Phng php gii cc bi ton chia ht 35

1. Nu n = 4m th n2 k = 16m2 k chia ht cho 4 khi v ch khi


4 | k nn k = 0.

2. Nu n = 4m 1 th n2 k = 16m2 8m + 1 k chia ht cho 4


khi v ch khi 4 | 1 k nn k = 1.

3. Nu n = 4m 2 th n2 k = 16m2 16m + 4 k chia ht cho 4


khi v ch khi 4 | k nn k = 0.

Vy k = 0 hoc k = 1. 

V d 3.6. Chng minh rng vi mi n N th 6 | n(2n + 7)(7n + 1).4


Li gii. Ta thy mt trong hai s n v 7n + 1 l s chn n N. Do
2 | n(2n + 7)(7n + 1). Ta s chng minh 3 | n(2n + 7)(7n + 1). Tht
vy, xt

1. Vi n = 3k th 3 | n(2n + 7)(7n + 1).

2. Vi n = 3k + 1 th 2n + 7 = 6k + 9 chia ht cho 3 nn 3 |
n(2n + 7)(7n + 1).

3. Vi n = 3k + 2 th 7n + 1 = 21k + 15 chia ht cho 3 nn 3 |


n(2n + 7)(7n + 1).

Do 3 | n(2n + 7)(7n + 1) m (2, 3) = 1 nn 6 | n(2n + 7)(7n + 1) n


N. 

V d 3.7. (HSG 9, Tp H Ch Minh, vng 2, 1995) Cho x, y, z l cc


s nguyn tha mn

(x y)(y z)(z x) = x + y + z (3.1)

Chng minh rng 27 | (x + y + z). 4


Li gii. Xt hai trng hp sau

Chuyn S hc Din n Ton hc


36 3.2. Phng php gii cc bi ton chia ht

1. Nu ba s x, y, z chia ht cho 3 c cc s d khc nhau th cc


hiu xy, y z, z x cng khng chia ht cho 3. M 3 | (x+y +z)
nn t (3.1) suy ra v l .

2. Nu ba s x, y, z ch c hai s chia cho 3 c cng s d th trong ba


hiu xy, y z, z x c mt hiu chia ht cho 3. M 3 - (x+y +z)
nn t (3.1) suy ra v l.

Vy x, y, z chia cho 3 c cng s d, khi x y, y z, z x u chia


ht cho 3. T (3.1) ta suy ra 27 | (x + y + z), ta c pcm. 

Bi tp ngh
Bi 1. i) Tm s t nhin n 7 | (2n 1).
ii) Chng minh rng 7 - (2n + 1) n N.

Bi 2. Chng minh rng vi mi s nguyn a th a(a6 1) chia ht


cho 7.

Bi 3. Tm n 13 | 32n + 3n + 1.

Bi 4. Chng minh rng vi mi a, b N th ab(a2 b2 )(4a2 b2 ) lun


lun chia ht cho 5.

Bi 5. Chng minh rng 24 | (p 1)(p + 1) vi p l s nguyn t ln


hn 3.

Bi 6. Chng minh rng khng tn ti s nguyn a a2 + 1 chia ht


cho 12.

Bi 7. Chng minh rng vi mi s nguyn x, y, z nu 6 | x + y + z th


6 | x3 + y 3 + z 3 .

Bi 8. Cho ab = 20112012 , vi a, b N. Hi tng a + b c chia ht cho


2012 hay khng ?

Bi 9. S 3n + 2003 trong n l s nguyn dng c chia ht cho 184


khng ?

Din n Ton hc Chuyn S hc


3.2. Phng php gii cc bi ton chia ht 37

Bi 10. Cho cc s nguyn dng x, y, z tha mn x2 + y 2 = z 2 . Chng


minh rng xyz chia ht cho 60.

Bi 11. Cho cc s nguyn dng x, y, z tha mn x2 +y 2 = 2z 2 . Chng


minh rng x2 y 2 chia ht cho 84.

Bi 12. Cho n > 3, (n N). Chng minh rng nu 2n = 10a+b, (0 <


b < 9) th 6 | ab.

3.2.3 Phn tch


Phn tch thnh tch
C s: chng minh A(n) chia ht cho p, ta phn tch A(n) = D(n)p,
cn nu trong ta khng th a ra cch phn tch nh vy, ta c th
vit p = kq.

Nu (k, q) = 1 th ta chng minh A(n) cng chia ht cho k v q.

Nu (k, q) 6= 1 th ta vit A(n) = B(n)C(n) v chng minh B(n)


chia ht cho k, C(n) chia ht cho q.

V d 3.8. Cho n l mt s nguyn dng. Chng minh rng

2n | (n + 1) (n + 2) (2n) .

Li gii. Ta c

(2n)! (1.3.5...(2n 1)) (2.4.6...2n)


(n + 1) (n + 2) (2n) = =
n! n!
n n!
= 1.3.5...(2n 1).2 .
n!
n
= 1.3.5...(2n 1).2 .

Do 2n | (n + 1) (n + 2) (2n) . 

Chuyn S hc Din n Ton hc


38 3.2. Phng php gii cc bi ton chia ht

V d 3.9. Chng minh rng vi mi s nguyn n th 6 | n3 n. 4


Li gii. Phn tch

n3 n = n(n2 1) = n(n 1)(n + 1)

Biu thc l tch ba s nguyn lin tip nn tn ti t nht mt trong


ba s mt s chia ht cho 2 v mt s chia ht cho 3. M (2, 3) = 1 nn
6 | n3 n. 

V d 3.10. Chng minh rng n6 n4 n2 + 1 chia ht cho 128 vi n


l. 4
Li gii. Ta c

n6 n4 n2 + 1 = (n2 1)2 (n + 1) = (n 1)2 (n + 1)2

V n l nn t n = 2k, k N, suy ra

(n2 1)2 = (2k + 1)2 1 = (4k 2 + 4k)2 = [4k(k + 1)]2


 

Vy 64 | (n2 1)2 . V n l nn 2 | n + 1, suy ra pcm. 

V d 3.11. Cho ba s nguyn dng khc nhau x, y, z. Chng minh


rng (x y)5 + (y z)5 + (x z)5 chia ht cho 5(x y)(y z)(x z).4
Li gii. Ta c

(x y)5 + (y z)5 + (x z)5


= (x z + z y)5 + (y z)5 + (z x)5
= (x z)5 + 5(x z)4 (z y) + 10(x z)3 (z y)2
+10(x z)4 (z y) + 10(x z)3 (z y)2
+10(x z)2 (z y)3 + 5(x z)(z y)4
 z)(z y)
= 5(x
(x z)3 + 2(x z)2 (z y) + 2(x z)(z y)2 + (z y)3 .


Din n Ton hc Chuyn S hc


3.2. Phng php gii cc bi ton chia ht 39

Nhng ta cng c:

(x z)3 + 2(x z)2 (z y) + 2(x z)(z y)2 + (z y)3


= (x y + y z)3 + 2(x y + y z)2 (z y)
+2(x y + y z)(z y)2 + (z y)3
= (x y)3 + 2(x y)2 (y z) + 3(x y)(y z)2
+(y z)3 + 2(x y)2 (z y)
+4(x y)(y z)(z y) + 2(y z)2 (z y)
+2(x y)(z y)2 + 2(y z)(z y)2 + (z y)3
= (x y)3 + 3(x y)2 (y z) + 3(x y)(y z)2
+2(x y)2 (z y) + 4(x y)(y z)(z y) + 2(x y)(z y)2 ,

Biu thc cui cng c nhn t chung (x y). Ta suy ra iu phi


chng minh. 

Bi tp ngh
Bi 1. Chng minh rng nu a, k l cc s nguyn, a l th 2k+1 |
k
(a2 1).

Bi 2. Chng minh rng n5 n chia ht cho 30 vi mi n Z.

Bi 3. Chng minh rng 3n4 14n3 + 21n2 10n chia ht cho 24 vi


mi n Z.

Bi 4. Chng minh rng n5 5n3 + 4n chia ht cho 120 vi mi n Z.

Bi 5. Chng minh rng n3 3n2 n + 3 chia ht cho 48 vi mi n


l, n Z.

Bi 6. Chng minh rng n8 n6 n4 + n2 chia ht cho 1152 vi mi


s nguyn n l.

Bi 7. Chng minh rng n4 4n3 4n2 + 16n chia ht cho 348 vi mi


n l s nguyn chn.

Bi 8. Chng minh rng n4 14n3 + 71n2 154n + 120 chia ht cho


24 vi mi s t nhin n.

Chuyn S hc Din n Ton hc


40 3.2. Phng php gii cc bi ton chia ht

Bi 9. Cho x, y, z l cc s nguyn khc 0. Chng minh rng nu


x2 yz = a, y 2 zx = b, z 2 xy = c th tng (ax + by + cz)
chia ht cho tng (a + b + c).

Bi 10. Cho m, n l hai s chnh phng l lin tip. Chng minh rng
mn m n + 1 chia ht cho 192.

Bi 11. (HSG 9 TQ 1970) Chng minh rng n12 n8 n4 + 1 chia ht


cho 512 vi mi s t nhin n l.

Bi 12. (HSG 9 TQ 1975) Chng minh rng n4 + 6n3 + 11n2 + 6n chia


ht cho 24 vi mi s nguyn dng n.

Tch tng
C s: chng minh A(n) chia ht cho p, ta bin i A(n) thnh
tng nhiu hng t ri chng minh mi hng t u chia ht cho p.

Ta c th s dng mt s hng ng thc p dng vo chia ht, v d


nh:

Cho a, b l cc s thc v n l s nguyn dng. Khi ta


c

an bn = (a b)(an1 + an2 b + + abn2 + bn1 )

Ta s c h qu l:

H qu 3.2 Nu a b 6= 0 th an bn chia ht cho a b. 

H qu 3.3 Nu a + b 6= 0 v n l th an + bn chia ht cho a + b. 

H qu 3.4 Nu a + b 6= 0 v n chn th an bn chia ht cho a + b

V d 3.12. Chng minh rng ax2 + bx + c Z, x Z khi v ch khi


2a, a + b, c Z 4

Din n Ton hc Chuyn S hc


3.2. Phng php gii cc bi ton chia ht 41

Li gii. Phn tch

ax2 + bx + c = ax2 ax + (a + b)x + c


x(x 1)
= 2a. + (a + b)x + c Z, x Z.
2

V d 3.13. Chng minh rng 6 | (a3 + 5a) a N. 4


Li gii. Phn tch a3 +5a = (a3 a)+6a. Hin nhin ng v 6 | n3 n
(chng minh v d Equation 4.27). 

Nhn xt. T v d Equation 4.27 ta cng c th a ra cc bi ton


sau, chng minh cng bng cch vn dng phng php tch tng:

Bi ton 3.1. Cho m, n Z. Chng minh rng 6 | m2 n2 (m n). 4

Bi ton 3.2. Cho a, b, c Z. Chng minh rng 6 | (a3 + b3 + c3 ) khi


v ch khi 6 | (a + b + c) 4

a a2 a3
Bi ton 3.3. Cho a Z. Chng minh rng + + Z 4
3 2 6

Bi ton 3.4. Vit s 20112012 thnh tng cc s nguyn dng. em


tng lp phng tt c cc s hng chia cho 3 th c d l bao
nhiu ? 4

V d 3.14. Cho m, n l cc s nguyn tha mn:

m 1 1 1 1 1
= 1 + + +
n 2 3 4 1334 1335

Chng minh rng 2003 | m. 4

Chuyn S hc Din n Ton hc


42 3.2. Phng php gii cc bi ton chia ht

Li gii. rng 2003 l s nguyn t. Ta c


m 1 1 1 1 1
= 1 + + +
n  2 3 4 1334
 1335
 
1 1 1 1 1 1 1
= 1 + + + + 2 + + + +
 2 3 1335   2 4 6 1334

1 1 1 1 1 1
= 1 + + + + 1 + + + +
2 3 1335 2 3 667
1 1 1
= + + +
668 669  1335
   
1 1 1 1 1 1
= + + + + + +
668 1335 669 1334 1001 1002
1 1 1
= 2003 + + +
668.1335 669.1334 1001.1002
p
= 2003.
q

y p l s nguyn cn q = 668 669 1335. V 2003 nguyn t nn


(q, 2003) = 1.
Do t () suy ra 2003pn = mq.
V p, n nguyn nn suy ra 2003|mq m (q, 2003) = 1 nn 2003|m. 

V d 3.15. Chng minh rng vi mi s t nhin n th A = 2005n +


60n 1897n 168n chia ht cho 2004. 4
Li gii. Ta c 2004 = 12 167. V (12, 167) = 1 nn chng minh
A chia ht cho 2004 ta chng minh A chia ht cho 12 v 167.
p dng tnh cht an bn chia ht cho a b vi mi n t nhin v
a b 6= 0 suy ra 2005n 1897n chia ht cho 2005 1897 = 108 = 12 9,
hay 2005n 1897n chia ht cho 12. Tng t th 168n 60n chia ht
cho 12. Vy A chia ht cho 12.
Tip tc phn tch

A = (2005n 168n ) (1897n 60n ).

Lp lun tng t nh trn th 2005n 168n v 1897n 60n chia ht


cho 167, tc A chia ht cho 167. Vy ta c iu phi chng minh. 

Din n Ton hc Chuyn S hc


3.2. Phng php gii cc bi ton chia ht 43

V d 3.16. ( thi tuyn sinh HKHTN-HQG H Ni, vng 1, nm


2007-2008) Cho a, b l hai s nguyn dng v a + 1, b + 2007 u chia
ht cho 6. Chng minh rng 4a + a + b chia ht cho 6. 4
Li gii. Phn tch

4a + a + b = (4a + 2) + (a + 1) + (b + 2007) 2010


4a + 2 = 4a 1 + 3 = (4 1)(4a1 + 1) + 3

Nh vy 3 | 4a + 2. Do 4a + a + b l tng ca cc s nguyn dng


chia ht cho 6 nn 4a + a + b chia ht cho 6. 

Bi tp ngh
Bi 1. a ra cc m rng t bi tp ngh ca phng php phn
tch thnh tch thnh cc bi ton vn dng phng php tch
tng (ging nh cch m rng ca v d 1.9).

Bi 2. (Hungary MO 1947) Chng minh rng 46n + 296.13n chia ht


cho 1947 vi mi s t nhin n l.

Bi 3. Chng minh rng 20n + 16n 3n 1 chia ht cho 323 vi mi


s t nhin n chn.

Bi 4. Chng minh rng 2903n 803n 464n + 261n chia ht cho 1897
vi mi s t nhin n.

Bi 5. Chng minh rng vi mi s nguyn n > 1 ta c nn + 5n2


11n + 5 chia ht cho (n 1)2 .

Bi 6. (HSG 9 Tp H Ni, vng 2, 1998) Chng minh rng 1997 | m


vi m, n N tha mn
m 1 1 1 1 1 1
= 1 + + + + .
n 2 3 4 1329 1330 1331

Bi 7. Chng minh rng 32n+1 + 2n+2 chia ht cho 7 vi mi n N.

Chuyn S hc Din n Ton hc


44 3.2. Phng php gii cc bi ton chia ht

Bi 8. Chng minh rng 20032005 + 20172015 chia ht cho 12.


Bi 9. Cho p l s t nhin l v cc s nguyn a, b, c, d, e tha mn
a + b + c + d + e v a2 + b2 + c2 + d2 + e2 u chia ht cho p.
Chng minh rng s a5 + b5 + c5 + d5 + e5 5abcde cng chia
ht cho p.
Bi 10. (Canada Training for IMO 1987)
K hiu:
1 3 5 (2n 1) = (2n 1)!!
2 4 6 (2n) = (2n)!!.
Chng minh rng (1985)!! + (1986)!! chia ht cho 1987.
Bi 11. Chng minh rng s 22225555 + 55552222 chia ht cho 7.
Bi 12. Cho k l s nguyn dng sao cho s p = 3k + 1 l s nguyn
t v
1 1 1 m
+ + + =
12 34 (2k 1)2k n
vi hai s nguyn dng nguyn t cng nhau m v n.Chng
minh m chia ht cho p.
(Tp ch Mathematics Reflections, ng bi T.Andreescu)

3.2.4 Xt ng d
nh ngha v mt s tnh cht
nh ngha 3.2 Cho a, b l cc s nguyn v n l s nguyn dng. Ta
ni, a ng d vi b theo modun n v k hiu a b (mod n) nu a v
b c cng s d khi chia cho n. 4
Nh vy a n (mod n) n | (a b). V d: 2012 2 (mod 5).

Tnh cht (bn c t chng minh)


Cho a, b, c, d, n l cc s nguyn.
a a (mod n),
Tnh cht 3.10 a b (mod n) b a (mod n), 
a b (mod n), b c (mod n) a c (mod n).

Din n Ton hc Chuyn S hc


3.2. Phng php gii cc bi ton chia ht 45

( (
a b (mod n) a c b d (mod n)
Tnh cht 3.11 
c d (mod n) ac bd (mod n)

Tnh cht 3.12 a b (mod n) ak bk (mod n), k 1. 

Tnh cht 3.13 a b (mod n) ac bc (mod mc), c > 0 

Tnh cht 3.14 (a + b)n bn (mod a), (a > 0). 

Tnh cht 3.15 Nu d l c chung dng ca a, b v m th a b


a b m
(mod m) th (mod ).
d d d

Tnh cht 3.16 a b (mod m), c l c chung ca a v b, (c, m) = 1


a b
th (mod m).
c c

Phng php ng d thc gii cc bi ton chia ht


C s: S dng cc tnh cht v nh ngha trn gii cc bi ton
chia ht.

V d 3.17. Chng minh rng vi mi s t nhin n th 7 | 8n + 6. 4


Li gii. Ta c 8n 1 (mod 7) = 8n + 6 7 0 (mod 7). 

V d 3.18. Chng minh rng 19 | 7 52n + 12 6n . vi mi s nguyn


dng n. 4
Li gii. Ta c 52 = 25 6 (mod 19) = (52 )n 6n (mod 19) =
7 52n 7 6n (mod 19) = 7 52n + 12 6n 19 6n 0 (mod 19).

V d 3.19. Vit lin tip cc s 111, 112, , 888 c s A =


111112 888. Chng minh rng 1998 | A. 4

Chuyn S hc Din n Ton hc


46 3.2. Phng php gii cc bi ton chia ht

Li gii. Ta thy A chn nn 2 | A. Mt khc

A = 111 1000777 + 112 1000776 + + 888.

Do 1000k 1 (mod 999), k N nn

A 111 + 112 + + 888 0 (mod 999).

Suy ra 999 | A, v (999, 2) = 1 nn 1998 | A. 

V d 3.20. Chng minh rng 7 | 55552222 + 22225555 . 4


Li gii. Ta c

2222 4 (mod 7) = 22225555 (4)5555 (mod 7)


5555 4 (mod 7) = 55552222 4 (mod 7)

= 55552222 + 22225555 45555 + 42222 (mod 7)


Li c
45555 + 42222 = 42222 43333 1


= 42222 641111 1


V 64 1 (mod 7) = 641111 1 0 (mod 7).


Do 7 | 55552222 + 22225555 

Bi tp ngh
Bi 1. Mt s bi tp phng php phn tch c th gii bng phng
php ng d thc.
777 333
Bi 2. Chng minh rng 333555 + 777555 chia ht cho 10.
1967
Bi 3. Chng minh rng s 1110 1 chia ht cho 101968 .

Bi 4. Cho 9 | a3 + b3 + c3 , a, b, c Z. Chng minh rng 3 | a b c.

Bi 5. Chng minh rng 222333 + 333222 chia ht cho 13.

Din n Ton hc Chuyn S hc


3.2. Phng php gii cc bi ton chia ht 47

Bi 6. Chng minh rng 9n + 1 khng chia ht cho 100, n N.


Bi 7. Chng minh rng vi mi s nguyn khng m n th 25n+3 +
5n 3n+1 chia ht cho 17.
Bi 8. Tm n N sao cho 2n3 + 3n = 19851986.
Bi 9. Vit lin tip 2000 s 1999 ta c s X = 19991999 1999.
Tm s d trong php chia X cho 10001.
7
77 7
Bi 10. Chng minh rng 100 | 77 77 .
Bi 11. Cho b2 4ac v b2 + 4ac l hai s chnh phng vi a, b, c N.
Chng minh rng 30 | abc.

3.2.5 Quy np
C s : chng minh mnh ng vi mi s t nhin n p, ta
lm nh sau:
Kim tra mnh ng vi n = p.
Gi s mnh ng vi n = k. Ta i chng minh mnh cng
ng vi n = k + 1.

V d 3.21. Chng minh rng A = 4n + 15 1 chia ht cho 9 vi mi


n N . 4
Li gii. Vi n = 1 = A = 18 chia ht cho 9.
Gi s bi ton ng vi n = k. Khi 9 | 4k +15k 1, hay 4k +15k 1 =
9q vi q N . Suy ra 4k = 9q 15k + 1.
Ta i chng minh bi ton ng vi n = k+1, tc 9 | 4k+1 +15(k+1)1.
Tht vy:
4k+1 + 15(k + 1) 1 = 4 4k + 15k + 14
= 4 (9q 15k + 1) + 15k + 14
= 36q 45k + 18

chia ht cho 9. Ta c pcm. 

Chuyn S hc Din n Ton hc


48 3.2. Phng php gii cc bi ton chia ht

V d 3.22. (HSG 9 TQ 1978)Chng minh rng s c to bi 3n ch


s ging nhau th chia ht cho 3n vi 1 n, n N. 4
Li gii. Vi n = 1, bi ton hin nhin ng.
Gi s bi ton ng vi n = k, tc 3k | aa a}.
| {z
3n s a
Vi n = k + 1 ta c:

aa a} = aa
| {z a} aa
| {z a} aa
| {z a}
| {z
3k+1 3k 3k 3k
= aa a} 1 |00 {z
| {z 0} 1
0} |00 {z
3k 3k 1 3k 1

chia ht cho 3k+1 . Ta c pcm. 

V d 3.23. Chng minh rng vi mi n N , k l s t nhin l th


n
2n+2 | k 2 1
n
Li gii. Vi n = 1 th k 2 1 = k 2 1 = (k + 1)(k 1). Do k l,nn
t k = 2m + 1 vi m N , th khi (k + 1)(k 1) = 4k(k + 1) chia
ht cho 23 = 8.
p p
Gi s bi ton ng vi n = p, tc 2p+2 | k 2 1 hay k 2 = q 2p+2 + 1
vi q N .
Ta chng minh bi ton ng vi n = p + 1. Tht vy
p+1 p p 2
A = k2 1 = k 22 1 = k 2 1
p p
= k2 1 k2 + 1
 

= q 2p+2 2 + q 2p+2


= q 2p+3 1 + q 2p+1


chia ht cho 2p+3 . Ta c pcm. 

Din n Ton hc Chuyn S hc


3.2. Phng php gii cc bi ton chia ht 49

Bi tp ngh
Bi 1. Mt s bi ton cc phng php nu trn c th gii bng
phng php quy np.

Bi 2. Chng minh rng 255 | 16n 15n 1 vi n N.

Bi 3. Chng minh rng 64 | 32n+3 + 40n 27 vi n N.

Bi 4. Chng minh rng 16 | 32n+2 + 8n 9 vi n N.

Bi 5. Chng minh rng 676 | 33n+3 16n 27 vi n N, n 1.

Bi 6. Chng minh rng 700 | 292n 140n 1 vi n N.

Bi 7. Chng minh rng 270 | 2002n 138n 1 vi n N.


4n+1 4n+1
Bi 8. Chng minh rng 22 | 32 + 23 + 5 vi n N.
n
Bi 9. Chng minh rng s 23 + 1 chia ht cho 3n nhng khng chia
ht cho 3n+1 vi n N.
n
Bi 10. Chng minh rng s 20012 1 chia ht cho 2n+4 nhng khng
chia ht cho 2n+5 vi n N.

Bi 11. Chng minh rng vi mi s t nhin n 2, tn ti mt s t


nhin m sao cho 3n | (m3 + 17), nhng 3n+1 - (m3 + 17).

Bi 12. C tn ti hay khng mt s nguyn dng l bi ca 2007 v


c bn ch s tn cng l 2008.

Bi 13. Chng minh rng tn ti mt s c 2011 ch s gm ton ch


s 1 v 2 sao cho s chia ht cho 22011 .
n
Bi 14. Tm phn d khi chia 32 cho 2n+3 , trong n l s nguyn
dng.
.
..
Bi 15. Cho n N, n 2. t A = 77 (ly tha n ln). Chng minh
rng An + 17 chia ht cho 20.

Chuyn S hc Din n Ton hc


50 3.2. Phng php gii cc bi ton chia ht

3.2.6 S dng nguyn l Dirichlet


Ni dung: Nht 5 con th vo 3 chung th tn ti chung cha t nht
2 con.
nh l 3.3 Nht m = nk + 1 con th vo k chung (k < n) th tn
ti chung cha t nht n + 1 con th. 
Chng minh. Gi s khng c chung no cha t nht n + 1 con th,
khi mi chung cha nhiu nht n con th, nn k chung cha nhiu
nht kn con th, mu thun vi s th l nk + 1. 

nh l 3.4 (p dng vo s hc) Trong m = nk + 1 s c t


nht n + 1 s chia cho k c cng s d. 
Tuy nguyn l c pht biu kh n gin nhng li c nhng ng
dng ht sc bt ng, th v. Bi vit ny ch xin nu mt s ng dng
ca nguyn l trong vic gii cc bi ton v chia ht.

V d 3.24. Chng minh rng lun tn ti s c dng

20112011 201100 0

chia ht cho 2012. 4


Li gii. Ly 2013 s c dng

2011; 20112011, , 20112011


| {z 2011} .
2012 s 2011

Ly 2013 s ny chia cho 2012. Theo nguyn l Dirichlet th tn ti hai


s c cng s d khi chia cho 2012.
{z 2011} v 20112011
Gi s hai s l |20112011 | {z 2011} (m > n >
m s 2011 n s 2011
0).
= 2012 | |20112011
{z 2011} 20112011
| {z 2011}
m s 2011 n s 2011

Din n Ton hc Chuyn S hc


3.2. Phng php gii cc bi ton chia ht 51

= 2012 | |20112011
{z 2011} |00 {z
00}
mn s 2011 n s 2011

Vy tn ti s tha mn bi. 

V d 3.25. Chng minh rng trong 101 s nguyn bt k c th tm


c hai s c 2 ch s tn cng ging nhau. 4
Li gii. Ly 101 s nguyn cho chia cho 100 th theo nguyn l
Dirichlet tn ti hai s c cng s d khi chia cho 100. Suy ra trong
101 s nguyn cho tn ti hai s c ch s tn cng ging nhau. 

V d 3.26 (Tuyn sinh 10 chuyn HSPHN, 1993). Cho 5 s nguyn


phn bit ty a1 , a2 , a3 , a4 , a5 . Chng minh rng tch

P = (a1 a2 )(a1 a3 )(a1 a4 )(a1 a5 )(a2 a3 )


(a2 a4 )(a2 a5 )(a3 a4 )(a3 a5 )(a4 a5 )

chia ht cho 288. 4


Li gii. Phn tch 288 = 25 32 .

1. Chng minh 9 | P : Theo nguyn l Dirichlet th trong 4 s


a1 , a2 , a3 c hai s c hiu chia ht cho 3. Khng mt tnh tng
qut, gi s: 3 | a1 a2 . Xt 4 s a2 , a3 , a4 , a5 cng c hai s c
hiu chia ht cho 3. Nh vy P c t nht hai hiu khc nhau
chia ht cho 3, tc 9 | p.

2. Chng minh 32 | P : Theo nguyn l Dirichlet th tng 5 s cho


tn ti t nht 3 s c cng tnh chn l. Ch c th c hai kh
nng sau xy ra:

Nu c t nht 4 s c cng tnh chn l, th t bn s c th


lp thnh su hiu khc nhau chia ht cho 2. Do 32 | P .

Chuyn S hc Din n Ton hc


52 3.2. Phng php gii cc bi ton chia ht

Nu c 3 s c cng tnh chn l. Khng mt tnh tng qut,


gi s ba s l a1 , a2 , a3 . Khi a4 , a5 cng cng tnh
chn l nhng li khc tnh chn l ca a1 , a2 , a3 . Khi
cc hiu sau chia ht cho 2: a1 a2 , a1 a3 , a2 a3 , a4 a5 .
Mt khc, trong 5 s cho c t nht hai hiu chia ht cho
4, cho nn trong 4 hiu a1 a2 , a1 a3 , a2 a3 , a4 a5 c
t nht mt hiu chia ht cho 4. Vy 32 | P .

Ta c pcm. 

V d 3.27. Cho 2012 s t nhin bt k a1 , a2 , , a2012 . Chng minh


rng tn ti mt s chia ht cho 2012 hoc tng mt s s chia ht cho
2012. 4
Li gii. Xt 2012 s

S1 = a2
S2 = a1 + a2

S2012 = a1 + a2 + + a2012

Trng hp 1: Nu tn ti s Si (i = 1, 2, , 2012) chia ht cho


2012 th bi ton chng minh xong.

Trng hp 2: Nu 2012 - Si vi mi i = 1, 2, , 2012. em 2012


s ny chia cho 2012 nhn c 2012 s d. Cc s d nhn gi
tr thuc tp {1; 2; ; 2011}. V c 2012 s d m ch c 2011
gi tr nn theo nguyn l Dirichlet chc chn c hai s d bng
nhau. Ga s gi hai s l Sm v Sn c cng s d khi chia
cho 2012 (m, n N, 1 n < m 2012) th hiu

Sm Sn = an+1 + an+2 + + am

chia ht cho 2012. 

Din n Ton hc Chuyn S hc


3.2. Phng php gii cc bi ton chia ht 53

Nhn xt. Ta c th rt ra bi ton tng qut v bi ton m rng


sau:

Bi ton 3.5 (Bi ton tng qut). Cho n s a1 , a2 , , an . Chng


minh rng trong n s trn tn ti mt s chia ht cho n hoc tng mt
s s chia ht cho n. 4

Bi ton 3.6 (Bi ton m rng). (Tp ch Ton Tui Th s 115)


Cho n l mt s chuyn dng v n s nguyn dng a1 , a2 , , an c
tng bng 2n 1. Chng minh rng tn ti mt s s trong n s
cho c tng bng n. 4

Bi tp ngh
356
Bi 1. Chng minh rng c v s s chia ht cho 201311 m trong
biu din thp phn ca cc s khng c cc ch s 0, 1, 2, 3.

Bi 2. (HSG 9 H Ni, 2006) Chng minh rng tn ti s t nhin


n 6= 0 tha mn 313579 | (13579n 1).

Bi 3. Chng minh rng trong 52 s nguyn dng bt k lun lun


tm c hai s c tng hoc hiu chia ht cho 100.

Bi 4. Cho 10 s nguyn dng a1 , a2 , , a10 . Chng minh rng tn


ti cc s ci {0, 1, 1}, (i = 1, 10) khng ng thi bng
0 sao cho
A = c1 a1 + c2 a2 + + c10 a10

chia ht cho 1032.

Bi 5. Chng minh rng tn ti s t nhin k sao cho 2002k 1 chia


ht cho 200310 .

Bi 6. Bit rng ba s a, a + k, a + 2k u l cc s nguyn t ln hn


3. Chng minh rng khi k chia ht cho 6.

Chuyn S hc Din n Ton hc


54 3.2. Phng php gii cc bi ton chia ht

3.2.7 Phn chng


C s: chng minh p - A(n), ta lm nh sau:

Gi s ngc li p | A(n).

Chng minh iu ngc li sai.

V d 3.28. Chng minh rng vi mi s nguyn n th n2 +n+1 khng


chia ht cho 9. 4
2 2
Li gii. Gi s 9 | (n + n + 1). Khi n + n + 1 = (n + 2)(n 1) + 3
chia ht cho 3. Suy ra 3 | n + 2 v 3 | n 1. Nh vy (n + 2)(n 1)
chia ht cho 9, tc n2 + n + 1 chia 9 d 3, mu thun. Ta c pcm. 

Nhn xt. Bi ton ny vn c th gii theo phng php xt s d.

V d 3.29. Gi s p = k.2t + 1 l s nguyn t l, t l s nguyn


 tv k l
dng
t
 s t nhin l. Gi thit x v y l cc s t nhin m
2 2
p | x + y . Chng minh rng khi x v y ng thi chia ht cho
p. 4
Li gii. Gi s tri li p - x, suy ra p - y.
Do p l s nguyn t nn theo nh l Fermat nh ta c
 p1
x 1 (mod p)
y p1 1 (mod p)

Theo gi thit th p 1 = k.2t , do


 k.2t
x 1 (mod p)
k.2t
y 1 (mod p)

T ta c
t t
xk.2 + y k.2 2 (mod p). (i)
Theo gi thit th
t t
x2 + y 2 0 (mod p).

Din n Ton hc Chuyn S hc


3.2. Phng php gii cc bi ton chia ht 55

Do k l nn
 t  k  t k .  t 
t
xk.2 + y k.2
t
= x2 + y2 .. x2 + y 2t
 t t

xk.2 + y k.2 0 (mod p) (ii)

T (i) v (ii) suy ra iu mu thun. Vy gi thit phn chng sai. Do


x, y ng thi chia ht cho p. 

Bi tp ngh
Bi 1. Chng minh n2 + n + 2 khng chia ht cho 15 vi mi n Z.

Bi 2. Chng minh n2 + 3n + 5 khng chia ht cho 121 vi mi n N.

Bi 3. Chng minh 9n3 + 9n2 + 3n 16 khng chia ht cho 343 vi


mi n N.

Bi 4. Chng minh 4n3 6n2 + 3n + 37 khng chia ht cho 125 vi


mi n N.

Bi 5. Chng minh n3 + 3n 38 khng chia ht cho 49 vi mi n N.

Chuyn S hc Din n Ton hc


Chng

4
Phng trnh nghim
nguyn
4.1 Xt tnh chia ht 57
4.2 S dng bt ng thc 74
4.3 Nguyn tc cc hn, li v hn 86

Trn Nguyn Thit Qun (L Lawliet)


Phm Quang Ton (Phm Quang Ton)

Trong chng trnh THCS v THPT th phng trnh nghim nguyn


vn lun l mt ti hay v kh i vi hc sinh. Cc bi ton nghim
nguyn thng xuyn xut hin ti cc k thi ln, nh, trong v ngoi
nc. Trong bi vit ny ti ch mun cp n cc vn c bn ca
nghim nguyn (cc dng, cc phng php gii) ch khng i nghin
cu su sc v n. Ti cng khng cp ti phng trnh Pell, phng
trnh Pythagore, phng trnh Fermat v n c nhiu trong cc sch,
cc chuyn khc.

4.1 Xt tnh chia ht


4.1.1 Pht hin tnh chia ht ca 1 n
V d 4.1. Gii phng trnh nghim nguyn

13x + 5y = 175 (4.1)

57
58 4.1. Xt tnh chia ht

Li gii. Gi s x, y l cc s nguyn tha mn phng trnh (4.1). Ta


. .
thy 175 v 5y u chia ht cho 5 nn 13x..5 x..5 (do GCD(13; 5) = 1).
t x = 5t (t Z). Thay vo phng trnh (4.1), ta c

13.5t + 5y = 175 13t + y = 35 y = 35 13t

Do , phng trnh (4.1) c v s nghim nguyn biu din di dng

(x; y) = (5t; 35 13t), (t Z)


Bi tp ngh
Bi 1. Gii phng trnh nghim nguyn 12x 19y = 285

Bi 2. Gii phng trnh nghim nguyn 7x + 13y = 65

Bi 3. Gii phng trnh nghim nguyn 5x + 7y = 112

4.1.2 a v phng trnh c s


V d 4.2. Tm nghim nguyn ca phng trnh

3xy + 6x + y 52 = 0 (4.2)

Li gii. Nhn xt. i vi phng trnh ny, ta khng th p dng


phng php trn l pht hin tnh chia ht, vy ta phi gii nh th
no?
Ta gii nh sau:
(4.2) 3xy + y + 6x + 2 54 = 0
y (3x + 1) + 2 (3x + 1) 54 = 0
(3x + 1) (y + 2) = 54

Nh vy, n y ta c x v y nguyn nn 3x + 1 v y + 2 phi l c


ca 54. Nhng nu nh vy th ta phi xt n hn 10 trng hp sao?
V:

4 = 1.54 = 2.27 = 3.18 = 6.9


= (1).(54) = (2).(27) = (3).(18) = (6).(9)

Din n Ton hc Chuyn S hc


4.1. Xt tnh chia ht 59

C cch no khc khng? Cu tr li l c! Nu ta mt cht n


tha s 3x + 1, biu thc ny chia cho 3 lun d 1 vi mi x nguyn.
Vi lp lun trn, ta c:
 
3x + 1 = 1 x=0

 y + 2 = 54 y = 52

3x + 1 = 2 x = 1

y + 2 = 54 y = 56

V d 4.3. Gii phng trnh nghim nguyn sau:

2x + 5y + 3xy = 8 (4.3)

Li gii. Ta c

(4.3) x(2 + 3y) + 5y = 8


3x(2 + 3y) + 15y = 24
3x(2 + 3y) + 5(2 + 3y) = 34
(3x + 5)(3y + 3) = 34

n y phn tch 34 = 1 34 = 2 17 ri xt cc trng hp. Ch


rng 3x + 5, 3y + 2 l hai s nguyn chia 3 d 2, vn dng iu ny ta
c th gim bt s trng hp cn xt. 

V d 4.4. Gii phng trnh nghim nguyn

x2 y 2 = 2011 (4.4)

Li gii. (4.4) (x y)(x + y) = 2011. V 2011 l s nguyn t nn


c nguyn ca 2011 ch c th l 1, 2011. T suy ra nghim
(x; y) l (1006; 1005); (1006; 1005); (1006; 1005); (1006; 1005). 

V d 4.5. Tm cc s nguyn x, y tho mn iu kin

x2 + y 2 = (x y)(xy + 2) + 9 (4.5)

Chuyn S hc Din n Ton hc


60 4.1. Xt tnh chia ht

Li gii. t a = x y, b = xy. Khi (4.5) tr thnh

a2 + 2b = a(b + 2) + 9 (a 2)(a b) = 9 (4.6)

V x, y Z nn a, , a 2, a b u l cc s nguyn. T (4.6) ta c cc
trng hp sau:
( ( (
a2=9 a = 11 x y = 11
(4.7)
ab=1 b = 10 xy = 10
( ( (
a2=3 a=5 xy =5
(4.8)
ab=3 b=2 xy = 2
( ( (
a2=1 a=3 xy =3
(4.9)
ab=9 b = 6 xy = 6
( ( (
a 2 = 1 a=1 xy =1
(4.10)
a b = 9 b = 10 xy = 10
( ( (
a 2 = 3 a = 1 x y = 1
(4.11)
a b = 3 b=2 xy = 2
( ( (
a 2 = 3 a = 1 x y = 1
(4.12)
a b = 3 b=2 xy = 2

D thy cc h (4.7),(4.8),(4.10) khng c nghim nguyn, h (4.9) v


nghim, h (4.11) c hai nghim nguyn (1; 2) v (2; 1), h (4.12)
c hai nghim nguyn (1; 6) v (6; 1).
Tm li phng trnh (4.5) c cc cp nghim nguyn (x; y) l (1; 2);
(2; 1); (1; 6); (6; 1). 

V d 4.6. Tm nghim nguyn ca phng trnh:

x2 + 1 y 2 + 1 + 2 (x y) (1 xy) = 4 (1 + xy)
 
(4.13)

Din n Ton hc Chuyn S hc


4.1. Xt tnh chia ht 61

Li gii. Phng trnh (4.13) tng ng vi:

x2 y 2 + x2 + y 2 + 1 + 2x 2x2 y 2y + 2xy 2 = 4 + 4xy


(x2 + 2x + 1)y 2 2(x2 + 2x + 1)y + (x2 + 2x + 1) = 4
(x 2 2
 + 1) (y 1) = 4
(x + 1)(y 1) = 2

(x + 1)(y 1) = 2

Vi (x + 1)(y 1) = 2 m x, y Z nn ta c cc trng hp sau:


 
x+1=1 x=0

y1=2 y=3
 
x+1=2 x=1

y1=1 y=2
 
x + 1 = 2 x = 3

y 1 = 1 y=0
 
x + 1 = 1 x = 2

y 1 = 2 y = 1

Vi (x + 1)(y 1) = 2 , tng t ta cng suy ra c:


 
x + 1 = 1 x = 2

 y 1 = 2  y=3
x+1=1 x=0

 y 1 = 2  y = 1
x+1=2 x=1

 y 1 = 1  y=0
x + 1 = 2 x = 3

y1=1 y=2

Vy phng trnh cho c cc cp nghim nguyn:

(x; y) = {(0; 3); (1; 2); (3; 0); (2; 1); (2; 3); (0; 1); (1; 0); (3; 2)}

V d 4.7. Tm nghim nguyn ca phng trnh

x6 + 3x3 + 1 = y 4 (4.14)

Chuyn S hc Din n Ton hc


62 4.1. Xt tnh chia ht

Li gii. Nhn hai v ca phng trnh (4.14) cho 4, ta c:


4x6 + 12x3 + 4 = 4y 4
(4x6 + 12x3 + 9) 4y 4 = 5
(2x3 + 3)2 4y 4 = 5
(2x3 2y 2 + 3)(2x3 + 2y 2 + 3) = 5.

Vi lu rng 5 = 1.5 = 5.1 = (1).(5) = (5).(1) v x, y Z nn


ta suy ra c cc trng hp sau:
2x3 2y 2 + 3 = 1
 3
x y 2 = 1
  3
x =0

2x3 + 2y 2 + 3 = 5 x3 + y 2 = 1 y2 = 1


x=0
 y=1



x=0
y = 1

2x3 2y 2 + 3 = 1
 3
x y 2 = 2
  3
x = 3
(loi)
2x3 + 2y 2 + 3 = 5 x3 + y 2 = 4 y 2 = 1

2x3 2y 2 + 3 = 5
 3
x y2 = 1
  3
x =0
(loi)
2x3 + 2y 2 + 3 = 1 x3 + y 2 = 1 y 2 = 1

2x3 2y 2 + 3 = 5
 3
x y 2 = 4
  3
x = 3
(loi)
2x3 + 2y 2 + 3 = 1 x3 + y 2 = 2 y2 = 1

Vy phng trnh cho c cc cp nghim nguyn:

(x; y) = {(0; 1); (0; 1)}

Nhn xt. Bi ton ny cng c th gii bng phng php kp.

V d 4.8. Gii phng trnh nghim nguyn dng:


1 1 1
+ = (4.15)
x y p
trong p l s nguyn t. 4

Din n Ton hc Chuyn S hc


4.1. Xt tnh chia ht 63

Li gii.

(4.15) xy = px + py (x y)(y p) = p2 .

V p l s nguyn t nn c s nguyn ca p2 ch c th l 1, p, p2 .
Th ln lt vi cc c trn ta d tm c kt qu. Phn trnh by
xin dnh cho bn c. 

Nhn xt. Phng php ny cn hai bc chnh: Phn tch thnh c


s v xt trng hp tm kt qu. Hai bc ny c th ni l khng
qu kh i vi bn c, nhng xin ni mt s lu thm v bc xt
trng hp. Trong mt s bi ton, hng s nguyn v phi sau khi
phn tch l mt s c nhiu c, nh vy i hi xt trng hp v
tnh ton rt nhiu. Mt cu hi t ra l: Lm th no gim s
trng hp b xt y? V tr li c cu hi , ta s tham kho
v d di y.

V d 4.9. Tm nghim nguyn ca phng trnh:

x2 + 12x = y 2 . (4.16)

Li gii. (thng thng) Phng trnh (4.16) cho tng ng vi:

(x + 6)2 y 2 = 36 (x + 6 + y)(x + 6 y) = 36

Suy ra x + y + 6, x + 6 y l c ca 36. M s 36 c tt c 18 c
nn ta phi xt 18 trng hp tng ng vi

x + 6 + y {1; 2; 3; 4; 6; 9; 12; 18; 36}

. Kt qu l ta tm c cc cp nghim nguyn (x; y) l

(0; 0); (12; 0); (16; 8); (16; 8); (4; 8); (4; 8)

Nhn xt. ng nh vn m ta nu ra trn, s c qu nhiu


xt. Cho nn ta s c cc nhn xt sau thc hin thao tc "siu
phm" chuyn t con s 18 xung ch cn 2!

Chuyn S hc Din n Ton hc


64 4.1. Xt tnh chia ht

V y c s m chn trong phng trnh nn c th gi s y 0. Khi


x + 6 y x + 6 + y, do vy ta loi c tm trng hp v cn
li cc trng hp sau:
( ( (
x+6+y =9 x + 6 + y = 9 x + y + 6 = 1
, , ,
x+6y =4 x + 6 y = 4 x + y 6 = 36
( ( (
x + y + 6 = 36 x + y + 6 = 2 x + y + 6 = 18
, , ,
xy+6=1 x y + 6 = 18 xy+6=2
( ( (
x + y + 6 = 3 x + y + 6 = 12 x + y + 6 = 6
, , ,
x y + 6 = 12 xy+6=3 x y + 6 = 6
(
x+y+6=6
.
x+y6=6

By gi ta c 10 trng hp, ta s tip tc lc b. Nhn thy


(x + y + 6) (x + 6 y) = 2y nn x + 6 y v x + 6 + y c cng tnh
chn l, do ta loi thm 6 trng hp, ch cn
( (
x + y + 6 = 18 x + y + 6 = 2
, ,
x+y6=2 x + y 6 = 18
( (
x + y + 6 = 6 x+y+6=6
,
x y + 6 = 6 x+y6=6
. ( (
x + y + 6 = 6 x+y+6=6
Tip tc xt hai phng trnh v ,
x y + 6 = 6 x+y6=6
hai phng trnh ny u tm c y = 0. Vy sao khng n gin
hn, ta xt y = 0 ngay t u. Phng trnh c dng x(x + 12) = y 2 ,
xt hai kh nng:
Nu y = 0 th x = 0 hoc x = 12.
Nu y 6= 0 th x+6+y
( > x+6y, p dng hai (
nhn xt trn ta ch
x + y + 6 = 2 x + y + 6 = 18
c hai trng hp: v .
x y + 6 = 18 xy+6=2


Din n Ton hc Chuyn S hc


4.1. Xt tnh chia ht 65

Phng trnh cho c 6 nghim nguyn

(x; y) = (16; 8), (0; 0), (12; 0), (16; 8), (4; 8), (4; 8)

Nhn xt. Nh vy bi ton ngn gn, chnh xc nh linh hot trong


vic xt tnh chn l, gii hn hai s gim s trng hp cn xt.
Ngoi cc cch nh gi trn ta cn c th p dng xt s d tng v
nh gi (y cng l mt phng php gii phng trnh nghim
nguyn).

Bi tp ngh

Bi 1. Th bin i cc bi ton gii phng trnh nghim nguyn


phng php Biu th mt n theo n cn li bng phng
php a v c s.

Bi 2. Tm di cnh mt tam gic vung sao cho tch hai cnh


huyn gp ba ln chu vi tam gic .

Bi 3. Gii phng trnh nghim nguyn x y + 2xy = 6

Bi 4. Gii phng trnh nghim nguyn 2x + 5y + 2xy = 8

Bi 5. (Thi HSG lp 9 tnh Qung Ngi nm 2011-2012) Gii phng


trnh nghim nguyn 6x + 5y + 18 = 2xy

Bi 6. Tm nghim nguyn (xy 7)2 = x2 + y 2

Bi 7. Tm x, y Z tha mn 2x2 2xy = 5x y 19.

Bi 8. Tm nghim nguyn ca phng trnh x2 +6xy +8y 2 +3x+6y =


2.

Bi 9. Tm nghim nguyn dng ca phng trnh x3 y 3 = xy + 61

Bi 10. Tm nghim nguyn ca phng trnh 4x2 y 2 = 22 + x(1 + x) +


y(1 + y)

Bi 11. Gii phng trnh nghim nguyn x(x + 1)(x + 7)(x + 8) = y 2 .

Chuyn S hc Din n Ton hc


66 4.1. Xt tnh chia ht

Bi 12. Tm nghim nguyn dng ca phng trnh 6x3 xy(11x +


3y) + 2y 3 = 6 (Tp ch TTT2 s 106).

Bi 13. Tm nghim nguyn dng ca phng trnh x(x + 2y)3 y(y +


2x)3 = 27 (tp ch THTT s 398).

Bi 14. Tm nghim nguyn ca phng trnh 9x2 + 16x + 96 = 3x
16y 24.

Bi 15. Tm nghim nguyn dng ca phng trnh


s r
1 1
2+ x+ + x+ =y
2 4
.

Bi 16. Tm s nguyn x x2 4x 52 l s chnh phng.

Bi 17. Gii phng trnh nghim nguyn x2 + 2y 2 + 3xy 2x y = 6.

Bi 18. Gii phng trnh nghim nguyn x2 + 3xy y 2 + 2x 3y = 5.

Bi 19. Gii phng trnh nghim nguyn 2x2 + 3y 2 + xy 3x 3 = y.

Bi 20. (Tuyn sinh vo lp 10 THPT chuyn trng KHTN H Ni


nm hc 2012-2013) Tm tt c cc cp s nguyn x, y tha
mn ng thc (x + y + 1)(xy + x + y) = 5 + 2(x + y).

Bi 21. Gii phng trnh nghim nguyn x4 2y 4 x2 y 2 4x2 7y 2


5 = 0.
(Thi HSG lp 9 tnh Hng Yn nm 2011-2012)

Bi 22. (Romanian 1999) Chng minh rng phng trnh sau khng c
nghim nguyn

x5 x4 y 13x3 y 2 + 13x2 y 3 + 36xy 4 36y 5 = 1937

Din n Ton hc Chuyn S hc


4.1. Xt tnh chia ht 67

4.1.3 Biu th mt n theo n cn li ri s dng tnh chia


ht
V d 4.10. Tm nghim nguyn ca phng trnh
2x xy + 3 = 0 (4.17)

Li gii. Nhn xt. phng trnh ny ta khng th p dng cc cch


bit, vy ta phi lm sao? Ch hn mt xu na ta thy c th
biu din y theo x c ri vn dng kin thc tm gi tr nguyn
lp 8 tm nghim nguyn ca phng trnh, th lm theo tng
xem sao.

(4.17) xy = 2x + 3
Nu x = 0 th phng trnh (4.17) cho v nghim nguyn y.
6 0 th
Nu x =
2x + 3 3
(4.17) y = =2+
x x
3
Nh vy mun y nguyn th ta cn nguyn hay ni cch khc x l
x
c ca 3. Vi mi gi tr nguyn x ta tm c mt gi tr y nguyn.
T , ta c b nghim ca (4.17) l
(x; y) = (3; 1); (1; 1); (1; 5); (3; 3)

V d 4.11 (Thi HSG lp 9 Qung Ngi nm 2011-2012). Tm cc s


nguyn dng x, y sao cho
6x + 5y + 18 = 2xy (4.18)

Nhn xt. Hng phn tch v nh hng li gii. xc nh c


phng php ca dng ny th by gi ta s biu din n x theo y.
5y 18
Khng kh vit thnh x = . Ta dng nh nhn thy biu
6 2y
thc ny rt kh phn tch nh biu thc v d u. Tuy nhin, nu
k s thy bn mu l 2y v t l 5y, do ta mnh dn nhn 2
vo t xut hin 2y ging nh mu.

Chuyn S hc Din n Ton hc


68 4.1. Xt tnh chia ht

Li gii. Ta c

5y 18
(4.18) x =
6 2y
10y 36
2x =
6 2y
66 + 5(6 2y) 66
2x = = +5
6 2y 6 2y
33
2x = +5
3y

Nh vy mun x l s nguyn dng th 3 y l phi l c ca 33.


Hay 3 y {1; 3; 11, 33}. Li rng v y 1 nn 3 y 2.
Do ch c th 3 y {1; 3; 11; 33}. Ta c bng sau:

3y 1 1 3 11 33
y 2 4 6 14 36
x 14 19 8 4 3

Th li thy cc cp (x; y) nguyn dng tha mn (4.18) l (x; y) =


(19; 4), (8; 6), (4; 14), (3; 36). 

Nhn xt. Bi ny ta cng c th s dng phng php a v phng


trnh c s. Cng xin ch vi bn rng li gii trn th ta nhn
2 x bin i, do phi c mt bc th li coi gi tr x, y tm
c c tha mn (4.18) hay khng ri mi c th kt lun.

Bi tp ngh

Bi 1. Gii phng trnh nghim nguyn x2 xy = 6x 5y 8.

Bi 2. Gii phng trnh nghim nguyn x2 + x + 1 = 2xy + y.

Bi 3. Gii phng trnh nghim nguyn x3 x2 y + 3x 2y 5 = 0.

Bi 4. (Vo 10 chuyn THPT HKHTN H Ni nm 2001-2002) Tm


gi tr x, y nguyn tha mn ng thc (y 2)x2 + 1 = y 2 .

Din n Ton hc Chuyn S hc


4.1. Xt tnh chia ht 69

Bi 5. (Vo 10 chuyn THPT HKHTN H Ni nm 2000-2001) Tm


cp s nguyn (x, y) tha mn ng thc y(x 1) = x2 + 2.

Bi 6. Tm s nh nht trong cc s nguyn dng l bi ca 2007 v


c 4 ch s cui cng l 2008.

Bi 7. Tm nghim nguyn ca phng trnh 5x 3y = 2xy 11.

4.1.4 Xt s d tng v
C s phng php. c ngay tiu phng php th chc bn s
hiu ngay phng php ny ni n vic xt s d tng v cho cng
mt s. Vy, ti sao li phi xt v xt nh vy c li ch g trong "cng
cuc" gii ton? Hy cng tm hiu qua v d u sau:

V d 4.12. Tm nghim nguyn ca phng trnh

x2 + y 2 = 2011 (4.19)

Li gii. Ta c x2 ; y 2 chia 4 c th d 0 hoc 1 nn tng chng chia 4


ch c th d 0; 1 hoc 2. Mt khc 2011 chia 4 da 3 nn phng trnh
(4.19) v nghim nguyn. 

Nhn xt. Qua v d u ny th ta thy r s d khi chia cho 4 ca


hai s khc nhau th phng trnh v nghim. Do ta li cng hiu
thm mc ch ca phng php ny. Bt m thm t na th phng
php ny ch yu dng cho cc phng trnh khng c nghim nguyn.
Cho nn, nu bn bt gp mt phng trnh bt k m bn khng th
tm ra c nghim cho phng trnh , th hy ngh n phng
php ny u tin. Cn by gi ta tip tc n vi v d sau:

V d 4.13 (Balkan MO 1998). Tm nghim nguyn ca phng trnh

x2 = y 5 4 (4.20)

Li gii. Ta c: x2 0; 1; 3; 4; 5; 9 (mod 11). Trong khi y 5 4


6; 7; 8 (mod 11): v l. Vy phng trnh (4.20) v nghim nguyn. 

Chuyn S hc Din n Ton hc


70 4.1. Xt tnh chia ht

Nhn xt. Mt cu hi na li le ln trong u ta: Lm th no li c


th tm c con s 11 m xt ng d c nh? p n ca cu
hi ny cng chnh l ci ct li bn vn dng phng php ny, v
cng l nhng kinh nghim sau:
1. i vi phng trnh nghim nguyn c s tham gia ca cc bnh
phng th ta thng xt ng d vi 3, 4, 5, 8. C th l:
a2 0, 1 (mod 3)
a2 0, 1 (mod 4)
a2 0, 1, 4 (mod 5)
a2 0, 1, 4 (mod 8)

2. i vi cc phng trnh nghim nguyn c s tham gia ca cc


s lp phng th ta thng xt ng d vi 9, v x3 0; 1; 8
(mod 9) v ng d vi 7, v x3 0, 1, 6 (mod 7).
3. i vi phng trnh nghim nguyn c s tham gia ca cc
ly tha bc 4 th ta thng xt ng d vi 8, nh: z 4 0, 1
(mod 8).
4. Mt vn cui cng l nh l Fermat: i vi phng trnh
nghim nguyn c s tham gia ca cc ly tha c s m l mt
s nguyn t hay l mt s m khi cng 1 vo s ta c mt
s nguyn t th ta thng s dng nh l nh Fermat xt
ng d.
Trn y l mt s kinh nghim bn thn, cn nu cc bn mun vn
dng c phng php xt s d ny, yu cu hy ghi nh kinh nghim
trn v tm cch chng minh n. Ngoi ra, nu bn mun m rng tm
hiu bit hn na, bn c th tm cc ng d vi ly tha khc nhau
(chng hn qua v d 2 ta rt ra c moun 11 cho ly tha bc
hai, bc nm). Cn by gi, hy th xem kinh nghim trn c hiu qu
khng nh!
V d 4.14 (Bi ton trong tun - diendantoanhoc.net). Chng minh
rng phng trnh sau khng c nghim nguyn
x10 + y 10 = z 10 + 199

Din n Ton hc Chuyn S hc


4.1. Xt tnh chia ht 71

Nhn xt. Thng thng cc bi ton khi t cu hi phng trnh


c nghim hay khng th thng c cu tr li l khng. Do
chng minh phng trnh trn khng c nghim, th ta s tm mt con
s sao cho khi chia VT v VP cho con s ny th c hai s d khc
nhau.

Nh vy, cng vic by gi ca ta l tm con s . n s m 10


th s khin ta lin tng con s 11 l s nguyn t. Nh vy li gii
ca ta s p dng nh l Fermat nh cho s 11 chng minh hai v
phng trnh chia cho 11 khng cng s d.

10
x 0, 1 (mod 11)

Li gii. p dng nh l Fermat nh th y 10 0, 1 (mod 11) .

10
z 0, 1 (mod 11)
10 10 10
Do x + y z 0, 1, 2, 10 (mod 11) m 199 8 (mod 11) nn
phng trnh v nghim nguyn. 

V d 4.15 ( thi chn HSG ton quc gia nm 2003 - Bng B).
H phng trnh sau c tn ti nghim nguyn hay khng:

x2 + y 2 = (x + 1)2 + u2 = (x + 2)2 + v 2 = (x + 3)2 + t2 (4.21)

Nhn xt. Ta d on phng trnh trn cng s v nghim. Do


cn tm mt s v khi chia c 5 v c cc s d khc nhau. bi
ton ny c bnh phng nn ta ngh ti vic s dng cc tnh cht
nh: a2 0, 1 (mod 3), a2 0, 1 (mod 4), a2 0, 1, 4 (mod 5), a2
0, 1, 4 (mod 8). bi ton ny, ta s chn 8. By gi ch cn xt tnh
d khi chia cho 8.
Li gii. Gi s phng trnh (4.21) c nghim nguyn (x0 , y0 , u0 , v0 , t0 ),
tc l:

x20 + y02 = (x0 + 1)2 + u20 = (x0 + 2)2 + v02 = (x0 + 3)2 + t20 (4.22)

Vi a Z th a2 0, 1, 4 (mod 8). Ta xt cc kh nng sau:

Chuyn S hc Din n Ton hc


72 4.1. Xt tnh chia ht

1. Nu x0 0 (mod 4) th x20 + y02 0, 1, 4 (mod 8). V

x0 + 1 1 (mod 8) (x0 + 1)2 1 (mod 8)


2
(x0 + 1) + u20 1, 2, 5 (mod 8)
2
x0 + 2 2 (mod 4) (x0 + 2) 4 (mod 8)
2
(x0 + 2) + v02 0, 4, 5 (mod 8)
x0 + 3 3 (mod 4) (x0 + 3)2 1 (mod 8)
2
(x0 + 3) + t20 1, 2, 5 (mod 8)

Nhn thy {0, 1, 4} {1, 2, 5} {0, 4, 5} {1, 2, 5} = 0 nn do


phng trnh khng c nghim nguyn vi x 0 (mod 4).

2. Tng t vi x0 1 (mod 4), x0 2 (mod 4) v x0 3 (mod 4)


ta cng thc hin tng t v cng cho kt qu phng trnh
khng c nghim nguyn.

Vy phng trnh (4.21) cho khng c nghim nguyn. 

Nhn xt. V d 4 ta c th tng qut ln:

V d 4.16. Tm s nguyn dng n ln nht sao cho h phng trnh

(x + 1)2 + y12 = (x + 2)2 + y22 = . . . = (x + n)2 + yn2

c nghim nguyn. 4

y cng chnh l thi chn i tuyn HSG quc gia ton nm 2003
- Bng A. Li gii xin ginh cho bn c. Cng xin ni thm mt tha
nhn rng, phng php xt s d tng v ny, chng ta c tng
chng nh n gin, nhng thc cht khng phi th. Dn chng l
cc v d trn, u l cc bi ton hay v kh ly t khc cuc thi
trong nc v ngoi nc.

Bi tp ngh

Bi 1. Cho a thc f (x) c cc h s nguyn. Bit rng f (1).f (2) l s


l. Chng minh rng phng trnh f (0) = 0 khng c nghim
nghim nguyn.

Din n Ton hc Chuyn S hc


4.1. Xt tnh chia ht 73

Bi 2. Tn ti hay khng nghimnguyn ca phng trnh x12 + y 12 +


z 12 = 2 372012 + 20141995 .

Bi 3. Gii phng trnh nghim nguyn 312x + 122x + 19972x = y 2 .

Bi 4. Gii phng trnh nghim nguyn dng 7z = 2x 3y 1

Bi 5. Gii phng trnh nghim nguyn dng 2x 3y = 1 + 5z


30
Bi 6. Gii phng trnh nghim t nhin 19x + 5y + 1890 = 19754 +
1993.

Bi 7. Gii phng trnh nghim nguyn x3 + y 3 + z 3 = 1012

Bi 8. (Tuyn sinh vo lp 10 chuyn Trn Ph, Hi Phng nm hc


2012-2013) x4 + y 4 + z 4 = 2012
10n 1
Bi 9. |x y| + |y z| + |z x| = vi mi n N
9
Bi 10. Tm nghim nguyn ca phng trnh (2x + 1)(2x + 2)(2x +
3)(2x + 4) 5y = 11879

Bi 11. Tm nghim nguyn ca phng trnh x2 + (x + 1)2 + (x + 2)2 =


y2.

Bi 12. (Tuyn sinh vo THPT chuyn HKHTN H Ni nm 2011-


2012) Chng minh rng khng tn ti b ba s nguyn (x; y; z)
tha mn x4 + y 4 = 7z 4 + 5.

Bi 13. Gii phng trnh nghim nguyn x41 +x42 + = x413 +20122015.

Bi 14. Cho p l s nguyn t l. Chng minh rng phng trnh xp +


y p = p [(p 1)!]p khng c nghim nguyn

Bi 15. Tm nghim nguyn ca phng trnh x2012 y 2010 = 7.

Bi 16. Chng minh rng khng tn ti s nguyn x, y tha mn x5 +


y 5 + 1 = (x + 2)5 + (y 3)5 .

Chuyn S hc Din n Ton hc


74 4.2. S dng bt ng thc

4.2 S dng bt ng thc


4.2.1 Sp th t cc n
V d 4.17. Gii phng trnh nghim nguyn dng sau
1 1 1
+ + =1 (4.23)
x y z

Li gii. Khng mt tnh tng qut, ta c th gi s


1 1 1 3
1xyz + + =1 x3
x y z x

Vi x = 1 th (4.23) khng c nghim nguyn dng.


1 1 1 1 1 1 2
Vi x = 2 th + + = 1 + = y 4 Mt
2 y z y z 2 y
khc, y x = 2 y {2; 3; 4}. Ta th ln lt cc gi tr ca y

Vi y = 2 th (4.23) v nghim nguyn.


Vi y = 3 th z = 6.
Vi y = 4 th z = 4.
1 1 1 1 1 2 2
Vi x = 3, ta c + + = 1 + = y 3 Mt
3 y z y z 3 y
khc, do y x = 3 y = 3 z = 3

Vy nghim nguyn (x; y; z) ca (4.23) l hon v ca cc b (2; 3; 6);


(2; 4; 4); (3; 3; 3). 

Nhn xt. Phng php ny c s dng ch sp th t cc n


1 x y z ri gii hn nghim gii.
Ta ch s dng phng php sp th t cc n khi vai tr cc n l
bnh ng vi nhau. D khi vn dng phng php ny cc bn cn
ch trnh nhm ln. C th, ta s n vi v d sau:

V d 4.18. Gii phng trnh nghim nguyn dng

x + y + 1 = xyz (4.24)

Din n Ton hc Chuyn S hc


4.2. S dng bt ng thc 75

Li gii (Li gii sai). Khng mt tnh tng qut, gi s 1 x y


z. Khi x+y +1 3z hay xyz 3z, suy ra xy 3. M z y x 1
nn x = y = z = 1.

Nhn xt. Ci li sai li gii ny l do x, y, z khng bnh ng, nn


khng th sp th t cc n nh trn. Sau y l li gii ng:
Li gii. Khng mt tnh tng qut, gi s 1 x y. Ta xt trng
hp:

Nu x = y th

(4.24) 2y + 1 = y 2 z
y(z 2) = 1
(
y=1

yz 2 = 1
(
y=1

z=3

Nu x < y th t (4.24) suy ra 2y + 1 > xyz. 2y xyz


xz 2 xz {1; 2}.

Vi xz = 1 x = z = 1, thay vo (4.24) suy ra y + 2 = y


(v nghim).
( (
x=1 x=2
Vi xz = 2 hoc . T y ta tm
z=2 z=1
c nghim x = 1, y = 2, z = 2 hoc x = 1, y = 3, z = 1.

Vy phng trnh c nghim nguyn dng l (1; 1; 3), (1; 2; 2), (2; 1; 2),
(2; 3; 1), (3; 2; 1). 

Nhn xt. By gi bn hiu v cch sp xp cc n nh th no.


Nhng ti sao bi ny li xt x = y v x < y m li khng i vo phn

Chuyn S hc Din n Ton hc


76 4.2. S dng bt ng thc

tch lun nh bi trc. Nu bn rng nu khng phn chia thnh


hai trng hp nhu trn th phng trnh (4.24) s thnh 2y+1 y 2 z,
rt kh tip tc phn tch ra nghim. Do vic xt nhu trn l
hp l.
Bi tp ngh
Bi 1. Gii phng trnh nghim nguyn dng 2(x+y+z)+9 = 3xyz.
Bi 2. Gii phng trnh nghim nguyn dng xyz = 3(x + y + z).
Bi 3. Gii phng trnh nghim nguyn dng 5(x + y + z + t) + 10 =
2xyzt
Bi 4. Gii phng trnh nghim nguyn dng x! + y! = (x + y)!
(K hiu x! l tch cc s t nhin lin tip t 1 n x).
Bi 5. Tm nghim nguyn dng ca phng trnh x3 + 7y = y 3 + 7x.
Bi 6. Tm nghim nguyn dng ca phng trnh x1 +x2 + +x12 =
x1 x2 x12 .
x
Bi 7. Tm tt c cc nghim nguyn dng ca phng trnh 2 2 +
y z
y z
+ = t.
z 2 x2 x2 y 2
Bi 8. Tm nghim nguyn dng ca phng trnh x! + y! + z! = u!.

4.2.2 S dng bt ng thc


Nhn xt. gii phng trnh ny, ta thng s dng cc bt ng
thc quen thuc nh gi mt v ca phng trnh khng nh hn
(hoc khng ln hn) v cn li. Mun cho hai v bng nhau th bt
ng thc phi tr thnh ng thc.
C th, ta c mt s bt ng thc c bn thng dng:

1. Bt ng thc Cauchy (hay cn gi l bt ng thc AM-GM):


Nu a1 , a2 , , an l cc s thc khng m th
a1 + a2 + + an
n a1 a2 an
n

Din n Ton hc Chuyn S hc


4.2. S dng bt ng thc 77

Du ng thc xy ra khi v ch khi a1 = a2 = = an .

2. Bt ng thc Bunhiacopxki (hay cn c gi l bt ng thc


Cauchy - Bunyakovsky - Schwarz): Vi hai b s thc bt k
(a1 , a2 , , an ) v (b1 , b2 , , bn ), ta c

a21 + a22 + + a2n b21 + b22 + + b2n


 

(a1 b1 + a2 b2 + + an bn )2 .

ng thc xy ra khi v ch khi tn ti s thc k sao cho ai = kbi


vi mi i = 1, 2, , n.

3. Bt ng thc Trebusep (hay cn vit l bt ng thc Chebyshev):


Cho dy hu hn cc s thc c sp theo th t a1 a2
an v b1 b2 bn . Khi ta c:

n(a1 b1 + a2 b2 + + an bn ) (a1 + a2 + + an )(b1 + b2 + + bn )



a1 = a2 = = an
Du ng thc xy ra khi v ch khi .
b1 = b2 = = bn

By gi ta s cng xem xt mt s v d sau:

V d 4.19. Gii phng trnh nghim nguyn dng sau:

x6 + z 3 15x2 z = 3x2 y 2 z (y 2 + 5)3 (4.25)

Li gii. Nhn xt. phng trnh ny khi mi nhn vo hn a s


cc bn s c phn ri, khng xc nh c phng php lm, khng
vn dng c cc phng php hc. Tuy nhin nu k mt
x th ta thy x6 = (x2 )3 iu ny c g c bit? Ta thy (x2 )3 , z 3 v
(y 2 + 5)3 u c cng bc ba v bi cho nguyn dng nn ta
ngh ngay n mt Bt ng thc kinh in: Bt ng thc Cauchy
hay cn gi l bt ng thc AM-GM.
Ta gii nh sau

(4.25) (x2 )3 + (y 2 + 5)3 + z 3 = 3x2 z(y 2 + 5)

Chuyn S hc Din n Ton hc


78 4.2. S dng bt ng thc

p dng Bt ng thc AM-GM cho b ba s dng (x2 )3 , z 3 v (y 2 +


5)3 ta c:
p
(x2 )3 +(y 2 +5)3 +z 3 3 3 (x2 )3 .(y 2 + 5)3 .z 3 = 3x2 z(y 2 +5) = V P (4.25)

Du bng ch xy ra khi x2 = y 2 + 5 = 5.
Mt khc ta c:

x2 = y 2 + 5 (x y)(x + y) = 5
y l mt dng phng trnh nghim nguyn quen thuc ta hc,
ti tin chc cc bn u c th d dng gii phng trnh trn, v t
x; y trn ta c th tm c z mt cch d dng.
p s: Nghim nguyn ca phng trnh (4.25) l (x; y; z) = (3; 2; 9).

V d 4.20. Tm nghim nguyn ca phng trnh

(x + y + z)2 = 3(x2 + y 2 + 1)

Li gii. p dng bt ng thc Bunyakovsky cho hai b s (x, y, 1)


v (1, 1, 1) ta c

(x + y + 1)2 (12 + 12 + 12 )(x2 + y 2 + 1) = 3(x2 + y 2 + 1)

ng thc xy ra khi v ch khi x = y = 1.


Vy phng trnh c nghim nguyn l (x, y) = (1, 1). 

Nhn xt. Cc bi Ton v phng trnh nghim nguyn m gii bng


cch s dng Bt ng thc rt t dung v rt d b l dng nu
ngi ra khng kho lo. Tuy nhin, ta vn phi thnh tho phng
php ny khng c xem thng n trnh nhng sai lm ng tic
khng th sa c.

Bi tp ngh

Bi 1. Tm nghim nguyn dng x, y tha mn phng trnh (x2 +


1)(x2 + y 2 ) = 4x2 y
xy yz zx
Bi 2. Tm nghim nguyn ca phng trnh + + = 3.
z x y

Din n Ton hc Chuyn S hc


4.2. S dng bt ng thc 79

Bi 3. ( thi tuyn sinh vo i hc Vinh) Tm nghim nguyn ca


phng trnh

(x2 + 1)(y 2 + 4)(z 2 + 9) = 48xyz

Bi 4. Gii phng trnh nghim nguyn


4 1 25 p
+ + = 16 x 2 y 1 z 5
x2 y1 z5

2 2
x + z = 9

Bi 5. Tm nghim nguyn ca h phng trnh y 2 + t2 = 16

xt + yz = 12

Bi 6. Tm nghim nguyn dng ca phng trnh x3 +y 3 6xy +8 =


0.
(
xy + yz + zx = 12
Bi 7. Tm nghim nguyn ca h phng trnh .
x4 + y 4 + z 4 = 48

Bi 8. Cho phng trnh x3 + y 3 + z 3 = nxyz.

a, Chng minh rng khi m = 1 v m = 2 th phng trnh


khng c nghim nguyn dng.
b, Gii phng trnh nghim nguyn dng khi m = 3.

Bi 9. Gii phng trnh nghim nguyn dng (x3 +y 3 )+4(x2 +y 2 )+


4(x + y) = 16xy.

Bi 10. Gii phng trnh nghim nguyn dng

3(x4 + y 4 + x2 + y 2 + 2) = 2(x2 x + 1)(y 2 y + 1)

Bi 11. Gii phng trnh nghim nguyn dng vi x, y, z l cc s i


mt khc nhau

x3 + y 3 + z 3 = (x + y + z)2

Chuyn S hc Din n Ton hc


80 4.2. S dng bt ng thc

4.2.3 Ch ra nghim
Nhn xt. Phng php ny dnh cho nhng bi ton gii phng
trnh nghim nguyn khi m ta tm c chnh xc nghim nguyn
v mun chng minh phng trnh ch c duy nht cc nghim nguyn
m thi.

V d 4.21. Tm nghim nguyn dng ca phng trnh

2x + 3x = 5x (4.26)

Li gii. Chia 2 v ca phng trnh (4.26) cho s dng 5x , ta c:


 x  x
2 3
(4.26) + =1
5 5
2 3
Vi x = 1 th ta c + = 1:ng nn x = 1 l 1 nghim ca
5 5
(4.26).
Vi x > 1 th
 x  x
2 3 2 3
+ > + =1
5 5 5 5
Do mi gi tr x > 1 u khng l nghim ca (4.26). Vy nghim
nguyn dng ca (4.26) l x = 1. 

Nhn xt. v d trn, ta d nhn thy x = 1 l nghim duy nht


ca phng trnh nn ch cn chng minh vi x > 1 th phng trnh
v nghim. Ngoi ra, t bi ton trn ta c th m rng thnh hai bi
ton mi:

Bi ton 4.1. Tm nghim nguyn dng ca phng trnh



( 3)x + ( 4)x = ( 5)x

Bng cch gii tng t ta cng tm c nghim duy nht ca phng


trnh trn l x = 4.

Din n Ton hc Chuyn S hc


4.2. S dng bt ng thc 81

Bi ton 4.2. Tm nghim nguyn dng ca phng trnh

3x + 4y = 5z

Bi ton 4.2 r rng c nng cao ln r rt, nhng li gii ca bi


ton ny l s dng phng php xt s d hc. Sau y l li gii
rt p ca khanh3570883 hin l iu hnh vin THPT ca VMF:
Li gii. Xt theo module 3 ta c:

5z (1)z (mod 3) 4y (1)z (mod 3) z = 2h (h N)


(5h 2y )(5h + 2y ) = 3x

Do 5h 2y v 5h + 2y khng ng thi chia ht cho 3 nn 5h + 2y = 3x


v 5h 2y = 1.
Ta c 5h +2y (1)h +(1)y = 0 (mod 3) v 5h 2y (1)h (1)y =
1 (mod 3) h l v y chn.
Nu y > 2 th 5h + 2y 1 (mod 4) 3x 1 (mod 4) 3x 1
(mod 8).
Mt khc 5 5h + 2y (mod 8) 5 3x (mod 8) 5 1 (mod 8):
v l.
Do y = 2. Suy ra x = y = z = 2. 

Phng php ny thng hay s dng cho cc phng trnh c n s


m v cc phng trnh c nghim nh.

4.2.4 S dng ca phng trnh bc 2


Nhn xt. Vit phng trnh di dng phng trnh bc hai i vi
mt n, dng iu kin. 0 hoc l s chnh phng. Ta s ty
trng hp chn mt trong hai cch xt vo vic gii ton.

V d 4.22. Gii phng trnh nghim nguyn

3x2 + (3y 1)x + 3y 2 8y = 0 (4.27)

Chuyn S hc Din n Ton hc


82 4.2. S dng bt ng thc

Li gii. Coi (4.27) l phng trnh bc 2 n x. Xt x = 27y 2 +


9y + 1.
(4.27) c nghim x th

x 0 27y 2 + 9y + 1 0 0, 01 y 3, 3 y {0; 1; 2; 3}

Nu y = 0 3x2 x = 0 x = 0 v x Z.
Nu y = 1 3x2 + 2x 5 = 0 x = 1 v x Z.
Nu y = 2 hoc y = 3 th khng tm c x nguyn nn loi.
Vy (4.27) c nghim nguyn (x; y) = (0; 0); (1; 1). 

V d 4.23. Gii phng trnh nghim nguyn

3x2 y 2 2xy 2x 2y + 8 = 0 (4.28)

Li gii. Ta c

(4.28) y 2 + 2(x + 1)y (3x2 2x + 8) = 0


0y = (x + 1)2 + 3x2 2x + 8 = 4x2 + 9

(4.28) c nghim th 0y = 4x2 +9 l s chnh phng. t 4x2 +9 =


k 2 vi k N, ta a v phng trnh c s v tm c x {2; 0; 2}.

Vi x = 2 ta c y 2 + 6y 16 = 0 nn y {8; 2}.

Vi x = 0 th y 2 + 2y 8 = 0 nn y {4; 2}.

Vi x = 2 th y 2 2y 24 = 0 nn y {6; 4}.

Kt lun. Vy phng trnh (4.28) c nghim (x; y) l (2; 8), (2; 2),
(0; 4), (0; 2), (2; 6), (2; 4). 

Nhn xt. Hai bi ton trn u c th s dng phng php a v


phng trnh c s gii.

Bi tp ngh

Bi 1. Tm cc phng php trc (nht l phng php a v


phng trnh c s) cc bi ton gii bng phng php
ny.

Din n Ton hc Chuyn S hc


4.2. S dng bt ng thc 83

Bi 2. Tm nghim nguyn ca phng trnh x + xy + y = x2 + y 2 .

Bi 3. Gii phng trnh nghim nguyn 10x2 + 5y 2 + 38 12xy +


16y 36x = 0.

Bi 4. Tm nghim nguyn phng trnh 9x2 + x2 + 4y 2 + 34 12xy +


20y 36x = 0.

Bi 5. Tm nghim nguyn dng ca x + 2y 2 + 3xy + 3x + 5y = 14.

Bi 6. Tm nghim nguyn phng trnh x2 xy6y 2 +2x6y10 = 0.

Bi 7. Tm nghim nguyn ca phng trnh x2 +2y62+3xy+3x+5y =


15.

Bi 8. Tm nghim nguyn ca phng trnh 2x2 + 6y 2 + 7xy x y =


25.

Bi 9. Tm nghim nguyn ca phng trnh 9x2 10y 2 9xy + 3x


5y = 9.

Bi 10. Tm nghim nguyn ca phng trnh 12x2 +6xy+3y 2 = 28(x+


y).
(Thi vo lp 10 chuyn, HKHTN-HQGHN nm 1994)

Bi 11. Tm nghim nguyn ca phng trnh 3(x2 + xy + y 2 ) = x + 8y.

Bi 12. Tm nghim nguyn ca phng trnh 7(x2 + xy + y 2 ) = 39(x +


y).

Bi 13. Tm nghim nguyn ca phng trnh 2x2 + y 2 + 3xy + 3x +


2y + 2 = 0.

Bi 14. Tm nghim nguyn ca phng trnh x2 +2y 2 +3xyxy+3 =


0.

Bi 15. Tm nghim nguyn ca phng trnh 3x2 +4y 2 +12x+3y +5 =


0.

Chuyn S hc Din n Ton hc


84 4.2. S dng bt ng thc

4.2.5 Phng php kp


Nhn xt. S dng tnh cht ly tha cng bc ca s nguyn lin
tip hoc tch cc s nguyn lin tip ... a phng trnh nghim
nguyn cn gii v dng phng trnh khc t n hn v quen thuc
hn. Phng php ny cn c cch gi khc l phng php kh n.
Ta thng vn dng cc nhn xt sau:
1. X n Y n (X + a)n (a N ) th Y n = (X + a i)n vi
i = 0; 1; 2; ; a.
V d vi n = 2 th:
Khng tn ti x Z a2 < x2 < (a + 1)2 vi a Z.
Nu a2 < x2 < (a + 2)2 th x2 = (a + 1)2
2. X(X + 1) (X + n) Y (Y + 1) (Y + n) (X + a)(X + a +
1) (X + a + n) th Y (Y + 1) (Y + n) = (X + i)(X + 1 +
i) (X + a + i) vi i = 0; 1; 2; ; a.
V d:
Khng tn ti b Z a(a + 1) < b(b + 1) < (a + 1)(a + 2)
vi a Z.
Vi a(a + 1) < b(b + 1) < (b + 2)(b + 3) th b(b + 1) =
(b + 2)(b + 3).
V d 4.24. Tm cc s nguyn dng x biu thc sau l s chnh
phng
A = x4 + 2x3 + 2x2 + x + 3 (4.29)
Li gii. V A l s chnh phng nn ta c th t

A = x4 + 2x3 + 2x2 + x + 3 = y 2 (y N)

Ta thy
y 2 = (x4 + 2x3 + x2 ) + x2 + x + 3
1 2 11
 
2 2
= (x + x) + x + +
2 4
> (x2 + x)2
y 2 > (x2 + x)2 , (i)

Din n Ton hc Chuyn S hc


4.2. S dng bt ng thc 85

Nu x = 1 A = 9: l s chnh phng nn tha .


Nu x > 1 th xt hiu

(x2 +x+1)2 y 2 = x2 +x2 = (x+2)(x1) > 0 y 2 < (x2 +x+1)2 , (ii)

T (i) v (ii), ta c

(x2 + x)2 < y 2 < (x2 + x + 1)2

Suy ra, khng tn ti y N y 2 = A khi x > 1.


Vy x = 1 l gi tr cn tm. 

V d 4.25. Gii phng trnh nghim nguyn

x4 + x2 + 4 = y 2 y (4.30)

Li gii. Ta c nh gi sau

x2 (x2 + 1) < x4 + x2 + 4 < (x2 + 2)(x2 + 3) (4.31)

T (4.30) v (4.31) suy ra

x2 (x2 + 1) < y(y 1) < (x2 + 2)(x2 + 3). (4.32)

V x, y, z nguyn nn t (4.32) suy ra

y(y 1) = (x2 + 1)(x2 + 2) (4.33)

T (4.30) v (4.33) th

x4 + x2 + 4 = (x2 + 1)(x2 + 2) x2 = 1 x = 1

T y d tm c y = 1 hoc y = 3.
Vy pt cho c bn cp nghim

(x, y) = {(1, 2), (1, 3), (1, 2), (1, 3)}

Bi tp ngh
Tm nghim nguyn ca cc phng trnh sau:

Chuyn S hc Din n Ton hc


86 4.3. Nguyn tc cc hn, li v hn

Bi 1. x4 + x2 + 1 = y 2

Bi 2. 3(x4 + y 4 + x2 + y 2 + 2) = 2(x2 x + 1)(y 2 y + 1)

Bi 3. 2x4 + 3x2 + 1 y 2 = 0

Bi 4. x2 + (x + y)2 = (x + 9)2

Bi 5. y 3 x3 = 2x + 1

Bi 6. x4 y 4 + z 4 + 2x2 z 2 + 3x2 + 4z 2 + 1 = 0

Bi 7. x3 y 3 2y 2 3y 1 = 0

Bi 8. x4 + (x + 1)4 = y 2 + (y + 1)2

Bi 9. 9x 3x = y 4 + 2y 3 + y 2 + 2y

Bi 10. x4 + x2 y 2 + y + 10 = 0

Bi 11. x6 4y 3 4y 4 = 2 + 3y + 6y 2

Bi 12. (x 2)4 x4 = y 3

Bi 13. x3 + 8x2 6x + 8 = y 3

4.3 Nguyn tc cc hn, li v hn


4.3.1 Li v hn
V d 4.26 (Korea 1996). Gii phng trnh nghim nguyn sau:

x2 + y 2 + z 2 = 2xyz (4.34)

Li gii. Gi s (x0 ; y0 ; z0 ) l b nghim nguyn ca (4.34) th ta c

x20 + y02 + z02 = 2x0 y0 z0


R rng VT (4.34) chn do VP (4.34) chn nn c 2 trng hp xy
ra:

Din n Ton hc Chuyn S hc


4.3. Nguyn tc cc hn, li v hn 87

Trng hp 1. Trong x0 ; y0 ; z0 , c 2 s l, 1 s chn. Khng mt tnh


tng qut, gi s x0 ; y0 l cn z0 chn. Xt theo module 4 th

V T (4.34) 2 (mod 4), V P (4.34) 0 (mod 4) : v l!

Vy trng hp ny khng xy ra.


Trng hp 2. x0 ; y0 ; z0 u chn. t x0 = 2x1 ; y0 = 2y1 ; z0 = 2z1
vi x1 ; y1 ; z1 Z. Thay vo (4.34) v rt gn, ta thu c

x21 + y12 + z12 = 4x1 y1 z1

Lp lun nh trn, ta li c x1 ; y1 ; z1 u chn.


.
Qu trnh din ra tip tc nn x0 ; y0 ; z0 ..2k vi k t nhin ty .
iu ch xy ra khi v ch khi x0 = y0 = z0 = 0. 

4.3.2 Nguyn tc cc hn
nh ngha 4.1 Nguyn tc cc hn hay cn gi l nguyn l khi u
cc tr. V mt hnh thc th phng php ny khc vi phng php
li v hn nhng cch s dng u nh nhau u chng minh phng
trnh ch c nghim tm thng (nghim tm thng l nghim bng
0). Phng php gii nh sau:
Gi s (x0 ; y0 ; z0 ; ...) l nghim ca f (x; y; z; ...) vi mt iu kin no
rng buc b (x0 ; y0 ; z0 ; ...). Chng hn x0 nh nht hoc x0 + y0 +
z0 + ... nh nht v sau bng cc php bin i s hc ta li tm
c 1 b nghim (x1 ; y1 ; z1 ; ...) tri vi nhng iu kin rng buc
trn. V d ta chn b (x0 ; y0 ; z0 ; ...) vi iu kin x0 nh nht sau
ta li tm c 1 b (x1 ; y1 ; z1 ; ...) vi x1 < x0 dn n phng trnh
c nghim tm thng. 4

V d 4.27. Gii phng trnh nghim nguyn sau

8x4 + 4y 4 + 2z 4 = t4 (4.35)

Li gii. Gi s (x0 ; y0 ; z0 ; t0 ) l nghim nguyn khng tm thng ca


(4.35) vi x0 nh nht.

Chuyn S hc Din n Ton hc


88 4.3. Nguyn tc cc hn, li v hn

T (4.35) suy ra t0 chn. t t = 2t1 (t1 Z) th vo (4.35) v rt gn,


ta c
4x4o + 2yo4 + zo4 = 8t41
Do vy z0 chn. t z0 = 2z1 (z1 Z), th vo v rt gn ta c

2x4o + yo4 + 8z14 = 4t41

Do vy y0 chn. t y0 = 2y1 (y1 Z), th vo v rt gn ta c

x4o + 8y14 + 4z14 = 2t41

Do vy x0 chn. t x0 = 2x1 (x1 Z), th vo phng trnh ta c

8x41 + 4y14 + 2z14 = t41

Suy ra (x1 ; y1 ; z1 ; t1 ) cng l nghim ca (4.35) . D thy x1 < x0 (v


l vi iu gi s). Do phng trnh c nghim nguyn duy nht l
(x; y; z; t) = (0; 0; 0; 0). 

Bi tp ngh

Bi 1. Gii cc phng trnh nghim nguyn x2 + y 2 = 3z 2

Bi 2. Gii cc phng trnh nghim nguyn x3 + 2y 3 = 4z 3

Bi 3. Gii cc phng trnh nghim nguyn 3x2 + 6y 2 + 12z 2 = t2

Bi 4. Gii cc phng trnh nghim nguyn x2 + 6y 2 + 2z 2 = 4t2

Bi 5. Gii phng trnh nghim nguyn x2 + y 2 + z 2 + t2 = x2 y 2 z 2 .

Bi 6. Gii phng trnh nghim nguyn 5x3 + 11y 3 + 13z 3 = 0.

Din n Ton hc Chuyn S hc


Chng

5
Phng trnh ng d
5.1 Phng trnh ng d tuyn tnh 89
5.2 Phng trnh ng d bc cao 90
5.3 H phng trnh ng d bc nht
mt n 90
5.4 Bc ca phng trnh ng d 95
5.5 Bi tp 95
5.6 ng dng nh l Euler gii
phng trnh ng d 96
5.7 Bi tp 101

Trn Trung Kin (Ispectorgadget)


Nguyn nh Tng (tungc3sp)

5.1 Phng trnh ng d tuyn tnh


nh ngha 5.1 Phng trnh ng d dng ax b (mod m) c gi
l phng trnh ng d tuyn tnh vi a, b, m l cc s bit.
x0 l mt nghim ca phng trnh khi v ch khi ax0 b (mod m).
Nu x0 l mt nghim ca phng trnh th cc phn t thuc lp x0
cng l nghim. 4

V d 5.1. Gii phng trnh ng d sau: 12x 7 (mod 23)


Li gii. Do (12; 23) = 1 nn phng trnh lun c nghim duy nht.
Ta tm mt s nguyn sao cho 7 + 23k chia ht cho 12. Chn k = 7
suy ra 12x 7.24 (mod 23) x 14 (mod 23) 

89
90 5.2. Phng trnh ng d bc cao

V d 5.2. Gii phng trnh 5x 2 (mod 7) 4


Li gii. V (5; 2) = 1 nn tn ti s k = 4 sao cho 2 + 7k chia ht cho
30
5. Khi y 5x 2 + 6.7 (mod 7) ta c nghim x 6 (mod 7)
5
hay x = 6 + 7k 

V d 5.3. Gii phng trnh: 5x 4 (mod 11) 4


Li gii. Ta c: 
5x 4 (mod 11)
4 4 (mod 11)

p dng tnh cht bc cu ta c: 5x 4 (mod 11) 5x = 11t + 4


Ta c th ly t = 1; x = 3. T phng trnh c nghim duy nht l
x 3 (mod 11) 

Nhn xt. Cch xc nh nghim ny l n gin nhng ch dng c


trong trng hp a l mt s nh hoc d thy ngay s k.

5.2 Phng trnh ng d bc cao


V d 5.4. Gii phng trnh 2x3 + 4 0 (mod 5) 4
Li gii. Ta thy x = 2 suy ra 2x3 4 (mod 5).
Nn x = 2 l nghim duy nht ca phng trnh cho. 

5.3 H phng trnh ng d bc nht mt n


nh ngha 5.2 H phng trnh c dng sau c gi l h phng
trnh ng d bc nht mt n


x b1 (mod m1 )
x b2 (mod m2 )


....
x bk (mod mk )

Vi m1 ; m2 ; ...mk l nhng s nguyn ln hn 1 v b1 ; b2 ; ...; bk l nhng


s nguyn ty . 4

Din n Ton hc Chuyn S hc


5.3. H phng trnh ng d bc nht mt n 91

Nhn xt. Trong trng hp tng qut, chng ta c th chng


minh c rng: iu kin cn v h phng trnh (5.2) c
nghim l U CLN (mi ; mj ) chia ht bi bj vi i 6= j(1 i, j k).
a2
Gi s m = p1 k
1 p2 ...pk l phn tch tiu chun ca m. Khi
y phng trnh ng d f (x) 0 (mod m) tng ng vi h
phng trnh ng d f (x) 0 (mod p1 i ), i = 1, 2, ..., k. T
suy ra rng nu x b1 (mod p1 1 ) l mt nghim ca phng
trnh f (x) 0 (mod pi ), i = 1, 2, ..., k th nghim ca h phng
trnh ca h phng trnh ng d

x b1 (modp1 1 )


x b2 (modp2 2 )


...
x bk modpk k



cho ta nghim ca phng trnh f (x) 0(modm).


Vy trong Trng hp tng qut gii mt phng trnh ng
d dn n gii h trn. Vi cc module m1 , m2 , ..., mk i mt
nguyn t cng nhau.

Phng php chung gii:

Trng hp 1: h 2 phng trnh



x b1 (mod m1 )
x b2 (mod m2 )

Vi gi thit d = (m1 , m2 ) chia ht cho b1 b2 . Trc tin ta nhn


xt rng, mi s x = b1 + m1 t, t Z l nghim ca phng trnh
th nht. Sau ta tm cch xc nh t sao cho x nghim ng
phng trnh th hai, ngha l h hai phng trnh trn tng
ng vi h phng trnh

x = b1 + m1 t
b1 + m1 t b2 (mod m2 )

Chuyn S hc Din n Ton hc


92 5.3. H phng trnh ng d bc nht mt n

V gi thit d = (m1 , m2 ) l c b1 b2 nn phng trnh: b1 +


m1 t b2 (mod m2 ) tng ng vi phng trnh:
m1 b2 b1 m2
t (mod )
d d d
m1 m2
Nhng ( , ) = 1 nn phng trnh ng d ny cho ta
d d
m2
nghim t t0 (mod ), l tp hp tt c cc s nguyn
d
m2
t = t0 + u, u Z
d
Thay biu thc ca t vo biu thc tnh x ta c tp hp cc
gi tr ca x nghim ng c hai phng trnh ng d ang xt
l:
m2
x = b1 + m1 (t0 + u) = b1 + m1 t0 + m1dm2 u, hay x = x0 + mu
d
vi x0 = b1 + m1 t0 , m = BCN N (m1 , m2 ).

Vy x x0 (mod m) l nghim ca h hai phng trnh ng d


ang xt.
Trng hp 2: H gm n phng trnh. u tin gii h hai
phng trnh no ca h cho, ri thay trong h hai phng
trnh gii bng nghim tm thy, ta s c mt h gm n 1
phng trnh tng ng vi vi h cho. Tip tc nh vy
sau n 1 bc ta s c nghim cn tm.


x 26 (mod 36)
x 62 (mod 60)

V d 5.5. Gii h phng trnh: 4

x 92 (mod 150)
x 11 (mod 231)

Li gii. H hai phng trnh:
 
x 26 (mod 36) x = 26 + 36t
, t Z.
x 62 (mod 60) 26 + 36t 62

26 + 36t 62 (mod 60)


36t 36 (mod 60)
t 1 (mod 5)

Din n Ton hc Chuyn S hc


5.3. H phng trnh ng d bc nht mt n 93

Vy nghim ca h l: x 26 + 36.1 (mod 180) hay x 62 (mod 180)


Do h phng trnh cho tng ng vi h:

x 62 (mod 180)
x 92 (mod 150)
x 11 (mod 231)

V d 5.6. Gii h phng trnh


 
x 62 (mod 180) x = 62 + 180t
, t Z.
x 92 (mod 150) 62 + 180t 92 (mod 150)

Li gii. Ta c:

62 + 180t 92 (mod 1)50)


180t 30 (mod 150)
6t 1 (mod 5) t1 (mod 5)

Vy nghim ca h l:

x 62 + 180.(1+) (mod 900) x 242 (mod 900)

H cho tng ng vi:



x 242 (mod900)
x 11 (mod231)

H ny c nghim x 242 (mod 69300) , v y cng l nghim ca


h cho cn tm. 

V d 5.7. Tm s nguyn dng nh nht tha tnh cht: chia 7 d 5,


chia 11 d 7 v chia 13 d 3. 4
Li gii. Ta c: n1 = 7; N1 = 11.13 = 143; n2 = 11; N2 = 7.13 =
91; n3 = 13; N3 = 7.11 = 77.
Ta c N1 b1 3b1 1 (mod 7) b1 = 2. Tng t b2 = 4; b3 = 1
Vy a = 143(2)5 + (91)(4)(7) + (77)(1)(3) = 1430 + 2548 231 =
887 vy cc s cn tm c dng b = 877 + 1001k.
Vy 877 l s cn tm. 

Chuyn S hc Din n Ton hc


94 5.3. H phng trnh ng d bc nht mt n

V d 5.8 (Chn i tuyn KHTN). Xt h ng d gm 3 phng


trnh:

xy 1 (mod z) (5.1)
yz 1 (mod x) (5.2)
xz 1 (mod y) (5.3)

Hy tm s b (x, y, z)nguyn dng phn bit vi1 trong 3 s l 19.4


Li gii. T ba phng trnh, theo tnh cht ng d ta ln lt c
. . .
xy + 1..z v yz 1..x v zx 1..y
Suy ra

.
(xy + 1)(yz 1)(zx 1)..xyz
.
x2 y 2 z 2 x2 yz xy 2 z + xyz 2 + xy yz zx + 1..xyz
.
xy yz zx + 1..xyz

Nhn thy do x, y, z nguyn dng cho nn xyz 1. Suy ra xy yz


zx + 1 2xyz
Mt khc yz + zx xy 1 2xyz (yz + zx xy 1) 2xyz
Do ta c bt phng trnh kp 2xyz xy yz zx + 1 2xyz
.
M xyyzzx+1..xyz xyyzzx+1 = 2xyz, 1xyz, 0, 1xyz, 2xyz
Trng hp 1: xy yz zx + 1 = 2xyz xy 1 (mod z), yz 1
(mod x), zx 1 (mod y)
Cho nn ta ch cn tm nghim ca xy yz zx + 1 = 2xyz l xong.
V x, y, z c mt s bng 19 nn ta thay ln lt vo.
Nu x = 19 19y yz 19z + 1 = 38yz 39yz 19y + 19z = 1
(39y + 19)(39z 19) = 322 Vi y = 19 hoc z = 19 th tng t.
Trng hp 2,3,4,5: xyyzzx+1 = 1xyz, 0, 1xyz, 2xyz lm hon
ton tng t, ta y c v phng trnh c dng au+bv = ab+uv+x
vi x l hng s.
a v (a v)(b u) = x v gii kiu phng trnh c s. Bi ton
hon tt. 

Din n Ton hc Chuyn S hc


5.4. Bc ca phng trnh ng d 95

Nhn xt. Bi ton ny m khng cho iu kin mt s bng 19 th


khng a c dng au + bv = ab + uv + x (a v)(b u) = x lc
suy ra v hn nghim.

5.4 Bc ca phng trnh ng d


nh ngha 5.3 Xt phng trnh ng d f (x) = 0 (mod m) vi
f (x) = a0 xn + a1 xn1 + ... + an , ai N, i = 0, 1, ..., n
Nu a0 khng ng d 0 (mod m) th ta ni n l bc ca phng trnh
ng d. 4

V d 5.9. Xc nh bc ca phng trnh 15x6 8x4 + x2 + 6x + 8 0


(mod 3) 4
Li gii. Ta thy 15 0 (mod 3) nn bc ca phng trnh khng
phi l bc 6. Phng trnh trn tng ng vi 8x4 + x2 + 2 0
(mod 3)
V 8 6 0 (mod 3) nn bc phng trnh l n = 4. 

5.5 Bi tp
Bi 1. Gii cc phng trnh sau: a) 7x 6 (mod 13) b) (a + b)x
a2 + b2 (mod ab) vi (a, b) = 1 c) 17x 13 (mod 11) d) x2 +
x 2 1 (mod 3)


x1 (mod 3)
x4 (mod 4)

Bi 2. Gii cc h phng trnh: a)

x2 (mod 7)
x9 (mod 11)


5x 1 (mod 12)
b) 5x 2 (mod 8)
7x 3 (mod 11)

Bi 3. Tm a nguyn h phng trnh sau c nghim

Chuyn S hc Din n Ton hc


96 5.6. ng dng nh l Euler gii phng trnh ng d



x 3 (mod 3)
x 1 (mod 4)

a)

x 11 (mod 7)
x a (mod 11)


2x a (mod 3)
b)
3x 4 (mod 10)

Bi 4. Mt lp gm 40 hc sinh ng thnh vng trn v quay mt


v trong vng trn chi bng. Mi hc sinh nhn c bng
phi nm qua mt 6 bn bn tay tri mnh. Chng minh rng
tt c hc sinh trong lp u nhn c bng nm ti mnh
sau 40 ln nm bng lin tip.

5.6 ng dng nh l Euler gii phng trnh


ng d
Qua bi vit ny ti xin gii thiu mt phng php gii phng
trnh ng d bng cch khai thc nh l Euler

Trc ht, xin nhc li vi kin thc quen thuc.

nh ngha 5.4 Hm Euler (m) vi s nguyn dng m l cc s t


nhin nh hn m l cc s nguyn t vi m. 4

5.6.1 nh l Euler.
nh l 5.1 (Euler) Cho m l s nguyn dng v (a, m) = 1 th
a(m) 1 (mod m)

Hm c tnh cht sau:

(mn) = (m)(n) vi (m; n) = 1

Nu p nguyn t (p) = p 1; (pn ) = pn pn1 (n > 1)

Din n Ton hc Chuyn S hc


5.6. ng dng nh l Euler gii phng trnh ng d 97

k
Nu m = p1 1 p2 2 ...pk , pi l cc s nguyn t th
    
1 1 1
(m) = m 1 1 ... 1
p1 p2 pk

By gi ta xt m = a.b trong (a; b) = 1 th c cc kt qu sau

nh l 5.2
a(b) + b(a) 1 (mod ab) (5.4)

Chng minh. Theo nh l Euler ta c: a(b) 1 (mod b) m b(a) 0


(mod b)
Nn a(b) + b(a) 1 (mod b).
Tng t ta c:a(b) + b(a) 1 (mod a)
Theo tnh cht ng d th : a(b) + b(a) 1 (mod ab) 

nh l 5.3 Gi s c k(k 2) s nguyn dng m1 ; m2 ; . . . mk v


chng nguyn t vi nhau tng i mt. t M = m1 .m2 . . . mk = mi ti
vi i = 1, 2, 3. . . , k ta c
(m1 ) (m2 ) (mk )
t1 + t2 + ... + tk 1 (mod M ) (5.5)

Chng minh. T gi thit ta c (mi , ti ) = 1 vi mi i = 1, 2, . . . , k nn


theo nh l Euler th
(m1 )
t1 1 (mod mi ) (5.6)

Mt khc vi i; j thuc tp 1;2;. . . ;k v i 6= j th tj chia ht cho mj


nn (tj ; mi ) = mi hay
(mi )
tj 0 (mod mi ) (5.7)

(m ) (m ) (m )
t S = t1 1 + t2 2 + ... + tk k
T (5.6) v (5.7) c S timi 1 (mod mi )
V m1 ; m2 ; . . . mk nguyn t vi nhau tng i mt, nn theo tnh cht
ng d thc c
S 1 0 (mod m1 .m2 ...mk ) S 1 (mod M ), tc l c (5.5). 

Chuyn S hc Din n Ton hc


98 5.6. ng dng nh l Euler gii phng trnh ng d

Khi m rng (5.4) theo hng nng ln ly tha cc s hng ta c kt


qu sau.
nh l 5.4 Vi (a, b) = 1 v n, v l hai s nguyn dng no th
an(b) + bv(a) 1 (mod ab) (5.8)
Chng minh. tin lp lun t x = a(b) .
Theo nh l Euler th x = a(b) 1 (mod b) x 1 0 (mod b)
ng thi x = a(b) 0 (mod a).
T c x(x1) 0 (mod a) v x(x1) 0 (mod b) nn x(x1) 0
(mod ab)
T x3 x2 .x x.x x2 x (mod ab) v c lp lun nh th c
xn x (mod ab) hay an(b) a(b) (mod ab)
Tng t ta c: bv(a) b(a) (mod ab) nn theo (5.4) c an(b) +
bv(a) b(a) + a(b) 1 (mod ab).
(5.8) c chng minh. 
H qu 5.1 Vi (a; b) = 1 th an(b) + bn(a) 1 (mod ab) 
H qu ny c th chng minh trc tip khi nng hai v ca h thc
(5.4) ln ly tha bc n (s dng khi trin nh thc Newton) v ch
rng ab 0 (mod ab). Nn lu rng trong ng d thc th a 6 0
(mod ab)!

Vi k hiu nh nh l 5.3 ta c ti .tj 0 (mod M ) vi i khc j v mi


i; j thuc tp 1,2,...,k (nhng t 6 0 (mod M ) vi mi i = 1, 2, 3, ...k)

T khi nng hai v ca (5.5) ln ly tha bc n ta c kt qu sau.


nh l 5.5 Vi cc gi thit nh nh l 5.3 ta c:
n(m1 ) n(m2 ) n(mk )
t1 + t2 + ... + tk 1 (mod M ) (5.9)
Vi cc k hiu nh trn ta t a = mi v b = ti th theo (5.4) c
n(ti ) n(mi )
mi + ti 1 (mod M ) (5.10)
Cng tng v ca k ng thc dng (5.10) v s dng (5.5) ta c
kt qu sau:

Din n Ton hc Chuyn S hc


5.6. ng dng nh l Euler gii phng trnh ng d 99

nh l 5.6 Vi cc gi thit nh l 5.3 ta c:


(t1 ) (t2 ) n(tk )
m1 + m2 + ... + mk k1 (mod M ) (5.11)

Khi nhn 2 v ca (??) vi mi ta c


1+(ti ) (mi )
m1 + mi .ti + mi (mod M ) (5.12)

(mi ) (mi )1 (mi )1


Do mi .ti = mi .ti .ti = M.ti nn

1+(t1 )
mi mi (mod M ), i = 1, k (5.13)

Cng tng v k ng thc dng (5.13) ta c kt qu sau:

nh l 5.7 Vi cc gi thit nh nh l 5.3 ta c:


1+(t1 ) 2+(t2 ) 1+(tk )
m1 + m2 + ... + mk m1 + m2 + ... + mk (mod M )
(5.14)

Khi nhn 2 v ca (5.10) vi ti ta c


1+(t1 ) 2+(t2 ) 1+(tk )
m1 + m2 + ... + mk m1 + m2 + ... + mk (mod M )
(5.15)
1+(mi )
ti ti (mod M ), i = 1, k (5.16)
Cng tng v ca k ng d dng (5.16) ta c kt qu sau

nh l 5.8 Vi cc gi thit nh nh l 5.3 ta c:


1+(m1 ) 1+(m2 ) 1+(mk )
t1 +t2 +...+tk t1 +t2 +...+tk (mod M ) (5.17)

Ch rng ti .tj 0 (mod M ) nn khi nng ln ly tha bc n ca


tng t1 + t2 + ... + tk ta c kt qu sau.

nh l 5.9 Vi cc gi thit nh nh l 5.3 ta c:

tn1 + tn2 + ... + tnk (t1 + t2 + ... + tk )n (mod M ) (5.18)

Chuyn S hc Din n Ton hc


100 5.6. ng dng nh l Euler gii phng trnh ng d

Kh nng tm ra cc h thc ng d mi cha phi ht mi bn


c nghin cu thm. nm r c nhng phn trn ta tm hiu
qua mt s v d sau y.

V d 5.10. Tm t nht bn nghim ca phng trnh ng d:

x3 + y 7 1 (mod 30) (5.19)

Li gii. Do 30 = 5.6 v (6; 5) = 1 nn theo (5.4) c 5(6) + 6(5) 1


(mod 30)
v (6) = (2).(3) = 2 v (5) = 4; 62 6 (mod 30).
Tng t ta c: 257 25 (mod 30) v 63 6 (mod 30) nn 63 + 257
26 + 6 1 (mod 30)
Nu phn tch 30 = 3.10 vi (3; 10) = 1 th theo (5.4) c 3(10) +10(3)
1 (mod 30). Tnh ton tng t nh trn ta c 34 + 102 1 (mod 30).
V 34 = 81 21 (mod 30) v 102 10 (mod 30) nn theo (5.8) c
(34 )3 + (102 )7 1 (mod 30) v (34 )7 + (102 )3 1 (mod 30)
Suy ra phng trnh trn c t nht bn nghim (x; y) l (25; 6); (6; 25);
(21; 10); (10; 21). 
V d 5.11. Chng minh rng phng trnh ng d sau c nghim
(x; y; z; t) khc (0; 0; 0; 0):

x4 + y 4 + z 4 + t 4 t 3 (mod 60).

Li gii. 60 = 3.4.5 v (5; 3) = 1; (5; 4) = 1; (3; 4) = 1 nn t m1 =


3; m2 = 4; m3 = 5; t1 = 15; t2 = 1; t3 = 20 theo (5.18)

154 + 124 + 204 (15 + 20 + 12)4 1 (mod 60)

V d 5.12. Tm t nht mt nghim ca phng trnh ng d x17 +


y 19 1 (mod 35) 4
7 5
Li gii. Ta c: 35 = 5.7 m (5; 7) = 1 nn theo (5.4): 5 + 7 1
(mod 35))
V (5) = 4; (7) = 6 nn 54 + 76 1 (mod 35)
Theo (5.8): 1417 + 3019 14 + 30 1 (mod 35)
Vy phng trnh ng d c t nht mt nghim (x; y) = (14; 30) 

Din n Ton hc Chuyn S hc


5.7. Bi tp 101

5.7 Bi tp
Bi 1. Chng minh rng phng trnh ng d sau c nghim (x; y; z; t)
khc (0; 0; 0; 0):
a) x3 + y 3 + z 3 t3 (mod 210)
b) x5 + y 5 + z 5 t5 (mod 1155)

Bi 2. Tm t nht mt nghim ca phng trnh ng d sau:

x11 + y 13 1 (mod 45)

Bi 3. Chng t rng mi phng trnh sau c nghim nguyn dng.


a) 2x + 3y + 5z + 7t 3 (mod 210)
b) 3x + 5y + 7z 2 (mod 105)

Chuyn S hc Din n Ton hc


Chng

6
H thng d v nh l
Thng d Trung Hoa
6.1 Mt s k hiu s dng trong bi
vit 103
6.2 H thng d 104
6.3 nh l thng d Trung Hoa 117
6.4 Bi tp ngh & gi p s 125

Nguyn nh Tng (tungc3sp)

Bi vit ny trnh by v H thng d v nh l Thng d Trung Hoa.


Mt s k hiu s dng c phc ha trong Phn 6.1. Phn 6.2 gii
thiu n bn c mt s kin thc c bn v H thng d y
v H thng d thu gn km theo bi tp ng dng. nh l Thng
d Trung Hoa km ng dng ca n gip gii quyt mt s dng ton
c trnh by trong Phn 6.3. Phn 6.4 kt thc bi vit bao gm
mt s bi tp ngh km gi hoc p s.

6.1 Mt s k hiu s dng trong bi vit


[x, y] : bi chung nh nht ca hai s nguyn dng x, y (nu
khng ni g thm).
(x, y) : c chung ln nht ca hai s nguyn x, y.
x , y (mod p): x khng ng d vi y theo module p.
H: h thng d y .

103
104 6.2. H thng d

HTG: h thng d thu gn.

P: tp cc s nguyn t.

(n): hm le ca n.

|A|: s phn t ca tp A.

{x}: phn l ca s thc x, c xc nh nh sau: {x} = x [x],


trong [x] l phn nguyn ca s thc x (l s nguyn ln nht
khng vt qu x).
n
Q
pi = p1 p2 ...pn
i=1

6.2 H thng d
6.2.1 Kin thc c bn
H thng d y
nh ngha 6.1 Cho tp A = {a1 ; a2 ; ...; an }. Gi s ri , 0 ri n 1
l s d khi chia ai cho n. Nu tp s d {r1 ; r2 ; ...; rn } trng vi tp
{0; 1; 2; ...; n 1} th ta ni A l mt h thng d y (gi tt l
H) mod n.

Nhn xt. T nh ngha, d thy:

. Nu A = {a1 ; a2 ; ...; an } lp thnh H (mod n) nu v ch nu:


i 6= j ai 6= aj (mod n).

. Nu A = {a1 ; a2 ; ...; an } l H (mod n) th t nh ngha d


dng suy ra:

Vi mi m Z, tn ti duy nht ai A sao cho ai m


(mod n).
Vi mi a Z, tp a + A = {a + a1 ; a + a2 ; ...; a + an } l
mt H (mod n).

Din n Ton hc Chuyn S hc


6.2. H thng d 105

Vi mi c Z v (c; n) = 1; tp cA = {ca1 ; ca2 ; ...; can } l


mt H (mod n).

Ch : tp A = {0; 1; 2; 3; ...; n 1} l mt H (mod n) khng m


nh nht. S phn t ca tp A l |A| = n.
V d 6.1. Cho hai H (mod n): A = {a1 ; a2 ; ...; an } v
B = {b1 ; b2 ; ...; bn }.
a. Chng minh rng: Nu n chn th tp A + B = {a1 + b1 ; a2 +
b2 ; ...; an + bn } khng hp thnh H (mod n)
b. Kt lun cu a. s th no nu n l s l 4
Li gii. a. Ta c mt iu kin cn sau y i vi H (mod n),
khi n chn. Gi s C = {c1 ; c2 ; ...; cn } l mt H (mod n). Khi
theo nh ngha ta c:

n(n + 1)
c1 + c2 + ... + cn (1 + 2 + ... + (n 1)) (mod n)
2
Do n chn nn n = 2k, suy ra:

n(n + 1) .
= k(2k + 1) 6 ..n k(2k + 1) , 0 (mod n)
2
c1 + c2 + ... + cn , 0 (mod n) (6.1)

Ta c:
A + B = {a1 + b1 ; a2 + b2 ; ...; an + bn }
{(a
 1 + a2 + ... + an ) +  (b1 + b2 + ... + bn )} (mod n)
n(n + 1) n(n + 1)
+ (mod n)
2 2
[n(n + 1)] (mod n)
A+B 0 (mod n) (6.2)

( y ta cng s dng gi thit A v B l hai H mod n).


T (6.1) v (6.2) ta suy ra pcm.

Chuyn S hc Din n Ton hc


106 6.2. H thng d

b. Xt khi n l: Lc ny cha th kt lun g v tnh cht ca h


A + B.
Tht vy, ta xt n = 3; A = {1; 2; 3} ; B = {4; 5; 6}.
Khi A + B = {5; 7; 9} l mt H mod 3.
Nhng, xt h A = {1; 2; 3} , B = {5; 4; 6}.
Khi A + B = {6; 6; 9} khng phi l mt H mod 3. 

H thng d thu gn
nh ngha 6.2 Cho tp B = {b1 ; b2 ; ...; bk } l mt tp hp gm k s
nguyn v (bi ; n) = 1 vi mi i = 1; 2; ...; k.
Gi s: bi = qi n + ri vi 1 ri < n. Khi d thy (ri ; n) = 1.
Nu tp {r1 ; r2 ; ...; rn } bng tp K gm tt c cc s nguyn dng
nh hn n v nguyn t cng nhau vi n th B c gi l h thng
d thu gn mod n, gi tt l HTG (mod n). 4

Nhn xt. Ta c th rt ra hai nhn xt:

. D thy tp B = {b1 ; b2 ; ...; bk } gm k s nguyn lp thnh mt


HTG khi v ch khi

i. (bi ; n) = 1
ii. bi 6= bj (mod n) vi 1 i 6= j k
iii. |B| = (n)

iu kin (iii) tng ng vi (iii0 ): vi mi x Z; (x; n) = 1


tn ti duy nht bi B sao cho x bi (mod n).

. T nh ngha ta suy ra: cho tp B = {b1 ; b2 ; ...; bk } l HTG mod


n v c Z; (c; n) = 1 th tp cB = {cb1 ; cb2 ; ...; cbn } cng l HTG
mod n.

Din n Ton hc Chuyn S hc


6.2. H thng d 107

V d 6.2. Cho hai s nguyn dng m, n vi (m; n) = 1. Gi s A =


{a1 , a2 , ..., ah } ; B = {b1 , b2 , ..., bk } tng ng l cc h thu gn mod m
v mod n. Xt tp hp C = {ai n + bj m} ; 1 i h; 1 j k.. Chng
minh rng C l mt h thu gn HTG mod mn. 4
Li gii. + Ta chng minh (ai n + bj m, mn) = 1 i = 1, h; j = 1, k
(iu kin (i)).
Gi s tn ti i, j v s nguyn t p l c chung ca ai n + bj m
v mn.
. .
Ta c ai n + bj m..p v mn..p.
. .
Do mn..p m (m, n) = 1 nn c th gi s n..p, suy ra

. . .
ai n..p bj m..p bj ..p

Vy p l c nguyn t chung ca n v bj . iu ny mu thun


vi gi thit. Nn iu gi s l sai. Vy (ai n+bj m, mn) = 1 i =
1, h; j = 1, k.

+ Chng minh iu kin (ii).


Gi s tn ti a A; b B sao cho an + bm a0 n + b0 m
(mod mn)

an a0 n (mod m) a a0 (mod m) (do (m, n) = 1)

(iu ny mu thun).
Vy an + bm , a0 n + b0 m (mod mn).

+ Chng minh iu kin (iii0 ).


Gi s (x, mn) = 1 (x, m) = 1; (x, n) = 1.
V (m, n) = 1 nn tp B = {mb1 , mb2 , ..., mbk } l mt HTG mod
n.
Vy tn ti duy nht b B x mb (mod n).

Chuyn S hc Din n Ton hc


108 6.2. H thng d

Tng t, tn ti duy nht a A x na (mod m).


T suy ra x na + mb (mod n) v x na + mb (mod m).
T kt hp vi (m, n) = 1 suy ra x na + mb (mod mn). 

Nhn xt. T y, ta c th suy ra cng thc tnh hm le (n).

6.2.2 ng dng
Trong cc bi ton v a thc, dy s
V d 6.3. [THTT, s 340] Cho p l s nguyn t l v a thc Q(x) =
(p 1)xp x 1. Chng minh rng tn ti v hn s nguyn dng a
sao cho Q(a) chia ht cho pp . 4
Li gii. Thay cho vic chng minh tn ti v hn s nguyn dng a
sao cho Q(a) chia ht cho pp , ta s chng minh tp

H = {Q(1); Q(2); ...; Q(pp )}

l mt H mod pp .
Ta c nhn xt sau: trong tp s {1; 2; ...; pp } gm pp s, gi s c hai
s u, v khc nhau th Q(u) , Q(v) (mod pp ).
Ta chng minh iu ny bng phn chng. Gi s c Q(u) Q(v)
(mod pp )

(p 1)up u 1 (p 1)v p v 1 (mod pp )

(p 1)(up v p ) (u v) 0 (mod p) (6.3)


Theo nh l Ferma nh th up u (mod p) v v p vp (mod p) vi p
l s nguyn t nn up v p u v (mod p).
T (6.3) suy ra

(p 2)(u v) 0 (mod p) u v (mod p) (6.4)

Cng t (6.3) ta c:

(u v)((p 1)(up1 + up2 v + ... + uv p2 + v p1 ) 1) 0 (mod pp )

Din n Ton hc Chuyn S hc


6.2. H thng d 109

Kt hp vi (6.4) suy ra

(u v)((p 1).p.up1 1) 0 (mod pp ) u v 0 (mod pp )

iu ny mu thun vi gi s u , v (mod pp ). Vy nhn xt c


chng minh.

T nhn xt trn suy ra H = {Q(1); Q(2); ...; Q(pp )} l mt H


mod pp . T suy ra trong tp s {1; 2; ...; pp } gm pp s th tn
.
ti duy nht mt s a sao cho Q(a) 0 (mod pp ) hay Q(a)..pp .

Ta xt dy s hng ak = a + k.pp vi k = 0, 1, 2..., d thy rng:

Q(ap ) Q(a) 0 (mod pp ).

.
Ngha l tn ti v hn s ak (k = 0, 1, 2, ...) tha mn Q(ak )..pp .


V d 6.4. Cho a thc P (x) = x3 11x2 87x + m. Chng minh rng


vi mi s nguyn m, tn ti s nguyn n sao cho P (n) chia ht cho
191. 4
Li gii. tng cng tng t V d 6.3, ta s s dng H. Trc
ht ta a ra b sau:

B 6.1 Cho p l s nguyn t, p 2 (mod 3). Khi ,vi mi s


nguyn x, y m x3 y 3 (mod p) x y (mod p) 
Chng minh. Tht vy:

Nu x 0 (mod p) y 3 0 (mod p) y 0 (mod p) x


y(modp)

Nu x, y cng khng chia ht cho p, do p 2(mod3) p =


3k + 2(k Z).

Chuyn S hc Din n Ton hc


110 6.2. H thng d

Theo nh l Ferma:

xp1 = x3k+1 1 (mod p)


y p1 = y 3k+1 1 (mod p)
3k+1
x y 3k+1 (mod p) (6.5)

M theo gi thit, x3 y 3 mod p x3k y 3k (mod p).


T suy ra x y (mod p). Vy b c chng minh. 
Tr li bi ton, ta s chng minh P (n1 ) P (n2 ) (mod 191) vi
n1 ; n2 Z th n1 n2 (mod 191).
Tht vy, v
27P (n1 ) = (3n1 11)3 11.191.n1 + 113 + 27m
27P (n2 ) = (3n2 11)3 11.191.n2 + 113 + 27m
nn
P (n1 ) P (n2 ) (mod 191)
27P (n1 ) 27P (n2 ) (mod 191)
(3n1 11)3 (3n2 11)3 (mod 191)
3n1 11 3n2 11 (mod 191)(suy ra t b )
n1 n2 (mod 191)
Vi mi n1 , n2 A = {1; 2; 3; ...; 1991} (A l mt H mod 191),
n1 6= n2 ta c P (n1 ) , P (n2 ) (mod 191)
A = {P (1); P (2); ...; P (191)} l mt H mod 191.
T suy ra n A = {1; 2; 3; ...; 191} sao cho
.
P (n) 191 (mod 191) P (n)..191
. 
V d 6.5. Cho p l mt s nguyn t. Chng minh rng vi mi s m
nguyn khng m bt k, lun tn ti mt a thc Q(x) c h s nguyn
sao cho pm l c chung ln nht ca cc s an = (p + 1)n +Q(n); n =
1, 2, 3... 4

Din n Ton hc Chuyn S hc


6.2. H thng d 111

Li gii. Ta c b sau:

.
B 6.2 k N, k < m th tn ti bk Z sao cho bk pm + pk ..k! 
Chng minh. Gi s k! = pk Mk vi (Mk ; p) = 1. 
Khi e chy trong tp {0; 1; ...; Mk 1} th cc s epmk lp thnh

mt H modMk , thnh th tn ti bk Z sao cho bk pmk 1


(mod Mk )

.
(bk pmk + 1)..Mk
.
(b pm + pk )..pk .M
k k

Mt khc
 
X k X k
k < <k
pi pi
i=1 i=1

.
Vy (bk pm + pk )..pk .Mk = k!. B c chng minh. 

Tr v bi ton.  i 
x(x 1)...(x i + 1) Cn nu n i
t fi (x) = th fi (n) = .
i! 0 nu n < i
m1
fi (x)(bi pm + pi ) th theo B 6.2, R(x) l a thc
P
t R(x) =
i=0
c h s nguyn.
Ta c:
n
X m1
X m1
X
n
un = (p + 1) + R(n) = Cni pi i
fi (n)p p m
fi (n)bi
i=0 i=1 i=0

m1
fi (n)pi fi (n)pi (mod pm )
P P

i=0 i=1

(n)pi (mod pm ) n = 1, 2, 3...
P
fi 0
i=0

c bit u1 = (p + 1) + R(1) = epm

Chuyn S hc Din n Ton hc


112 6.2. H thng d

Ta chng minh a thc Q(x) = R(x)+pm (1e) l a thc cn tm.Tht


vy,

an = (p + 1)n + Q(n) = (p + 1)n + R(n) + pm (1 e)


.
= un + pm (1 e)..pm , n = 1, 2, 3... (6.6)

Mt khc
.
a1 = (p + 1) + Q(1) = p + 1 + R(1) + pm (1 e) = epm + pm (1 e)..pm

Do pm l CLN ca an vi mi n = 1, 2, 3... 

V d 6.6. Cho p 3 l mt s nguyn t v a1 , a2 , ..., ap2 l mt dy


cc s nguyn dng sao cho p khng l c s ca ak v ak k 1 vi
mi k = 1, 2, 3, ..., p 2. Chng minh rng tn ti mt s phn t trong
dy a1 , a2 , ..., ap2 c tch ng d vi 2 module p. 4
Li gii. Ta c b sau:

B 6.3 Vi mi s nguyn k = 1, 2, ..., p 1 tn ti mt tp cc s


nguyn {bk,1 , bk,2 , ..., bk,k } tha mn hai iu kin sau:

1. Mi bk,j hoc bng 1, hoc bng tch ca mt s phn t trong dy


a1 , a2 , ..., ap2 ,

2. bk,i , bk,j (mod p) vi 1 i 6= j k. 

Chng minh. Vi k=2 chn b21 = 1; b22 = a1 , 1 (mod p) (do a11 1


khng chia ht cho p).
Gi s vi 2 k p 2 ta chn c tp {bk,1 , bk,2 , ..., bk,k } tha
mn hai tnh cht trn.
.
V ak 6 ..p nn hai phn t khc nhau bt k trong tp

{ak bk,1 , ak bk,2 , ..., ak bk,k }

l phn bit theo mod p.

akk , 1(modp) (ak bk,1 )(ak bk,2 )...(ak bk,k ) , bk,1 bk,2 ...bk,k (mod p)

Din n Ton hc Chuyn S hc


6.2. H thng d 113

T hai iu trn suy ra tn ti ch s j(1 j k) sao cho ak bk,j


/
{bk,1 , bk,2 , ..., bk,k }.
Xt tp {bk,1 , bk,2 , ..., bk,k , ak bk,j }.
Sau khi nh s li cc phn t ta thu c tp

{bk+1,1 , bk+1,2 , ..., bk+1,k , bk+1,k+1 }

. Ta thy tp ny c k + 1 phn t tha mn hai tnh cht trn nn


theo nguyn l quy np, b c chng minh. 
Quay li bi ton, p dng b 6.3, xt tp {bp1,1 , bp1,2 , ..., bp1,p1 },
ta thy tp ny l mt HTG mod p nn n cha ng mt phn t
ng d vi 2 mod p. V phn t ny khc 1 nn n phi ng d vi
tch ca mt s ak . Suy ra pcm. 

Trong tp con tp s nguyn dng, bi ton s hc chia ht


V d 6.7. Cho p > 3 l s nguyn t c dng 3k + 2.
a. Chng minh rng tp A = 23 1; 33 1; 43 1; ...; p3 1 l


HTG mod p.
p
(i2 + i + 3) 3(modp).
Q
b. Chng minh rng 4
i=1

Li gii. a. Ta s chng minh tp A tha mn 3 iu kin nu


nh ngha 6.2.

Hin nhin mi phn t ca A u khng chia ht cho p


(tha mn iu kin (i)).
Gi s tn ti 1 i < j p 1 sao cho
i3 1 j 3 1 (mod p)
i3 j 3 (mod p)
i3k j 3k (mod p)

Mt khc, theo nh l Ferma, ta c: i3k+1 j 3k+1 (mod p)


T suy ra i j (mod p) i = j (mu thun). Vy A
tha mn iu kin (ii).

Chuyn S hc Din n Ton hc


114 6.2. H thng d

V (p) = p 1 = |A| nn iu kin (iii) tha mn. 

Vy A l mt HTG mod p.
b. V B = {1; 2; 3; ...; p 1} l mt HTG mod p. M A cng l mt
HTG mod p (theo phn a.) nn ta c:
p
(i3 1) (p 1)!
Q
(mod p)
i=2
p
(i2 + i + 1) 1
Q
(mod p)
i=2
p
(i2 + i + 1) 3
Q
(mod p)
i=1

Nhn xt. Ta c th m rng V d 6.7 nh sau:


V d 6.8. Cho p l s nguyn t l c dng mk + 2 (m, k l cc s
nguyn dng, m > 2). Tm s d ca php chia
p
Y
T = (tm1 + tm2 + ... + t + 1)
t=1

cho p. 4
V d 6.9. Chng minh rng vi mi s nguyn dng n, tn ti s t
nhin n gm n ch s u l v n chia ht cho 5n. 4
n +
Li gii. Xt s xn = a1 a2 ...an = 5 .a tha mn (vi ai Z l vi
mi i = 1, 2, ..., n v a Z+ )
Ta s chng minh bi ton bng phng php quy np ton hc.
.
Vi n = 1 a = 5..51 . Vy mnh ng vi n = 1.
1
Gi s mnh ng vi n xn = a1 a2 ...an = 5n .a, cn chng minh
mnh ng vi n + 1.
Xt 5 s sau y:
a1 = 1a1 a2 ...an = 5n (1.2n + a)
a2 = 3a1 a2 ...an = 5n (3.2n + a)
a3 = 5a1 a2 ...an = 5n (5.2n + a)
a4 = 7a1 a2 ...an = 5n (7.2n + a)
a5 = 9a1 a2 ...an = 5n (9.2n + a)

Din n Ton hc Chuyn S hc


6.2. H thng d 115

Do B = {1, 3, 5, 7, 9} l mt H mod 5 cho nn

B = {1.2n + 1; 3.2n + a; 5.2n + a; 7.2n + a; 9.2n + a}

cng l H mod 5 nn tn ti duy nht mt s trong B chia ht


cho 5.
Trong 5 s a1 ; a2 ; a3 ; a4 ; a5 c duy nht mt s chia ht cho 5(n + 1)
m s ny gm n + 1 ch s l. Vy mnh ng vi n + 1.
Theo nguyn l quy np, mnh ng vi mi n nguyn dng. Vy
vi mi s nguyn dng n, lun tn ti mt s t nhin gm n ch s
u l v chia ht cho 5n. 

Trong mt s dng ton S hc khc


Ngoi cc ng dng nu trn, h thng d cn c dng trong nhiu
dng ton s hc khc, n biu nh trong cc bi ton lin quan ti
tnh tng, gii phng trnh nghim nguyn (phng trnh Diophant
bc nht). Sau y xin nu ra mt s v d.

V d 6.10. Vi mi cp s nguyn t cng nhau (p,q), t


     
q 2q (p 1)q
S= + + ... +
p p p
(p 1)(q 1)
a. Chng minh rng: S =
2
b. Xc nh gi tr ca p, q S l s nguyn t 4
 
kq rk
Li gii. a. Ta c = , y rk l s d trong php chia q
p q
cho p (0 rk p 1).
Ta c:
 
q 2q (p 1)q r1 r2 rp1
S= + + ... + + + ... +
p p p p p p

V (p, q) = 1 rk 6= 0 k = 1, 2, ..., p 1, t ta thy tp A =


{r1 ; r2 ; ...; rp1 } chnh l mt hon v ca tp A = {1; 2; ...; p 1}.

Chuyn S hc Din n Ton hc


116 6.2. H thng d

Tht vy, ngc li, gi s i, j {1; 2; ...; p 1} , i < j m


ri = rj
( (
1jip2 jip2
. . (v l)
(j i)q ..p j i..p

Ta c:

r1 r2 rp1 1 + 2 + ... + p 1 p1
+ + ... + = =
p p p p 2
(p 1)(q 1)
S= (6.7)
2

b. T (6.7) suy ra S l s nguyn t cn c p, q > 1 v t nht


mt trong hai s p, q l.

Trng hp 1: p, q cng l p, q 3, p =
6 q (do (p,q)=1),
kt hp vi (6.7) S l s chn ln hn 2 S khng phi
l s nguyn t.
Trng hp 2: p l s chn, q l s l


(p, q) = 1

p1=1

q1 P

SP
2

(p, q) = 1
p1P


q1 =1

 2

p=2
 q = 2h + 1 (h P)

(6.8)
q=3
p = t + 1 (t P, t , 2 (mod 3))

Din n Ton hc Chuyn S hc


6.3. nh l thng d Trung Hoa 117

Trng hp 3: q l s chn, p l s l. Tng t trng hp


2, ta c:

p = 2m + 1(m P)
 q=2

(6.9)
p=3
q = n + 1(n P, n , 2 (mod 3))

T (6.8) v (6.9) ta c cc cp s p, q cn tm. 

V d 6.11. Cho a, b, c l cc s nguyn dng tha mn a b c


v (a, b, c) = 1. Chng minh rng nu n > ac + b th phng trnh
n = ax + by + cz c nghim nguyn dng. 4
d
Li gii. Gi (a, c) = d (b, d) = 1 A = {bi}i=1 l H mod d
.
y {1, 2, ..., d} sao cho by n (modd) (n by)..d.
Do (a, c) = d a = a1 d; c = c1 d (a1 , c1 Z + ; (a1 , c2 ) = 1) B =
{a1 j}cj=1
1
l H mod c1 .
n by
x {1, 2, ..., c1 } sao cho a1 x (mod c1 ) z Z sao cho
d
n by
= a1 x + c1 z.
d
Mt khc, ta c:
n by ac + b by ca1 b
> = (d 1) + a1 c1 a1 c1 a1 x z Z+
d d d
T y suy ra n by = ax + cz n = ax + by + cz.
Vy nu n > ac+b th phng trnh n = ax+by +cz c nghim nguyn
dng. 

6.3 nh l thng d Trung Hoa


6.3.1 Kin thc c bn
nh l 6.1 Cho k s nguyn dng n1 , n2 , ..., nk i mt ngun t
cng nhau v k s nguyn bt k a1 , a2 , ..., ak . Khi tn ti s nguyn
a tha mn a ai (mod ni ), i = 1, k.

Chuyn S hc Din n Ton hc


118 6.3. nh l thng d Trung Hoa

S nguyn b tha mn b ai (mod ni ), i = 1, k khi v ch khi b a


(mod n) vi n = n1 n2 ...nk . 
n
Li gii. t n = n1 n2 ...nk v t Ni = .
ni
Do (ni , nj ) = 1, i 6= j nn suy ra (Ni , ni ) = 1 i = 1; k.
Do (Ni , ni ) = 1, i = 1; k nn vi mi i(1 i k) tn ti bi sao
cho
Ni bi 1 (mod ni ) (6.10)

Nh vy ta c b b1 , b2 , ..., bk . Do Nj 0 (mod ni ) khi i 6= j, t


d nhin suy ra

Nj bj 0 (mod ni ) (6.11)

k
P
t a = Nj bj aj .
j=1

Vi mi i (1 i k) ta c
k
X
a = Ni bi ai + Nj bj aj (6.12)
j=1;j6=i

T (6.10),(6.11),(6.12) suy ra a ai (mod ni ), i = 1, k.

D thy, v n1 , n2 , ..., nk i mt nguyn t cng nhau nn ta c


kt lun sau: S nguyn b tha mn b ai (mod ni ), i = 1, k
khi v ch khi b a (mod n) vi n = n1 n2 ...nk . 

Nhn xt. 1. Ngoi cch chng minh trn, ta cn c th s dng


php quy np chng minh nh l thng d Trung Hoa.

2. nh l Thng d Trung Hoa khng nh v s tn ti duy nht


ca mt lp thng d cc s nguyn tha mn ng thi nhiu
ng d tuyn tnh. Do c th dng nh l gii quyt nhng
bi ton v s tn ti v m cc s nguyn tha mn mt h cc

Din n Ton hc Chuyn S hc


6.3. nh l thng d Trung Hoa 119

iu kin quan h, chia ht,..., hay m s nghim ca phng


trnh ng d. Vic s dng hp l cc b v (trong nh l) cho
ta rt nhiu kt qu th v v t c th a ra nhiu bi ton
hay v kh.

V d 6.12. Cho m1 , m2 , ..., mn l cc s nguyn dng, r1 , r2 , ..., rn


l cc s nguyn bt k. Chng minh rng iu kin cn v h
phng trnh ng d

x r1 (mod m1 )
x r2 (mod m2 )
...
x rn (mod mn )

c nghim l ri rj (mod GCD (mi , mj )); 1 i < j n.

Nu x0 v x1 l hai nghim tha mn h phng trnh trn th x0 x1


(mod m) vi m = LCM (m1 , m2 , ..., mn ). Tc l h phng trnh
cho c nghim duy nht theo module m. 4

Li gii. Trc ht ta gi s h phng trnh cho c nghim x0 . t


GCD (mi , mj ) = d, ta c:

xo ri 0 (mod mi )
xo rj 0 (mod mj )

Suy ra ri rj mod (GCD (mi , mj )). Do i, j ty chn nn ri rj


(mod GCD(mi , mj )), 1 i < j n. y l iu kin cn h
phng trnh c nghim.
Ngc li, ta s chng minh bng quy np theo n rng nu iu kin
trn c tha mn th h phng trnh lun c nghim duy nht theo
module m vi m = LCM (m1 , m2 , ..., mn ).
Vi trng hp n = 2, t GCD (m1 , m2 ) = d m1 = dd1 ; m2 = dd2
vi GCD (d1 , d2 ) = 1.
Suy ra ri rj r (mod d). t r1 = r + k1 d; r2 = r + k2 d.

Chuyn S hc Din n Ton hc


120 6.3. nh l thng d Trung Hoa

Ta c:

.
(x r) k1 d..dd1

x r1 (mod m1 )


x r2 (mod m2 ) (x r) k d...dd
2 2
xr
k2 (mod d1 )
d (6.13)
xr k (mod d2 )
2
d
Do (d1 , d2 ) = 1 nn theo nh l Thng d Trung Hoa, tn ti mt s
xr
dng x sao cho x k1 (mod d1 ); x k2 (mod d2 ). V x v
 d
x k1 (mod d1 ) xr
l hai nghim ca phng trnh nn x
x k2 (mod d2 ) d
(mod d1 d2 ) hay x xd + r (mod dd1 d2 ).
Do m = LCM (m1 , m2 ) = dd1 d2 nn theo nh l Thng d Trung Hoa,
h c nghim duy nht module m.
Gi s nh l ng n n 1. Ta s chng minh nh l ng n n.
0
t m1 = LCM (m1 , m2 , ..., mn1 ) ; m02 = mn ; r20 = rn . V ri
rj (modGCD (mi , mj ))  vi mi 1 i < j n nn theo gi thit quy
x ri (mod mi )
np, h phng trnh c duy nht nghim x r10
i = 1, n 1
(mod m01 ).
Mt khc t ri rj ( mod GCD(mi , mj )) vi mi 1 i < j n suy ra
0 0 0
r1 r2 (mod GCD(m01 , m2 )).
Theo chng minh trn cho trng hp n = 2 ta c h phng trnh
x r10 (mod m01 )

c nghim duy nht theo module
x r20 (mod m02 )
m = LCM m01 , m02 = LCM (m1 , m2 , ..., mn )


. Theo nguyn l quy np ta c iu phi chng minh. 


Nhn xt. y chnh l nh l Thng d Trung Hoa dng m rng,
n hon ton chng minh da trn c s nh l Thng d Trung Hoa.
Trong bi vit ny, ta s khng i su vo tm hiu nh l dng m
rng m ch i su vo cc ng dng ca nh l Thng d Trung Hoa
(dng thng).

Din n Ton hc Chuyn S hc


6.3. nh l thng d Trung Hoa 121

6.3.2 ng dng
Trong L thuyt s
V d 6.13. Chng minh rng vi mi s t nhin n, tn ti n s t
nhin lin tip m mi s trong n s u l hp s. 4
Li gii. tng: ta s to ra mt h phng trnh ng d gm n
phng trnh ng d. Da vo nh l thng d Trung Hoa, ta kt
lun c s tn ti nghim ca h .
Gi s p1 , p2 , ..., pn l n s nguyn t khc nhau tng i mt.
Xt h phng trnh ng d x k (mod p2k )(k = 1, 2, ..., n).
Theo nh l thng d Trung Hoa, tn ti x0 N sao cho x0 k
(mod p2k ), k = 1, 2, ..., n.
Khi cc s x0 + 1; x0 + 2, ...; x0 + n u l hp s.(pcm) 

V d 6.14. Chng minh rng vi mi s t nhin n, tn ti n s t


nhin lin tip sao cho bt k s no trong cc s cng u khng
phi ly tha (vi s m nguyn dng) ca mt s nguyn t. 4

Nhn xt. Bi ny cng gn tng t vi tng ca bi ton v d


cng c. Tuy nhin vic tm ra h phng trnh ng d kh hn mt
cht.
Li gii. Vi mi s t nhin n, xt n s nguyn t khc nhau tng i
mt p1 , p2 , ..., pn .
Theo nh l Thng d Trung Hoa, tn ti a N sao cho a pk k
(mod p2k ) (k = 1, 2, ..., n).
Khi d thy rng cc s a + 1, a + 2, ..., a + n u khng phi ly
tha vi s m nguyn dng ca mt s nguyn t (pcm). 

V d 6.15. Cho trc cc s nguyn dng n, s. Chng minh rng tn


ti n s nguyn dng lin tip m mi s u c c l ly tha bc
s ca mt s nguyn dng ln hn 1. 4
n
Li gii. Xt dy Fn = 22 + 1, (n = 0, 1, 2, ...). D chng minh b
sau:

B 6.4 Nu n 6= m th (Fn , Fm ) = 1. 

Chuyn S hc Din n Ton hc


122 6.3. nh l thng d Trung Hoa

p dng nh l Thng d Trung Hoa cho n s nguyn t cng nhau


F1s , F2s , ..., Fns v n s ri = i(i = 1, 2, .., n) ta c tn ti s nguyn c
.
sao cho c + i..F s .i
Vy dy {c + i}ni=1 l n s nguyn dng lin tip, s hng th i chia
ht cho Fis . 

V d 6.16. Chng minh rng tn ti mt a thc P (x) Z[x], khng


c nghim nguyn sao cho vi mi s nguyn dng n, tn ti s nguyn
x sao cho P (x) chia ht cho n. 4
Li gii. Ta c th xt a thc P (x) = (3x + 1)(2x + 1).
Vi mi s nguyn dng n, ta biu din n di dng n = 2k (2m + 1).
V GCD(2k , 3) = 1 nn tn ti a sao cho 3a 1 (mod 2k ). T

3x 1 (mod 2k ) x a (mod 2k )

Tng t GCD (2, 2m+1) = 1 nn tn ti b sao cho 2b 1 (mod (2m+


1)). T

2x 1 (mod (2m + 1)) x b (mod (2m + 1))

Cui cng, do GCD (2k , 2m + 1) = 1 nn theo nh l Thng d Trung


Hoa, tn ti s nguyn x l nghim ca h:

x a (mod 2k )


x b (mod (2m + 1))

.
V theo l lun trn, P (x) = (3x + 1)(2x + 1)..n. 

V d 6.17. Trong li im nguyn ca mt phng ta Oxy, mt


im A vi ta (x0 , y0 ) Z2 c gi l nhn thy t O nu on
thng OA khng cha im nguyn no khc ngoi A, O. Chng minh
rng vi mi n nguyn dng ln ty , tn ti hnh vung n n c cc
nh nguyn, hn na tt c cc im nguyn nm bn trong v trn
bin ca hnh vung u khng nhn thy c t O. 4

Din n Ton hc Chuyn S hc


6.3. nh l thng d Trung Hoa 123

Li gii. D thy iu kin cn v im A(x0 , y0 ) nhn thy c


t O l gcd(x0 , y0 ) = 1.
gii quyt bi ton, ta s xy dng mt hnh vung n n vi n
nguyn dng ln ty sao cho vi mi im nguyn (x, y) nm trong
hoc trn hnh vung u khng th nhn thy c t O.
Tht vy, chn pij l cc s nguyn t i mt khc nhau vi 0 i, j
n. Xt hai h ng d sau:


x 0 (mod p01 p02 ...p0n )
x+1 0 (mod p11 p12 ...p1n )


x+2 0 (mod p21 p22 ...p2n )
...




x+n 0 (mod pn1 pn2 ...pnn )

v

y 0 (mod p01 p02 ...p0n )
y + 1 0 (mod p11 p12 ...p1n )



y+2 0 (mod p21 p22 ...p2n )
...




y+n 0 (mod pn1 pn2 ...pnn )

Theo nh l Thng d Trung Hoa th tn ti (x0 , y0 ) tha mn hai h


ng d trn.
Khi , r rng gcd(x0 + i, y0 + i) > 1, i, j = 0, 1, 2, ..., n.
iu c ngha l mi im nm bn trong hoc trn bin hnh vung
n n xc nh bi im pha di bn tri l (x0 , y0 ) u khng th
nhn thy c t O. Bi ton c chng minh. 

Trong tm s lng nghim nguyn ca mt phng trnh


nghim nguyn
V d 6.18. Cho s nguyn dng n = p1 1 p2 2 ...pk k , trong p1 , p2 , ..., pk
l cc s nguyn t i mt khc nhau. Tm s nghim ca phng
trnh:
x2 + x 0 (mod n)

Chuyn S hc Din n Ton hc


124 6.3. nh l thng d Trung Hoa

Li gii. Ta c:

x(x + 1) 0 (mod pi i )

2
x +x0 (mod n)
i = 1, k
" i
x 0 (mod pi )

i
x 1 (mod pi ) (6.14)

i = 1, k

Theo nh lThng d Trung Hoa, mi h phng trnh x2 + x 0



x ai (mod pi i )
(mod n) ai {1; 0} c duy nht mt nghim v ta c 2k
i = 1, k

h (bng s b (a1 , a2 , ..., ak ), ai {1; 0}), nghim ca cc h khc
nhau. Suy ra phng trnh cho c ng 2k nghim. 

V d 6.19. Cho m = 20072008 . Hi c tt c bao nhiu s t nhin


n<m sao cho m|n(2n + 1)(5n + 2) . 4
Li gii. D thy GCD (m; 10) = 1. Do :

n(2n + 1)(5n + 2) 0 (mod m)


(6.15)
10n(10n + 5)(10n + 4) 0 (mod m)

Ta c: m = 34016 .2232008 . cho thun tin, t 10n = x; 34016 =


q1 ; 2232008 = q2 .
Khi GCD (q1 , q2 ) = 1 nn (6.15) tng ng vi:

x(x + 5)(x + 4) 0 (mod q1 ) (6.16)


x(x + 5)(x + 4) 0 (mod q2 ) (6.17)

D thy:

(6.16) xy ra khi v ch khi x 0 (mod q1 ) hoc x 5 (mod q1 )


hoc x 4 (mod q1 ).

(6.17) xy ra khi v ch khi x 0 (mod q2 ) hoc x 5 (mod q2 )


hoc x 4 (mod q2 ).

Din n Ton hc Chuyn S hc


6.4. Bi tp ngh & gi p s 125

Do t (6.16) v (6.17), vi lu rng x 0 (mod 10), suy ra n l


x
s t nhin tha mn cc iu kin bi khi v ch khi n = , vi x
10
l s nguyn tha mn h iu kin sau:


x 0 (mod 10)
1 (mod q1 )
x


x r2 (mod q2 ) (6.18)
0 x < 10q q

1 2



r1 , r2 {0; 4; 5}

V 10; q1 ; q2 i mt nguyn t cng nhau nn theo nh l Thng d


Trung Hoa, h (6.18) c nghim duy nht.
D thy s c 9 s x l nghim ca 9 h (6.18) tng ng. V mi s x
cho ta mt s n v hai s x cho hai s n khc nhau nn c 9 s n tha
mn cc iu kin bi. 

Nhn xt. V d 6.19 chnh l trng hp c bit ca bi ton tng


qut sau:

V d 6.20. Cho s nguyn dng n c phn tch tiu chun n =


p1 1 p2 2 ...pk k . Xt a thc P (x) c h s nguyn. Nghim
 x0 ca phng

trnh ng d P (x) 0 (mod n) l lp ng d x0 0, 1, 2, ..., n 1
tha mn P (x0 ) 0 (mod n). Khi , iu kin cn v phng
trnh P (x) 0 (mod n) c nghim l vi mi i = 1, 2, ..., s, phng
trnh P (x) 0 (mod pi i ) c nghim. Hn na, nu vi mi i =
1, 2, ..., s, phng trnh P (x) 0 (mod pi i ) c ri nghim module pi i
th phng trnh c r = r1 r2 ...rs nghim module n. 4

6.4 Bi tp ngh & gi p s


Bi tp ngh
Bi 1. a. Chng minh rng: Nu (a, m) = 1 v x chy qua mt
h thng d y modulo m th ax + b, vi b l mt
s nguyn ty , cng chy qua mt h thng d y
module m.

Chuyn S hc Din n Ton hc


126 6.4. Bi tp ngh & gi p s

b. Chng minh rng: Nu (a, m) = 1 v x chy qua mt h


thng d thu gn modulo m th ax cng chy qua mt h
thng d thu gn module m.

Bi 2. Mi s nguyn dng T c gi l s tam gic nu n c dng


k(k + 1)
T = , trong k l mt s nguyn dng. Chng minh
2
rng tn ti mt H module n gm n s tam gic.

Bi 3. a. Cho m1 , m2 l hai s nguyn dng nguyn t cng nhau.


Chng minh rng:

(m1 m2 ) = (m1 ).(m2 )

b. Gi s s nguyn dng m c phn tch chnh tc thnh


tch cc tha s nguyn t m = p1 1 p2 2 ...pk k . Chng minh
rng:

(m) = p11 1 p2 2 1 ...pk k 1 (p1 1)(p2 2)...(pk 1)

Bi 4. Tnh tng sau:


2012
X 17k

S=
11
k=6

Bi 5. Cho s nguyn dng n v s nguyn t p ln hn n + 1. Chng


x x2 xp
minh rng a thc P (x) = 1 + + + ... +
n + 1 2n + 1 pn + 1
khng c nghim nguyn.

Bi 6. Cho p l s nguyn t c dng 3k + 2 (k nguyn dng). Tm


p
(k 2 + k + 1) cho p.
P
s d khi chia S =
k=1

Bi 7. Cho cc s nguyn dng a, b tha mn (a, b) = 1. Chng minh


rng phng trnh ax + by = 1 c v s nghim nguyn (x, y)
v (x, a) = (y, b) = 1.

Din n Ton hc Chuyn S hc


6.4. Bi tp ngh & gi p s 127

Bi 8. Tm s nguyn dng nh nht c tnh cht: chia 7 d 5, chia


11 d 7, chia 13 d 3.

Bi 9. Chng minh rng tn ti mt dy tng {an } n=1 cc s t nhin


sao cho vi mi s t nhin k, dy {k + an } ch cha hu hn
cc s nguyn t.

Bi 10. S nguyn dng n c gi l c tnh cht P nu nh vi cc


. .
s nguyn dng a, b m a3 b + 1..n th a3 + b..n. Chng minh
rng s cc s nguyn dng c tnh cht P khng vt qu 24.

Bi 11. Tm tt c cc s t nhin n tha mn 2n 1 chia ht cho 3 v


2n 1
c mt s nguyn m m |4m2 + 1.
3
Bi 12. Chng minh rng tn ti s t nhin k sao cho tt c cc s
k.2n + 1 (n = 1, 2, ...) u l hp s.

Gi p s
Bi 1. Chng minh trc tip da vo nh ngha.

Bi 2. Ta chng minh n phi c dng n = 2k . Phn chng, gi s


n = 2k .m vi m l v m > 1. S dng tnh cht h thng d
y .

Bi 3. Ta c th chng minh da vo kin thc v h thng d y


, cng c th chng minh da vo nh l Thng d trung
Hoa.

Bi 4. S dng HTG.

Bi 5. Biu din P (x) di dng P (x) = ap xp + ap1 xp1 + ... + a2 x2 +


a1 x + a0 . Phn chng, gi s P (x) c nghim nguyn x = u.
Suy ra mu thun.

Bi 6. Tin hnh tng t V d 6.7.

Bi 7. S dng kin thc H.

Chuyn S hc Din n Ton hc


128 6.4. Bi tp ngh & gi p s

Bi 8. p s: 887.

Bi 9. Gi pk l s nguyn t th k, k > 0. Theo nh l Thng d


Trung Hoa, tn ti dy s {an }n=1 tha mn a1 = 2; an =
k(modpk+1 ), k n.

Bi 10. nh l Thng d Trung Hoa.

Bi 11. Chng minh n c dng 2k . S dng tnh cht ca s Fecma


(xem li V d 6.15).

Bi 12. V d 6.15 v Bi 3.

Din n Ton hc Chuyn S hc


Chng

7
Mt s bi ton s hc
hay trn VMF
.
7.1 m3 + 17..3n 129
7.2 c(ac + 1)2 = (5c + 2)(2c + b) 136

Phn ny gm mt s bi ton hay c tho lun nhiu trn Din


n Ton hc. Bn c c th vo trc tip topic ca bi ton
trn Din n Ton hc, bng cch click vo tiu ca bi ton
.

.
7.1 m3 + 17..3n
Bi ton 7.1. Chng minh rng vi mi s nguyn dng n, tn ti
mt s t nhin m sao cho
.
m3 + 17 ..3n

. 4

u tin, chng ta n vi chng minh xut cho bi ton u bi.


Chng minh. Ta s chng minh bi ton bng quy np.
Vi n = 1, ta chn m = 4.
Vi n = 2, ta chn m = 1.
.
Gi s bi ton ng n n = k, hay m N : m3 + 17..3k
Ta chng minh rng i vi trng hp n = k + 1 cng ng tc l tn
.
ti mt s m0 sao cho m03 + 17..3k+1 .
.
t m3 + 17 = 3k .n n 6 ..3.

129
.
130 7.1. m3 + 17..3n

m3 + 17 2.3k 
 
n2 k+1

(mod3) mod3
n1 m3 + 17 3k

Trng hp 1: m3 + 17 2.3k (mod 3k+1 )


Xt:

(m + 3k1 )3 = m3 + m2 3k + m32k1 + 33k3 m3 + m2 3k (mod 3k+1 )

. .
(Do k 2 32k1 ..3k+1 v 33k3 ..3k+1 ).
Suy ra:
 3
m + 3k1 + 17 m3 + m2 .3k + 17 2.3k + m2 .3k 0 (mod 3k+1 )

. . .
(v m 6 ..3 m2 1 (mod 3) 2 + m2 ..3 (2 + m2 ).3k ..3k+1 ).
3 .
Nh vy, trng hp 1, ta c: m + 3k1 + 17..3k+1 .
Trng hp 2: m3 + 17 3k (mod 3k+1 ).
Xt:
 3
m 3k1 = m3 m2 3k +m32k1 33k3 m3 m2 3k (mod 3k+1 )

. .
(Do k 2 32k1 ..3k+1 v 33k3 ..3k+1 ).
Suy ra:
 3
m 3k1 + 17 m3 m2 3k + 17 3k m2 3k 0 (mod 3k+1 )

. .  .
(v m 6 ..3 m2 1 (mod 3) 1 m2 ..3 1 m2 .3k ..3k+1 ).
3 .
Nh vy, trng hp 2 ta c: m 3k1 + 17..3k+1 .
. .
Tm li, ta u tm c s nguyn t 6 ..3 m t3 + 17..3k+1 .
Ta chng minh c vn ng trong trng hp n = k + 1.
Theo nguyn l quy np, ta c pcm.

Mu cht bi ton ny l b sau:

Din n Ton hc Chuyn S hc


.
7.1. m3 + 17..3n 131

B 7.1 Cho a, b, q l cc s nguyn tha (a; q) = 1 v q > 0.


.
Khi y, lun tn ti k Z sao cho ak b..q. 
..
Chng minh. Ta chng minh i din cho trng hp ak + b.q. Trng
hp cn li tng t.
Xt A = {1; 2; 3; ...; q} l 1 h y H mod q.
Theo tnh cht ca H thng d, ta c tp B = {a; 2a; 3a; ...; qa} cng
l H mod q.
C = {a + b; 2a + b; 3a + b; ...; qa + b} cng l H mod q.
.
Do , tn ti k [1; q] sao cho ak + b..q. 

Nhn xt. Bi ton cho thc cht l yu cu tm 1 s x nguyn sao


.
cho x + 17..3n v x l lp phng 1 s nguyn. B trn cho thy
.
s tn ti ca x nguyn x + 17..3n . Cn vic tm x l x l lp
phng 1 s nguyn th ta s dng phng php quy np nh trn.
i vi 1 ngi yu ton, ta phi khng ngng sng to. Ta hy th
tng qut bi ton cho:

thay v m3 , ta th thay mk vi k l s nguyn dng c nh.

thay v 3n , ta th thay pn vi p l 1 s nguyn t.

thay s 17 bi y N vi y c nh.

Kt hp cc thay i trn, ta c 1 bi ton "tng qut" hn

D on 7.1 Cho p l s nguyn t. y, k N v y, k c nh.


Khng nh hoc ph nh mnh sau

.
n N, x N : xk + y ..pn (7.1)

Ta th thay mt vi gi tr p, k, y vo th xem (7.1) c ng khng.

Khi thay k = 2, y = 1, p = 3 th mnh (7.1) tr thnh

.
n N, x N : x2 + 1..3n (7.2)

Chuyn S hc Din n Ton hc


.
132 7.1. m3 + 17..3n

Rt tic, khi ny, (7.2) li sai!!!. Ta s chng minh (7.2) sai khi n 1.

Tht vy, chng minh d on 7.1 sai, ta cn c b sau


B 7.2 Cho p l s nguyn t dng 4k + 3 v a, b Z. Khi
. . .
a2 + b2 ..p (a..p) (b..p)
. .
T (7.2), suy ra x2 + 1..3. p dng b 7.2 vi p = 3, ta suy ra 1..3:
v l.
.
Vy khi n 1 th 6 x Z : x2 + 1..3n .

Khng nn lng, ta th thm mt vi iu kin (7.1) tr nn cht


hn v ng. Nu bn c c kin no hay, xin hy gi vo topic ny
tho lun. Sau khi thm mt s iu kin, ta c 1 bi ton hp hn
nhng lun ng.
nh l 7.1 Cho p nguyn t l. y, k N v y, k c nh.
Bit rng gcd(k, p) = gcd(k, p 1) = gcd(y, p) = 1.
Chng minh rng:
.
n N, x N : xk + y ..pn (7.3)
Chng minh. Trc ht, chng minh (7.3), ta cn c b sau
B 7.3 Cho p l s nguyn t l. k nguyn dng tha
(k; p) = (k 1; p) = 1
Khi , {1k ; 2k ; ...; (p 1)k } l HTG modp. 
Chng minh. Gi g l cn nguyn thy ca p tc l ordp (g) = p 1.
Khi y th g 1 , g 2 , ..., g p1 lp thnh 1 HTG modp v r rng
g a1 , g a2 , ..., g ap1 l HTG modp a1 , a2 , .., ap1 l H ca p 1.
Vi 1 i p 1 th tn ti ai m i g ai (mod p) v r rng
ai lp thnh 1 HTG modp nn h 1k , 2k , .., (p 1)k c th vit li l
g k , g 2k , ..., g (p1)k , n l HTG modp khi v ch khi k, 2k, ..., (p 1)k l
h thng d y ca p 1, tc l k nguyn t cng nhau vi p 1.
B c chng minh. 

Din n Ton hc Chuyn S hc


.
7.1. m3 + 17..3n 133

Quay li bi ton. Ta chng minh (7.3) bng phng php quy np.
Vi n = 1, theo b 7.3 th
.
x0 {1; 2; ...; p 1} : xk0 y (mod p) xk0 + y ..p
.
Gi s bi ton ng nn hay tn ti xk + y ..pn
.
Ta s chng minh n + 1 cng ng hay tn ti xk0 + y ..pn+1
Tht vy, t gi thit quy np suy ra xk + y = pn .q
.
Trng hp 1: q ..p pcm
Trng hp 2:
gcd(q, p) = 1 (7.4)
Khi ta chn x0 = v.pn + x
Do

xk0 + y = (v.pn + x)k + y


   
k nk 1 k1 n(k1) k1
= v .p + .v .p .x + ... + .v.pn .xk1 + (xk + y)
k k
(7.5)

D dng chng minh


   
n+1 k nk 1 k1 n(k1) k2
p | v .p + .v .p .x + ... .v 2 .p2n .xk2
k k

Do vy ta xt
 
k1
.v.pn .xk1 + (xk + y) = k.v.pn .xk1 + pn .q = pn (k.v.xk1 + q)
k
.
Nhn thy gi s k.xk1 t (mod p) m gcd(k, p) = 1 v xk + y ..p
gcd(x, p) = 1 (do gcd(y, p) = 1) suy ra gcd(t, p) = 1
Do (k.v.xk1 +q) tv+q (mod p) m t (7.4) ta c gcd(q, p) = 1
.
Cho nn lun tn ti v tha mn tv + q ..p. Do bi ton c khng
inh vi n + 1.
Theo nguyn l quy np, bi ton c chng minh.

Chuyn S hc Din n Ton hc


.
134 7.1. m3 + 17..3n

Cha dng li y, nu trong (7.3), ta thay k bi x, ta s c 1 bi


ton khc:

nh l 7.2 Cho p nguyn t l. y N v y c nh. Bit rng


gcd(y, p) = 1. Khi :

.
n N, x N : xx + y ..pn (7.6)

Chng minh. Ta chng minh bi ton ny bng phng php quy


np. Ta coi nh l 7.1 nh 1 b . D thy nu x tha (7.6) th
gcd(x; p) = 1.
Khi , vi n = 1, ta xt h ng d (I)

x k (mod (p 1))
x x0 (mod p)
.
trong , x0 ; k N tha xk0 + y ..p.
Do gcd(p 1; p) = 1 nn theo nh l Thng d Trung Hoa th h (I)
lun c nghim x0 .
Chn x = x0 , ta chng minh x tha (7.6) khi n = 1. Tht vy

gcd(x; p) = 1 xp1 1 (mod p) xk xx (mod p)


xx + y xk + y xk0 + y 0 (mod p)
.
Vy x N : xx + y ..p.
.
Gi s (7.6) ng n n 1, tc l tn ti x0 x0 x0 + y ..pn1 .
Theo cch chng minh quy np (7.6), ta chn c xn = apn + x0
.
tha xx0 + y ..pn .
n
Khi , d nhn thy xn x0 (mod pn1 ). Ta xt h ng d (II)

X x0 (mod (pn1 (p 1)))




X xn (mod pn )

Do gcd(pn1 (p 1); pn ) = 1 nn theo nh l Th ng d Trung hoa, h


(II) c nghim X. Ta chng minh x = X tha (7.6). Tht vy
Do (p 1)pn1 = (pn ) X X X x0 (mod pn ) (nh l Euler).

Din n Ton hc Chuyn S hc


.
7.1. m3 + 17..3n 135

Mt khc X x0 xn x0 (mod pn ) (do cch chn trong h (II)).

X X + y x n x0 + y 0 (mod pn )
Theo nguyn l quy np, bi ton c chng minh. 

M rng ca bi ton u vn cn nhiu, nh tng thm iu kin


. .
chn nh (m3 + 17..3n ) (m3 + 17 6 ..3n+1 ), v.v. Rt mong nhn c
kin ng gp cho vic m rng.

Li cm n
Rt cm n Nguyen Lam Thinh, Karl Heinrich Marx,nguyenta98, The
Gunner ng gp kin v m rng cho bi vit ny.

Chuyn S hc Din n Ton hc


136 7.2. c(ac + 1)2 = (5c + 2)(2c + b)

7.2 c(ac + 1)2 = (5c + 2)(2c + b)


Bi ton 7.2. Cho 3 s nguyn dng a; b; c tho mn ng thc:
c(ac + 1)2 = (5c + 2b)(2c + b) (7.7)
Chng minh rng : c l s chnh phng l. 4
Nhn xt. Thot nhn vo bi ton, tht kh tm 1 phng php
cho loi ny. Nhn xt trong gi thit VP (7.7), th b xut hin vi
bc l 2. Th l ta c 1 hng ngh l dng tam thc bc 2 cho bi ton
ny. Ta khng nn chn c v bc ca c l 3, khng chn a v phng
trnh mi theo a hin nhin tr li (7.7)
Chng minh (Chng minh 1).
c (ac + 1)2 = (5c + 2b) (2c + b)
2
2b2 + 9bc + 10c 2
 c (ac + 1) = 0
b = 81c2 4.2. 10c2 c (ac + 1)2 = c2 + 8c (ac + 1)2
h i
b = c c + 8 (ac + 1)2 = x2 , (x N )

+ 1)2 ) d|8 (ac + 1)2


)
d = GCD(c; c + 8(ac
t   d|8
d|c (ac + 1)2 ; d = 1
c c 
Trng hp 1: d=8 ; + (ac + 1)2 = 1
8 8
h i c c   x 2
c c + 8 (ac + 1)2 = x2 (x N) . + (ac + 1)2 =
8c  8c  8
8|x x = 8x2 (x2 N ) . + (ac + 1)2 = x22
c 8 8
= t2
t; p N
 
8
c + (ac + 1)2 = p2 (t; p) = 1
8 
c = 8t2
2
t2 + 8t2 a + 1 = p2
M d chng minh
2 2 2
8t2 a + 1 < t2 + 8t2 a + 1 < 8t2 a + 2
2 2
8t2 a + 1 < p2 < 8t2 a + 2 : mu thun

Din n Ton hc Chuyn S hc


7.2. c(ac + 1)2 = (5c + 2)(2c + b) 137

Do , d = 8 b loi.
c c 
Trng hp 2: d=4 ; + 2 (ac + 1)2 = 1
4 4
c c 2
; + 2 (ac + 1) l nhng s chnh phng (*)
4 4
c c .
Nu l s chn + 2 (ac + 1)2 ..2
 c4 c 4
2
; + 2 (ac + 1) = 2: mu thun.
4 4
c c c
Do , l s l. M l s chnh phng 1 (mod 4)
4 4 4
Mt khc, do c chn nn ac + 1 l s l (ac + 1)2 1 (mod 4)
c
+ 2 (ac + 1)2 1 + 2.1 3 (mod 4): v l do (*).
4
Do , d = 4 b loi.
Trng hp 3: d=2 .
c c
Tng t t Trng hp 2, ta c l 1 (mod 8)
2 2
c chn nn ac + 1 l (ac + 1)2 1 (mod 8)

c
+ 4(ac + 1)2 1 + 4.1 5 (mod 8) : v l
2

Do , d = 2 b loi.
Trng hp 4: d=1
Tng t trng hp 2, ta c ngay c l v do (c; c + 8(ac + 1)2 ) = 1
nn c l s chnh phng.
Vy ta c pcm. 

Nhn xt. Ta thy trong bi ny, b v c c 1 mi lin quan kh cht


ch vi nhau nn ta th gii theo b, c s dng k thut GCD tc l t
d = GCD(b; c) ta c cch chng minh th 2.

m; n N
  
c = dm
Chng minh (Chng minh 2). t d = (b; c)
b = dn (m; n) = 1

Chuyn S hc Din n Ton hc


138 7.2. c(ac + 1)2 = (5c + 2)(2c + b)

Khi
2
(7.7) m (dam + 1) = d (5m + 2n) (2m + n)
d|m (dam + 1)2
d|m m = dp (p; n) = (d; n) = 1
(d; dam + 1) = 1
2
(7.7) p d2 ap + 1 = (5dp+ 2n) (2dp + n)


p| (5dp + 2n) (2dp + n)


p|5dp + 2n p|2n
(p; 2dp + n) = 1
(p; n) = 1 p|2 p {1; 2}
2
Trng hp 1: p=2 , khi 2 2ad2 + 1 = (10d + 2n) (4d + n),
2
suy ra 2ad2 + 1 = (5d + n) (4d + n). Nhng v (5d + n; 4d + n) =
(d; 4d + n) = (d; n) = 1 Cho nn ta phi c

5d + n = x2

(x; y N , (x; y) = 1)
4d + n = y 2

Suy ra d = x2 y 2 . Mt khc
xy 1 xy 1
2ad2 + 1 = xy a = =
2d2
2 (x2 y 2 )2
2
Ta chng minh 2 x2 y 2 > (x + y)2 > xy 1
Tht vy

(x + y)2 4xy > xy 1  


2
2 x2 y 2 (x + y)2 = (x + y)2 2 (x y)2 1 > 0
2
2 x2 y 2 > xy 1 a < 1 : Tri gt

Vy p = 2 b loi.
Trng hp 2: p=1

c = d2 , (i)

d=m
b = dn
2
(7.7) d2 ad2 + 1 = 5d2 + 2dn 2d2 + dn
 

2
ad2 + 1 = (5d + 2n) (2d + n) (7.8)

Din n Ton hc Chuyn S hc


7.2. c(ac + 1)2 = (5c + 2)(2c + b) 139

(5d+ 2n; 2d + n) = (d; 2d + n) = (d;


 n)= 1
5d + 2n = x2 x; y N d = x2 2y 2

2d + n = y 2 (x; y) = 1 n = 5y 2 2x2
d = 4z 2 2y 2


Nu x = 2z vi z N
n = 5y 2 8z 2
2 2
(7.8) ad2 + 1 = 4z 2 y 2 a 4z 2 2y 2 + 1 = 2zy
Phng trnh cui cng v nghim nguyn do 2 v khc tnh chn l.
Suy ra, x l d l c l. (ii)
Kt lun: (i), (ii) c l s chnh phng l. 

Khng ngng tm kim, ta s tm mt li gii khc sc tch hn. Nu


ta bit n cng c vp (n) th s thy n s rt hiu qu cho bi ton
ny, ta c cch chng minh th v sau.

Chng minh (Chng minh 3). Gi s c chn khi ta c:

v2 (c) = v2 (5c + 2b) + v2 (2c + b)

Nu b l th ta c v2 (c) = v2 (5c + 2b) = v2 (5c) v2 (5c) < v2 (2b) = 1.


iu ny v l!
Do c l. Xt p|c l mt c nguyn t ca c.
Ta c vp (c) = vp (5c + 2b) + vp (2c + b).
Ta thy rng vp (c) > vp (5c + 2b), vp (2c + b) > 0.
Do vp (5c + 2b) = min[vp (c); vp (4c + 2b)]
vp (5c + 2b) = vp (4c + 2b) = vp (2c + b)
vp (c) = 2vp (5c + 2b): s chn nn suy ra c l s chnh phng. 

V hi vng cn nhng li gii khc hay hn, sng to hn t cc bn.


Mong bn c tho lun thm v ng gp kin cho bi ton.

Li cm n
Rt cm n Karl Heinrich Marx,nguyenta98, Vng Nguyn Thy
Dng v perfectstrong ng gp kin cho bi vit ny.

Chuyn S hc Din n Ton hc


Ti liu tham kho

[1] V Hu Bnh,Phng trnh nghim nguyn v kinh nghim gii

[2] Phan Huy Khi, Cc chuyn bi dng hc sinh gii ton trung
hc. Chuyn 5: Phng trnh nghim nguyn

[3] Phm Minh Phng v nhm tc gi chuyn ton i hc S phm


H Ni, Cc chuyn S hc bi dng hc sinh gii Trung hc
c s

[4] Titu Andreescu, Dorin Andrica, Number Theory: Structures, Ex-


amples and Problems

[5] Tp ch Ton Tui Th, Ton hc v Tui tr, Mathematical Re-


flections, v.v

[6] Cc thi hc sinh gii, tuyn sinh vo THPT, TST, IMO,v.v

[7] Ti nguyn Internet, c bit:


http://diendantoanhoc.net/forum/,
http://www.artofproblemsolving.com/,
http://boxmath.vn

[8] Gv THPT chuyn HKHTN H Ni,Bi ging S hc

[9] ng Hng Thng, ng d v phng trnh ng d

[10] Phan Huy Khi, Cc bi ton c bn ca S hc

[11] H Huy Khoi, Chuyn bi dng HSG THPT S Hc

[12] K yu ca cc hi tho Ton hc, Tp ch Ton hc v Tui tr,


tp ch Crux,v.v

141
142 Ti liu tham kho

[13] Nguyn Trng Nam, L thuyt ng d v ng dng trong m sa


sai

Din n Ton hc Chuyn S hc

You might also like